NBME 26

¡Supera tus tareas y exámenes ahora con Quizwiz!

44 ---------- Exam Section 1: Item 45 of 50 National Board of Medical Examiners Comprehensive Basic Science Self-Assessment 45. A 70-year-old man with poorly controlled hypertension is brought to the emergency department 30 minutes after the sudden onset of weakness of the left side of his face and body. He is alert and able to follow commands. His pulse is 80/min, and blood pressure is 190/90 mm Hg. Physical examination shows mild flattening of the nasolabial fold on the left side and weakness and hyperreflexia of the left upper and lower extremities. There are no sensory deficits, but he has mild dysarthria. Six months later, histologic examination of the area of the injury is most likely to show proliferation of which of the following cell types? A) Astrocytes B) Ependymal cells C) Microglial cells O D) Neurons E) Oligodendrocytes

A. In the aftermath of a cerebral infarction (stroke), several histologic changes occur that culminate in a glial scar composed of proliferated astrocytes. One to 2 days after an ischemic event, neurons shrink and release lysosomal enzymes that lead to liquefactive necrosis. Three to 5 days after the event, macrophages (called microglial cells in the central nervous system) transiently proliferate and phagocytose dead neurons. Within weeks, astrocytes migrate to the site of injury (as opposed to fibroblasts in the peripheral nervous system) and begin to proliferate. After 2 weeks, the ischemic area becomes occupied by a permanent glial scar, which is composed of a dense network of neuroglial cells (astrocytes) and fluid-filled cystic spaces. Incorrect Answers: B, C, D, and E. Ependymal cells (Choice B) form the permeable lining of the ventricles that separates the CSF system and brain parenchyma. They do not migrate and are not involved in post-stroke histologic changes. Microglial cells (Choice C), derived from precursor monocytes, proliferate and phagocytose dead neurons days after an ischemic event and may remain in the region weeks after the initial injury. Microglial cells would not be present 6 months after the event. Neurons (Choice D) undergo liquefactive necrosis after a stroke. They are permanently differentiated cells, similar to cardiac myocytes, and are unable to re-enter the cell cycle for regeneration. Therefore, few neurons would be seen on histologic examination after 6 months. Oligodendrocytes (Choice E) are glial cells that produce myelin in the central nervous system. These cells are damaged in strokes. They slowly regenerate over months but would not be expected to multiply to the degree that scar-forming astrocytes proliferate. Educational Objective: After a ce bral infarction, neurons undergo liq

26 ---------- Exam Section 1: Item 27 of 50 National Board of Medical Examiners Comprehensive Basic Science Self-Assessment 27. A 32-year-old woman comes to the physician 1 day after she noticed a large red lesion on her back. One week ago, she returned from vacation in Cape Cod. After evaluation, a diagnosis of erythema chronicum migrans is made. Prior to prescribing an antibiotic, the physician asks the patient if she could possibly be pregnant because an adverse effect of the antibiotic active against the causal organism is incorporation into fetal bones and teeth, causing yellow discoloration. The most likely mechanism of this drug is inhibition of which of the following? O A) Attachment of tRNA to ribosomes B} Dihydrofelate reductase C) DNA repair by DNA gyrase Đ} Electron tranAspert E) Transpeptidation of peptidoglycan

A. Lyme disease is a tick-borne disease caused by Borrelia burgdorferi endemic in the Northeastern United States. In its early stages, it causes erythema chronicum migrans, which is an expanding rash that may have a central ring-like clearing. It can also be associated with a flu-like illness. In the early stages of Lyme disease, it can be treated with doxycycline, which acts by preventing the attachment of aminoacyl-tRNA to ribosomes. Side effects of tetracycline antibiotics include gastrointestinal distress, photosensitivity, and in children, discoloration of teeth and inhibition of bone growth, which makes it contraindicated in pregnancy. Later stages of Lyme disease can be characterized by carditis, atrioventricular heart block, facial nerve palsy (which is often bilateral), migratory arthralgias, and if left untreated, encephalopathy and chronic arthritis. Later stages of Lyme disease that affect the heart or central nervous system are generally treated with parenteral ceftriaxone. Incorrect Answers: B, C, D, and E. Dihydrofolate reductase (Choice B) is an enzyme used in the synthesis of purines and thymidine, precursors of nucleic acids. Inhibition of dihydrofolate reductase by trimethoprim prevents folate synthesis and is bacteriostatic. It is avoided in the first trimester of pregnancy but has no role in treating Lyme disease. DNA gyrase (Choice C) functions in DNA synthesis and repair by relaxing supercoils during replication and transcription. Fluoroquinolones inhibit prokaryotic DNA gyrase (also known as topoisomerase), side effects of which include cartilage damage, tendonitis, or tendon rupture. Inhibitors of electron transport (Choice D) such as antimycin A inhibit the proton gradient and prevent normal synthesis of ATP. Doxycycline does not inhibit electron transport. Transpeptidation of peptidoglycan (C

17 ---------- Exam Section 1: Item 18 of 50 National Board of Medical Examiners Comprehensive Basic Science Self-Assessment 18. A 35-year-old man is brought to the emergency department because of multiple injuries sustained in a motor vehicle collision 45 minutes ago. He is lethargic and confused. The right pupil is sluggishly reactive to light and is 1 mm larger than the left pupil. There is mild left hemiparesis. Further examination shows fracture of the right femur, ruptured spleen, and hemorrhagic pleural effusion. A CT scan of the head shows a subdural hematoma on the right with mass effect and midline shift. He is intubated and mechanically hyperventilated. Which of the following is the most likely rationale for hyperventilation in this patient? A) Cause cerebral vasoconstriction B) Decrease pulmonary atelectasis C) Increase cerebral blood flow Đ} Increase intracranial pressure E) Increase oxygenation

A. The intracranial compartment is a fixed compartment surrounded by a rigid skull containing brain parenchyma, cerebrospinal fluid, and intravascular blood. Pathologic lesions that take up space in the intracranial compartment such as tumors, abscesses, and extravascular blood such as subdural or epidural hematoma may displace normal intracranial structures, causing mass effect and midline shift. Cerebral blood flow (CBF) supplies intravascular blood in the intracranial space. Hypocapnia induces cerebral vasoconstriction, decreasing the cerebral blood flow and the volume of intravascular, intracranial blood. Therapeutic mechanical hyperventilation induces hypocapnia and has been shown to reduce intracranial pressure in the acute setting. This method for intracranial pressure reduction is not a long-term solution; rather, it is a temporizing measure prior to definitive treatment such as a craniotomy. Incorrect Answers: B, C, D, and E. Hyperventilation may decrease in pulmonary atelectasis (Choice B), however decrease in pulmonary atelectasis does not correlate with increased ventilation and hypocapnia directly. Decreasing atelectasis may be accomplished through incentive spirometry, or in the case of a mechanically-ventilated patient, through increased positive end-expiratory pressure (PEEP). Increased cerebral blood flow (Choice C) and increased intracranial pressure (Choice D) would not be helpful in this patient with a subdural hematoma displacing intracranial structures. The intracranial compartment is rigid, and increased blood flow or intracranial pressure would cause further mass effect in this patient. Hyperventilation decreases cerebral blood flow and intracranial pressure through inducing cerebral vasoconstriction. Hyperventilation has little effect on increasing oxygenation (Choice E). The oxygenation of a m

34 ---------- Exam Section 1: Item 35 of 50 National Board of Medical Examiners Comprehensive Basic Science Self-Assessment 35. A newborn is delivered at 40 weeks' gestation to a 20-year-old woman after an uncomplicated pregnancy. Physical examination shows male-like external genitalia, with clitoral hypertrophy and labial fusion. The karyotype is 46,XX. The serum concentration of cortisol is decreased and serum concentrations of 17-hydroxyprogesterone, androstenedione, and testosterone are increased. Which of the following is the most likely cause of the findings in this newborn? AJAndrogen receptor mutation B) Defect in adrenal steroid biosynthesis 67 Deficiency ofenzyme activity in-gonadal steroidhermone biosynthesis Đ} Increased anti-müllerian hormone secretion E) Translocation of the SRY gene to one of the X chromosomes

B. Congenital adrenal hyperplasia (CAH) is a defect in adrenal steroid biosynthesis, particularly of cortisol, with variable effects on mineralocorticoids and androgens. The most common form of CAH is 21-hydroxylase deficiency, which presents with salt wasting in a newborn or precocious puberty in a child, as well as virilization of infants with an XX karyotype (eg, clitoral hypertrophy, labial fusion), due to increased levels of androgens, as seen in this patient. Due to the enzyme deficiency, levels of 17-hydroxyprogesterone, androstenedione, and testosterone are increased, while aldosterone and cortisol are decreased, leading to hypotension and hyperkalemia. Treatment consists of exogenous glucocorticoid and mineralocorticoid supplementation. Incorrect Answers: A, C, D, and E. Androgen receptor mutations (Choice A) would lead to androgen insensitivity in genotypic males, which presents as normal-appearing female external genitalia, decreased or absent body hair, and lack of a uterus and ovaries. Patients will have functioning testes and testosterone levels will be increased. However, as this biological female is presenting with virilization, androgen receptor mutations are an unlikely diagnosis. Deficiency of enzyme activity in gonadal steroid hormone biosynthesis (Choice C) would lead to decreased levels of estradiol, progesterone, and testosterone, which is not seen in this patient. Increased anti-müllerian hormone secretion (Choice D) or müllerian inhibitory factor would lead to the absence of development of the female internal reproductive organs in this patient but would not cause abnormalities in cortisol levels. Translocation of the SRY gene to one of the X chromosomes (Choice E) would cause a fetus with an XX karyotype to develop as a biologic male, including external genitalia and testes. Hormone levels w

25 ---------- Exam Section 1: Item 26 of 50 National, Board of Medical Examiners Comprehensive Basic Science Self-Assessment 26. A health care screening for diabetes mellitus is conducted at a weekend market with subjects including both men and women ranging in age from 25 to 95 years. Blood samples are obtained, and the blood glucose concentrations are determined. The results are shown in the table: Patient Glucose (mg/dL) 70 120 120 140 110 95 75 145 105 10 85 85 12 90 120 100 190 13 14 15 Which of the following best represents the median blood glucose concentration for this group (in mg/dL)? A) 95 B) 105 C) 110 D) 120 E) 125

B. Measures of central tendency within distributions of data include the mean, median, and mode. The median is the middle value of listed data points ordered from least to greatest. If the dataset contains an even number of values, the median is considered the mean of the middle two values. The mode is the most frequently occurring value within a dataset. The mean is calculated by the sum of the values, divided by the total number of values, and is the most affected by the presence of extreme or outlier data points. In this case, ordering the fifteen values of patient blood glucose (in mg/dL) in order yields the series 70, 75, 85, 85, 90, 95, 100, 105, 110, 120, 120, 120, 140, 145, 190. The middle value in the set is 105 mg/dL, which is the median of this dataset. Incorrect Answers: A, C, D, and E. 95 mg/dL (Choice A) and 125 mg/dL (Choice E), are neither the mean, median, nor mode of the dataset. 110 mg/dL (Choice C), is the mean of the dataset and is computed by the sum of all data points divided by the total number of data points. The sum is equal to 1,650 mg/dL. Dividing by 15 total data points yields a mean of 110 mg/dL. 120 mg/dL (Choice D), is the most frequently occurring value in the set, and is therefore the mode. Educational Objective: Measures of central tendency within distributions of data include the mean, median, and mode. The median is the middle value of listed data points ordered from least to greatest. If the dataset contains an even number of values, the median is considered the mean of the middle two values. 123 4 567∞ o =N345

76 ---------- Exam Section 2: Item 26 of 50 National Board of Medical Examiners ansive Basic Saance amont Y 26. A 32-year-old woman comes to the physician because of a 3-month history of headaches. Two weeks ago, she moved to an area women's shelter because her husband had been hitting her. She takes no medications. Physical examination shows multiple bruises in different stages of healing over the face, neck, and trunk. There is circumferential bruising on both sides of the neck. Which of the following statements is most appropriate to gather pertinent information from this patient? O A) "Did you bring your records from the emergency room with you?" B) "I see some bruising on your neck. How did that happen?" C) "Please demonstrate to me how this neck bruising occurred." %3D O D) "This looks like you were choked. Is that true?" E) "What sort of device did your husband use on your neck?"

B. Physicians should ask direct, open-ended questions about physical examination findings indicative of abuse. Physicians should start by summarizing their objective observations and end with a question about the findings. To protect patients' well-being, physicians have an obligation to investigate evidence of abuse. Additionally, abuse may cause physical or psychological injury that may require diagnosis and treatment. Physicians should treat the immediate symptoms, which may manifest as medical or psychiatric symptoms, and continue to provide regular care to monitor for long-term consequences of abuse. Physicians are mandated by many states to report abuse of children and vulnerable adults to protective agencies. Incorrect Answers: A, C, D, and E. Asking about emergency room records (Choice A) would avoid the topic of ongoing abuse and be unlikely to inform the physician about the patient's current social situation. Physicians should directly ask about concerning physical examination findings. Physicians should not ask patients to reenact possible abuse (Choice C), which may retraumatize patients. Patients should be invited to discuss abuse using open-ended questions instead. The physician should not assume that the bruises came from the husband's abuse (Choices D and E) or that the patient is willing to discuss this sensitive topic. The patient may feel intruded upon or defensive, which would preclude effective discussion. Asking an open-ended question based on an objective observation would likely increase the patient's comfort level with a discussion about the abuse. Educational Objective: Physicians should ask direct, open-ended questions about physical examination findings indicative of abuse without verbalizing assumptions about the etiology of the examination findings.

38 ---------- Exam Section 1: Item 39 of 50 National Board of Medical Examiners Comprehensive Basic Science Self-Assessment 39. An investigator is studying a laboratory strain of HIV-1. During the experiment, point mutations are introduced in the long terminal repeat region of the virus. Infection of a human T-lymphocyte line with the mutant virus results in markedly decreased viral replication compared with infection with the wild-type virus. A mutation most likely directly affected which of the following phases of the viral life cycle? A) Attachment of the virus to the cell surface B) Budding of the virus from the cell membrane C) Fusion of the viral envelope and cell membrane D) Integration of the provirus into the host cell genome E) RNA splicing to produce an mRNA encoding the envelope glycoproteins

D. Long terminal repeats (LTRS) are found at each of the 5' and 3' ends of retroviral DNA and transposons. In the case of retroviral DNA, they are formed by reverse transcription, which is required for the replication and virulence of HIV. LTRS are used to integrate the viral genome with that of the host, and in this case, integration of the provirus into the host cell genome is most directly affected by mutations within the LTRS. The LTR contains regulators of gene expression. The integrated HIV provirus 5' LTR contains the promoter sequence for transcription, while the 3' end contains the polyadenylation sequence. The viral integrase enzyme catalyzes the integration of proviral DNA with host DNA. Once viral DNA is integrated, the cell is permanently infected and carries the genome for the remainder of its life. Integrase inhibitors prevent the action of this enzyme, limiting the degree of reverse transcribed DNA that can fuse with the host genome. Incorrect Answers: A, B, C, and E. Attachment of the virus to the cell surface (Choice A) involves glycoprotein (eg, gp120), CD4, CXCR4, and CCR5 surface proteins. Mutations in host proteins may confer immunity. Maraviroc, an entry inhibitor, blocks HIV attachment to CCR5. Budding of the virus from the cell membrane (Choice B) is the final step in the release of newly synthesized HIV virions. Proteases are involved in the final packaging and release of virions, and protease inhibitors prevent successful completion of these steps. Fusion of the viral envelope and cell membrane (Choice C) is required for viral penetration into the cytoplasm of the host cell, and involves interactions between surface glycoproteins (eg, gp120, gp41) and host membrane proteins (eg, CD4). Enfuvirtide inhibits the binding of gp41 to prevent the creation of entry pores in the cell membrane. RNA spl

79 ---------- Exam Section 2: Item 29 of 50 National, Board pf Medical Examiners ment * 29. A 15-year-old girl is brought to the physician by her mother because of a 1-year history of increasingly severe abdominal pain that occurs periodically, but not regularly. Physical examination shows abdominal tenderness that is most acute around the umbilicus. The physician suspects ectopic endometrial tissue, and an operation is scheduled. During the operation, an 8-cm piece of tissue is removed from the ileal region on the antimesenteric side of the bowel approximately 90 cm proximal to the ileocecal junction. The patient's pain resolves postoperatively. The ectopic tissue was most likely located in which of the following structures? O A) Ascending colon B) Falciform ligament C) lleal polyp O D) Meckel diverticulum O E) Vermiform appendix

D. Meckel diverticulum results from persistence of the vitelline duct and can demonstrate ectopic, functional gastric, pancreatic, or, rarely, endometrial tissue. It can present with acute or chronic hematochezia or melena. It is often painless but may also present with abdominal pain. It typically presents in childhood, most commonly before age 2 years, although because of its indolent nature, it may not be diag length. In children, it may serve as a potential lead-point for intussusception. A technetium 99m pertechnetate scan demonstrates uptake in the right lower quadrant, corresponding with ectopic gastric mucosa within the Meckel diverticulum. Treatment of symptomatic cases is through surgical resection. sed until later in life. It is co only located two feet proximal to the ileocecal valve and is approxir ately two inches in Incorrect Answers: A, B, C, and E. The ascending colon (Choice A) is a common site of colon polyps. The ascending colon is distal to the ileocecal valve and is not consistent with the location of the excised tissue. The falciform ligament (Choice B) is a peritoneal fold that divides the liver; it is anatomically distant from the described location of resection in this case. lleal polyps (Choice C) are less common than their counterparts in the colon, and may be associated with hereditary polyposis syndromes, such as Peutz-Jeghers syndrome or familial adenomatous polyposis. The length of the removed tissue and its presentation with intermittent abdominal pain is more suggestive of a Meckel diverticulum. The vermiform appendix (Choice E) is the site of acute appendicitis, which presents with periumbilical pain that migrates to the right lower quadrant, fever, anorexia, nausea, vomiting, diarrhea, and leukocytosis. Educational Objective: Meckel diverticulum results from persistence of the vitell

18 ---------- Exam Section 1: Item 19 of 50 National, Board of Medical Examiners Comprehensive Basic Science Self-Assessment 19. A 52-year-old woman with a long-standing history of generalized anxiety disorder comes to the physician for a follow-up examination. She has been taking an anxiolytic drug for 12 years, but she is not currently under a psychiatrist's care because she says she is feeling better and that she prefers this physician to psychiatrists. She has hyperlipidemia, hypertension, and mild type 2 diabetes mellitus treated with pharmacotherapy. Following physical examination and laboratory studies, the physician makes adjustments to the patient's medication regimen. Two days later, the physician receives 12 calls from the patient for clarification of treatment recommendations; this pattern has occurred frequently in the past. In particular, the patient fixates on the potential toxicities of her medication

E. The physician should manage this anxious patient's frequent phone calls by establishing limits and setting a regular appointment schedule. The physician-patient relationship has professional boundaries (also known as a therapeutic frame) that both parties should respect. These boundaries include specific meeting times and relationship expectations. For instance, a physician should abstain from dominating the conversation with details about their personal life since their role is to be attentive and therapeutic to the patient. When a patient transgresses these boundaries, such as making many phone calls outside of scheduled appointments, both parties will benefit from reestablishment of expectations. Many patients who are anxious are destabilized by poor interpersonal boundaries and will respond well to structure. Incorrect Answers: A, B, C, and D. Referring the patient elsewhere (Choices A and D) without attempting to problem-solve with the patient would be premature. Boundary transgressions occur regularly in physician-patient relationships and should not warrant immediate referral, especially if the transgressions are related to an anxiety disorder the physician is managing. Due to this patient's acute anxiety and attachment to this physician, terminating the relationship would likely lead to significant distress. Using other people to mediate the interpersonal relationship (Choices B and C) would prevent the direct communication and boundary setting that is necessary in this physician-patient relationship. One therapeutic role of the physician is to demonstrate that they can handle challenges and difficult feelings that arise, which can assuage the patient's anxiety. Educational Objective: The physician-patient relationship benefits from boundaries. When a patient transgresses these boundaries, the physician shou

24 ---------- Exam Section 1: Item 25 of 50 National, Board of Medical Examiners Comprehensive Basic Science Self-Assessment 25. A 1-year-old boy is brought to the physician by his parents because of a 4-week history of progressive generalized tonic-clonic seizures and a strange odor to his urine. He has a history of delayed development. He was adopted from an orphanage in Russia at the age of 6 months. Physical examination shows fair skin and blond hair. His phenylalanine hydroxylase gene is homozygous for a point mutation (GT AT) in intron 12 of the affected gene that causes skipping of exon 12. Which of the following is the most likely explanation for exon skipping in this patient's affected gene? A) Alternative polyadenylation site B) Deletion of 5' untranslated region C) Expansion of trinucleotide repeat D) Gene duplication E) Nonhomologous recombination F) Nonsense mutation G) RNA splice error H) X inactivation

G. After DNA is transcribed into pre-messenger RNA (pre-mRNA), processing occurs to form the finished mRNA product. One element of this processing is the splicing out of introns, non-coding portions of the genetic material which do not contribute to the final protein, leaving behind only exons. From a single pre-mRNA, several unique proteins may be made if splicing variations occur. Correct splicing is dependent on the spliceosome recognizing a splice site at the start of an intron, which typically consists of base pairs G-U-A-A-G. A point mutation in DNA changing a guanine to an adenine, as in this case, may mask a splice site and either cause an intron to remain in the final mRNA product erroneously or cause a larger region of pre-mRNA to be spliced, leading to the removal of an exon. In this case, incorrect removal of exon 12 has created a defective version of phenylalanine hydroxylase and led to the condition phenylketonuria. Patients with phenylketonuria do not have the ability to convert phenylalanine to tyrosine. Accumulation of phenylalanine leads to physical manifestations including a musty odor, impaired neurologic development, and seizures. Incorrect Answers: A, B, C, D, E, F, and H. Polyadenylation of the 3' end and capping of the 5' end with non-translated nucleotides are two additional processes completed during pre-MRNA processing. An alternative polyadenylation site (Choice A) would lead to an addition of numerous adenine residues. Deletion of the 5' untranslated region (Choice B) would remove a series of nucleotides at the 5' end of the MRNA. Neither would cause the substitution at a splice site seen in this case. Expansion of a trinucleotide repeat (Choice C) occurs when a series of three nucleotides in repetition is replicated incorrectly and too many repeats are included in the replicated product. C

19 ---------- Exam Section 1: Item 20 of 50 National, Board of Medical Examiners Comprehensive Basic Science Self-Assessment 20. A healthy 35-year-old woman who wishes to become a regular platelet donor comes to a blood donation center. Blood is obtained by venous phlebotomy into sodium citrate and processed to yield platelet-rich plasma (PRP) for platelet function studies. She has no personal or family history of bleeding disorders. Aliquots of the PRP are subsequently analyzed using a light-transmittance platelet aggregometer. Adding a 5 µM solution of which of the following reagents to the PRP is most likely to cause a rapid and irreversible aggregation pattern in these studies? A) Adenosine diphosphate B) Collagen monomer C) Immune globulin D) Norepinephrine E) Prostacyclin (PG|2)

A. Adenosine diphosphate (ADP) addition to PRP will result in rapid and irreversible aggregation. ADP is critical in normal hemostasis. It acts via the receptors P2Y1 and P2Y12 on the surface of platelets. Binding to these receptors results in G-protein coupled receptor signaling, with consequent alteration of platelet shape and activation of GPII6/Illa via the PI3K signaling pathway, which results in rapid platelet aggregation. The P2Y12 receptor is the target of clopidogrel, an anti-platelet medication used commonly for the treatment of acute coronary syndrome following percutaneous coronary intervention. Incorrect Answers: B, C, D, and E. Collagen monomers (Choice B) are essential building blocks of collagen superstructures that function to help adhere platelets at the site of clots, but they do not have a role in the process of rapid and irreversible platelet aggregation. Immune globulin (Choice C) is given in intravenous form (IVIG) for various indications. It has been shown to cause platelet aggregation and thrombosis via the interaction of platelets with the Fc portion of IgG, but this reaction is considered a potential side effect of IVIG and does not occur reliably. Norepinephrine (Choice D) may induce platelet aggregation by binding to alpha-adrenergic receptors on the surface of platelets with a subsequent increased risk of microthrombi formation, but this is not a reliable effect. Prostacyclin (PG|2) (Choice E) is produced through the cyclooxygenase pathway. It inhibits the activation and aggregation of platelets and it plays a role in vasodilation. Epoprostenol is a synthetic prostacyclin that can be used in the treatment of pulmonary arterial hypertension. Educational Objective: ADP rapidly and irreversibly causes platelet aggregation through its action on the P2Y1 and P2Y12 receptors.

83 ---------- Exam Section 2: Item 33 of 50 Natjonal, Board pf Medical Examinersment andive Rasic Saance 33. A 2-year-old boy is brought to the physician because of a 6-month history of failure to thrive. Cardiac examination shows a grade 4/6 systolic murmur caused by increased pulmonic flow, followed by a fixed, widely split S2. Echocardiography shows hypertrophy of the right atrium, right ventricle, and pulmonary arteries. This patient most likely has which of the following congenital cardiac anomalies? A) Atrial septal defect O B) Patent ductus arteriosus O G) Persistent truncus arteriesus O D) Tetralogy of Fallot O E) Transposition of the great arteries O F}VentriculaF-septal defect

A. Atrial septal defect is a common congenital malformation of the interatrial septum. The most common type is an ostium secundum defect, although ostium primum defects are commonly associated with trisomy 21. The atrial septal defect results in a left-to-right shunt with abnormal flow of blood from the left atrium to the right atrium, resulting in relative volume overload of the right atrium and ventricle. This increased stroke volume of the right ventricle results in delayed closure of the pulmonic valve, which presents as a fixed, split S2, and low-grade physiologic ejection murmur on cardiac auscultation. The increased right heart volumes also result in a prominent right ventricular impulse on physical examination and may present an increased risk for the development of a right bundle branch block. Chest radiographs characteristically demonstrate increased caliber of the main pulmonary artery and increased pulmonary vascular markings. If the atrial septal defect remains uncorrected, it can result in the development of Eisenmenger syndrome secondary to prolonged pulmonary vasculature remodeling resulting in pulmonary arterial hypertension and shunt reversal leading to cyanosis. Incorrect Answers: B, C, D, E, and F. Patent ductus arteriosus (PDA) (Choice B) is a persistent conduit between the aorta and the pulmonary artery that has failed to obliterate after birth. It results in a continuous, machine-like murmur best heard in the left second intercostal space, radiating to the clavicle. Persistent truncus arteriosus (Choice results from failure of the aorticopulmonary septum to divide the aorta and pulmonary trunk. Most patients present with a concomitant ventricular septal defect and cyanosis noted in the neonatal period. Tetralogy of Fallot (Choice D) consists of pulmonary infundibular stenosis, overriding aorta, v

58 ---------- Exam Section 2: Item 9 of 50 National Board of Medical Examiners Comprehensive Basic Science Self-Assessment 9. A 45-year-old man comes to the physician because of progressively worsening, constant pain in his left thigh over the past 3 months. He is a long-distance runner. An x-ray of the femur shows thickening of the diaphysis and disruption of the cortex with focal areas of increased calcification. A glistening mass containing several cysts is surgically excised. A photomicrograph of tissue from the mass is shown. Which of the following is the most likely diagnosis? A) Atypical stress fracture B) Chondrosarcoma C) Enchondroma O D) Ewing sarcoma E) Giant cell carcinoma O F) Multilobulated bone cyst G) Osteosarcoma

B. Chondrosarcoma is a malignant neoplasm that arises from the mesenchymal cells that produce cartilage. It commonly occurs in the axial skeleton, particularly in the pelvis, and often in patients in the fifth decade of life. It is a slow-growing tumor that is initially asymptomatic, permitting extensive growth before detection. Pelvic chondrosarcoma will often present with pelvic pain and compression of adjacent structures such as the lumbosacral plexus, leading to neurologic symptoms in the ipsilateral lower extremity. Chondrosarcoma can be primary or arise secondary to

94 ---------- Exam Section 2: Item 44 of 50 Natjonal, Board of Medical Examinersment anive Rasic Science S V 44. A 75-year-old man with alcoholism is brought to the emergency department 30 minutes after he began vomiting blood. He has a history of portal hypertension. Abdominal examination shows spider angiomata. Which of the following is the most likely cause of this patient's hematemesis? O AJAnastomosis between the superior and inferior mesenteric veins O B) Aortoesophageal fistula O C) Inflammation of the distal esophagus D) Retrograde blood flow between tributaries of the left gastric to the azygos veins O E}Shunting ef blood from the portal paraumbilical veins to the superier epigastric veins

D. Cirrhosis can present with edema, ascites, increased bilirubin, jaundice, spider angiomata, and sequelae of portal hypertension such as esophageal varices, splenomegaly, caput medusae, and rectal varices. It typically occurs in patients with preceding conditions such as alcohol abuse or chronic hepatitis. In the setting of portal hypertension, there is the potential for retrograde blood flow between tributaries of the left gastric and azygos veins (portosystemic anastomosis). This results in the formation of esophageal varices, which are dilated submucosal blood vessels. Acutely bleeding esophageal varices typically present with hematemesis, and treatment requires supportive therapy with intravenous fluids and transfusions, along with attempts to stop the bleeding through endoscopic guided sclerosis or banding of the esophageal varices. A transjugular intrahepatic portal shunt can be used in the prevention of variceal bleeding by creating a shunt between the portal vein and hepatic vein, which decreases portal pressure by shunting blood to the systemic circulation, bypassing the liver. Incorrect Answers: A, B, C, and E. Anastomosis between the superior and inferior mesenteric veins (Choice A) is a normal anatomic variant. The inferior mesenteric vein normally drains into the splenic vein, which in turn combines with the superior mesenteric vein to form the portal vein. An aortoesophageal fistula (Choice B) is a life-threatening abnormal communication between the aorta and esophagus, leading to gastrointestinal bleeding and hematemesis. It can be caused by a thoracic aortic aneurysm, foreign body such as an aortic graft, esophageal malignancy, or postoperative complication. It causes massive upper gastrointestinal hemorrhage and is often fatal. It is not associated with portal hypertension. Inflammation of the distal

86 ---------- Exam Section 2: Item 36 of 50 Natjonal, Board pf Medical Examinersment Sive Basic Science 36. An 81-year-old woman is admitted to the hospital because of a massive pulmonary embolism from a deep venous thrombosis. Her platelet count is 160,000/mm3. Appropriate pharmacotherapy is started. One week later, her platelet count is 55,000/mm3. The thrombocytopenia was most likely caused by a drug with which of the following mechanisms of action? O A) Activates tissue plasminogen O B) Interferes with the carboxylation of coagulation factors O C) Irreversibly inactivates cyclooxygenase D) Potentiates the action of antithrombin III E) Selectively inhibits factor Xa

D. Heparin results in immediate anticoagulation through binding and potentiating the action of anti-thrombin III, causing the inhibition of thrombin and factor Xa. It is commonly used for anticoagulation in the setting of deep vein thrombosis, pulmonary embolism, and acute coronary syndrome. Heparin-induced thrombocytopenia (HIT) is a potential complication of heparin therapy. HIT is characterized by thrombocytopenia (a decrease of greater than 50% from baseline) that typically occurs between 5 to 10 days following heparin exposure, along with evidence of arterial or venous thrombosis. Complications include organ ischemia, skin necrosis, and limb gangrene. HIT occurs secondary to the development of IgG antibodies against the complex of heparin and platelet factor 4. This leads to the activation of platelets, which can cause thrombosis and consumption of platelets. This abnormal activation yields progressively decreasing platelet counts, which typically nadir at greater than 50% from their initial level. Treatment requires cessation of all heparin- related products (eg, unfractionated heparin and low molecular weight heparin), and the initiation of a direct thrombin inhibitor or direct Xa inhibitor to ensure adequate anticoagulation without worsening the thrombocytopenia. Incorrect Answers: A, B, C, and E. Tissue plasminogen activator (TPA) activates tissue plasminogen (Choice A). TPA is a thrombolytic that cleaves plasminogen to form active plasmin. Plasmin degrades the cross-links between fibrin leading to the dissolution of blood clots. The major adverse effect of tPA is bleeding. Warfarin is an anticoagulant that interferes with the carboxylation of coagulation factors (Choice B). This occurs through the inhibition of vitamin K epoxide reductase, which is necessary for recycling vitamin K, an essential cofactor in t

10 ---------- Exam Section 1: Item 11 of 50 National, Board of Medical Examiners Comprehensive Basic Science Self-Assessment 11. An 8-year-old boy is brought to the physician by his parents because of disruptive behavior. They say, "He is easily distracted, constantly interrupts us while we are talking, and seems to be always moving. His teacher says he is always talking with his neighbors, has trouble completing tasks, and refuses to wait his turn when playing games." Physical examination shows no abnormalities. If drug therapy is indicated, administration of a drug with which of the following mechanisms of action is most appropriate? A) Antagonism at B-adrenergic receptors B) Blockade of voltage-gated Na+ channels C) Enhanced action of y-aminobutyric acid (GABA) at GABAA receptors D) Increased release of dopamine and norepinephrine E) Selective inhibition of uptake of serotonin at nerve terminals

D. The first-line treatment for attention-deficit/hyperactivity disorder (ADHD) is stimulant medication such as amphetamine salts, which increase presynaptic release of dopamine and norepinephrine. ADHD presents with chronic symptoms of hyperactivity/impulsivity and/or inattention that occur in more than one setting and impair academic, social, or emotional function. Children with predominant hyperactivity symptoms are unable to sit still and may have difficulty taking turns (postulated to result from a dysregulated reward pathway), whereas children with predominant inattentive symptoms tend to daydream, process information slowly, and demonstrate difficulty in completing tasks (likely related to norepinephrine dysregulation). This patient is experiencing symptoms of both inattention and hyperactivity. Combination treatment with medications and psychotherapy may be the most effective option, though medication monotherapy is also appropriate for children older than 6 years. Stimulants, which include amphetamine salts and methylphenidate, are first-line agents. Both types of stimulants increase synaptic dopamine and norepinephrine, which improve reward processing and attention, respectively. Incorrect Answers: A, B, C, and E. Antagonism at ßB-adrenergic receptors (Choice A) is the mechanism of B-adrenergic blocker medications. Some B-adrenergic blocker medications, such as propranolol, are centrally acting and can be used for performance anxiety or traumatic brain injury-related agitation. These medications are unhelpful in ADHD. Blockade of voltage-gated Na" channels (Choice B) is the mechanism utilized by several mood stabilizers and antiepileptics including carbamazepine and lamotrigine. These medications are used for bipolar disorder, not ADHD. Enhanced action of y-aminobutyric acid (GABA) at GABAA receptors (Choice

104 ---------- Exam Section 3: Item 4 of 50 National, Board pf Medical Examiners ancive Basic Science ment * 4. A 16-year-old boy has a 6-month history of type 1 diabetes mellitus and requires daily insulin injections. His blood glucose control has been poor recently, which the physician suspects may be related to the patient's reluctance to self-administer insulin. Which of the following questions is most appropriate to broach this issue with the patient? A) "Giving yourself insulin injections can be difficult. Whať's it like for you?" O B) "I get the feeling you have not been taking your insulin regularly. Would you like me to arrange some training for you by our nurses?" O C) "It's been my experience that most poorly controlled diabetics have trouble giving themselves insulin. Do you have this problem?" O D) "You are clearly having difficulty with insulin injections. Would you like to arrange for a family member

A. A physician who suspects that a patient is not adhering to treatment should take a nonjudgmental and curious stance and ask open-ended questions to elicit the patient's reasons for not adhering. This physician is nonjudgmental in acknowledging that taking insulin can be hard and assuming that the patient is experiencing an external barrier to adherence rather than inferring or implying that the patient is at fault. Using nonjudgmental, open-ended questioning can improve therapeutic alliance, help the physician and patient problem-solve around the nonadherence, and increase the patient's motivation to adhere. Motivational interviewing similarly uses open-ended questions to encourage patients to explore their reasons to change or maintain behavior and increases a patient's readiness to make or maintain a behavioral change such as adhering to insulin. Incorrect Answers: B, C, D, and E. Immediately proposing solutions to the problem such as arranging training (Choice B) or help from family members (Choice D) presumes knowledge of the underlying barrier to adherence. Alternatively, open-ended questions serve to identify the specific barriers. Choices B and D also assume that the patient has not been self-administering insulin and subtly infer that the patient is at fault, which may injure therapeutic alliance. Calling this patient a "poorly controlled diabetic" (Choice C) may be interpreted by the patient as judgmental. In this statement, the physician is placing the patient as a whole person into a category or stereotype rather than maintaining a narrow focus on the specific behavior of taking insulin. This type of statement may damage the therapeutic alliance and promote defensiveness rather than an open discussion about the factors contributing to this patient's inability to adhere. Asking how frequently the patient h

111 ---------- Exam Section 3: Item 11 of 50 National, Board pf Medical Examiners ancive Rasic Saance ment 11. A 27-year-old man has hepatomegaly and mild jaundice. Serum alkaline phosphatase and y-glutamyltransferase activities are increased. Which of the following properties of these enzymes allows them to be used to detect defects in bile flow? O A) Present in bile ductules O B) Present on the basal surface of hepatocytes O G)Released from the gallbladder O D) Required for bile salt synthesis O E}Required for-bilirubin-conjugation O F) Required for heme catabolism

A. Alkaline phosphatase and y-glutamyltransferase are enzymes present in bile ductules. Serum levels are increased in disorders of the biliary tract due to defects in bile flow. An increase in serum alkaline phosphatase, total bilirubin (especially with a conjugated or direct bilirubin predominance), and y-glutamyltransferase is termed a cholestatic liver injury pattern. Alkaline phosphatases are isoenzymes that catalyze the hydrolysis of organic phosphate esters. They are found on the outer membrane of numerous cell types. The majority of serum alkaline phosphatase is produced by the liver and bone. The small intestine may produce a small detectable amount. In the liver, alkaline phosphatase is found on the canalicular membrane of hepatocytes. Increased serum alkaline phosphatase is associated with cholestasis, infiltrative disease, and bone disease. Increased y-glutamyltransferase levels are associated with disorders of the liver, biliary tract, and pancreas, as well as alcohol ingestion. Incorrect Answers: B, C, D, E, and F. Present on the basal surface of hepatocytes (Choice B) is incorrect as alkaline phosphatase and y-glutamyltransferase are located on the canalicular membrane (apical surface) of hepatocytes. The basal surface interfaces with the sinusoids. Bile is the principle substance released from the gallbladder (Choice C). Bile aids in digestion, neutralizes gastric acid, promotes fat absorption, and is the primary means of excreting cholesterol and bilirubin. Alkaline phosphatase and y-glutamyltransferase are membrane-bound enzymes, which are not excreted into bile. Required for bile salt synthesis (Choice D) describes the enzyme cholesterol 7a-hydroxylase, which catalyzes the initial step in synthesizing bile salts from cholesterol. Deficiencies in this enzyme increase the risk of cholesterol gallstone f

177 ---------- Exam Section 4: Item 27 of 50 Natjonal, Board of Medical Ęxaminersent 27. A 22-year-old boy is admitted to the hospital because of generalized weakness, repeated episodes of vomiting, and coma after several days of reduced food intake because of a minor febrile illness. He is treated successfully with intravenous glucose. After 13 hours of fasting when the child is well, his serum glucose, B-hydroxybutyrate, and free carnitine concentrations decrease markedly, and serum concentrations of long-chain acylcarnitines increase. Triglycerides containing only medium-chain fatty acids are fed, and the serum B-hydroxybutyrate concentration increases to the reference range. Which of the following disorders of fatty acid metabolism is the most likely diagnosis? A) Carnitine-acylcarnitine translocase deficiency O B) Dietary carnitine deficiency O C) Fatty acyl COA synthetase deficiency O D) Medium-chain acyl CoA

A. Carnitine-acylcarnitine translocase functions in the transport of carnitine and carnitine-fatty acid complexes from the cytoplasm into the mitochondrial matrix as part of the carnitine shuttle system of the mitochondrial membrane. Short and medium-chain fatty acids can enter the mitochondria directly, but longe long-chain fatty acids, particularly in muscle and liver. It is a rare autosomal recessive disorder and onset of symptoms usually occurs in infancy or early childhood. Patients may present with encephalopathy, vomiting, hypoketotic hypoglycemia, hyperammonemia, muscle weakness, skeletal myopathy, cardiomyopathy, and ventricular dysrhythmias. Symptoms can be triggered by illness or fasting, and treatment involves the administration of a low-fat, high-carbohydrate formula and carnitine supplementation. Medium-chain fatty acids should only be administered as part of the patient's diet, as these do not require the carnitine shuttle. fatty acids require transport via the carnitine shuttle. Carnitine-acylcarnitine translocase deficiency disrupts this transport, leading to impaired and decreased fatty acid metabolism, which results in the accumulation of Incorrect Answers: B, C, D, and E. Dietary carnitine supplements are needed for patients with primary carnitine deficiency (Choice B), a rare disorder of insufficient carnitine that is characterized by poor feeding, irritability, hypoglycemia, and failure to thrive. Fatty acyl CoA synthetase deficiency (Choice C) is characterized by the impaired beta-oxidation of long-chain fatty acids. Fatty acyl CoA synthetase catalyzes the formation of fatty acyl CoA from fatty acids and coenzyme A, which is required prior to the transportation of fatty acids into the mitochondrial matrix by the carnitine shuttle. Deficiency of this enzyme results in the accumulation of long chai

120 ---------- Exam Section 3: Item 20 of 50 Natjonal, Board of Medical Examinersmen' ansive R aance 20. A 45-year-old woman with chronic myelogenous leukemia comes to the physician for a follow-up examination. Physical examination shows pallor and bleeding gums. Laboratory studies show a leukocyte count of 90,000/mm3 (60% blasts) and a platelet count of 27,000/mm3. A drug regimen of mitoxantrone, vinblastine, and thioguanine is initiated, with no improvement. During the last cycle of scheduled treatment, a sample of the blast cells is harvested, and the intracellular concentrations of mitoxantrone and vinblastine in these blast cells are found to be undetectable. An increase in which of the following mechanisms best explains this observation? A) Drug efflux pump P-glycoprotein activity in leukemic cells O B) Drug metabolism by the hepatic CYP450 system O C) Drug metabolism by novel heme enzymes in leukemic blast cel

A. Drug efflux pump P-glycoprotein activity in leukemic cells accounts for the undetectable levels of mitoxantrone and vinblastine. Chronic myelogenous leukemia (CML) is defined by the presence of the Philadelphia chromosome, a translocation between chromosomes 9 and 22, which causes constitutive activation of the ABL1 kinase. Patients demonstrate significant leukocytosis, with levels often exceeding 100,000 cells/mm°. Basophilia and eosinophilia are often present. Treatment is typically with tyrosine kinase inhibitors (TKIS) imatinib or dasatinib, but CML blast crisis may require additional chemotherapeutic agents. Leukemic cell resistance to chemotherapy can occur through a variety of mechanisms, but one common mechanism is the expression of transporter proteins that pump drugs out of the cells. Several examples include the ABC and MDR class of efflux pumps, which utilize ATP to shuttle drugs out of the cell against a concentration gradient. The P-glycoprotein efflux pump belongs to the ABC class of transporters. Both vinblastine and mitoxantrone must be intracellular to exert antineoplastic effects, so efflux of these proteins results in intracellular concentration below the levels required to exert their chemotherapeutic effects. Incorrect Answers: B, C, D, E, and F. Drug metabolism by the hepatic CYP450 system (Choice B) does not account for the lack of vinblastine or mitoxantrone within leukemic cells. Vinblastine is metabolized by CYP2D6 and CYP3A4 while mitoxantrone is metabolized by BCRP/ABCG2. Drug metabolism by novel heme enzymes in leukemic blast cells (Choice C) does not explain reduced intracellular drug levels. Heme oxygenase-1 is an inducible protein in CML cells that degrades heme and confers resistance to TKIS. This is a potential therapeutic target in CML. Production of sphingomyelin by the leukemic

101 ---------- Exam Section 3: Item 1 of 50 Natjonal, Board of Medical Examinersent * 1. The higher incidence of birth defects in children born to women more than 35 years old may be the result of environmental influences on chromosomes in developmentally arrested oocytes at which of the following stages of cell division? O A) Prophase, meiosis I O B) Prophase, meiosis II C) Metaphase, meiosis I O D) Metaphase, meiosis II E) Telophase, meiosis I

A. In the normal development of the oocyte, women are born with a complement of follicles that develop in utero and arrest in prophase of meiosis I. Upon reaching menarche, some of the follicles mature during the menstrual cycle prior to being released during ovulation. In response to the hormonal stimulation from pituitary follicle-stimulating hormone (FSH), the oocytes that may be ovulated advance from prophase of meiosis I to metaphase of meiosis II. If fertilization occurs, the oocyte advances through the remainder of metaphase Il to completion. In prophase of meiosis I, the chromosomes remain dormant in a paired tetrad (four total chromosomes: duplicated maternal and paternal chromosomes). As meiosis I advances through metaphase, anaphase, and telophase, the recombined, crossed-over tetrads normally divide into two separate cells, where they subsequently arrest in metaphase II. Normal progression through meiosis Il then leads to a single haploid chromosome in each of the four daughter cells. Nondisjunction during prophase I results in the daughter cells receiving an uneven complement of chromosomes. A nondisjunction event in meiosis I followed by a normal meiosis Il results in four gametes, two of which lack a maternal or paternal chromosome, and two of which contain both a maternal and paternal chromosome. In the instance of normal separation in meiosis I followed by a nondisjunction event of meiosis |l there are two chromosomes (both from the same parent) within one gamete, with the other gamete lacking the chromosome. In the case of a 35-year-old woman, oocytes maturing and ovulated at that time would have remained arrested in prophase I for the lifetime of the patient until that point. Incorrect Answers: B, C, D, and E. Prophase, meiosis II (Choice B); metaphase, meiosis I (Choice C); and telophase, meiosis I

139 ---------- Exam Section 3: Item 39 of 50 Natjonal, Board of Medical Examinersment ancive Rasic Saance 39. A study is conducted to evaluate the effectiveness of cough medication in children. A total of 120 patients under the age of 12 years who have been diagnosed with a viral upper respiratory tract infection are randomly selected to receive dextromethorphan or no treatment. The participants were recruited from several emergency departments in an urban area. Parents complete a rating scale for their children's cough at baseline and 1 day after receiving dextromethorphan or nothing. The results show a statistically significant improvement in cough scores for patients receiving dextromethorphan (p=0.04) compared with children receiving no treatment. Which of the following is the most significant limitation to the internal validity of this study? A) Lack of blinding between groups O B) Limited number of participants

A. Internal validity describes the degree to which a study establishes or supports a faithful conclusion about cause and effect-in other words, was the design of the study sufficient enough to support the conclusion that was drawn. Sufficient internal validity should limit the degree to which alternative explanations of cause and effect can be reasonably drawn. Internal support the claim. Common threats and limitations to internal validity include many types of selection bias and confounding, alterations in the characteristics of the study participants over the course of the study, experimental mortality, observer biases, Hawthorne effect, and Berkson bias. In this case, the lack of blinding between groups could theoretically cause parents to draw conclusions that may be based on expected results as opposed to true outcomes (eg, "My child received the actual medicine and is doing better."). idity thus refers to whether the treatment in question (or variable being tested) makes difference, and whether the experiment was designed in way to actually Incorrect Answers: B, C, D, and E. Limited number of participants (Choice B) often limits statistical power, the ability of the study to detect a difference if one exists. While the actual number required is calculated by effect size and beta level, there is no indication that this study lacks power. Marginal statistical significance (Choice C) implies that since the p value in the study was close to the designated alpha level, that the study may be less statistically significant than one with a greater difference between the p value and alpha level. The study demonstrated ap value less than the alpha level; it is therefore considered statistically significant. Such a statement would be equally true for a p value of 0.04, 0.03, 0.02, 0.01, or less. Study location limited to on

184 ---------- Exam Section 4: Item 34 of 50 Natjonal, Board pf Medical Examinersment 34. A 1-year-old boy is brought to the emergency department because of a 6-day history of temperatures to 39.4°C (103°F) and a 2-day history of a severe diaper rash and swelling of his hands and feet. His temperature is now 39.4°C (103°F). A photograph of the genital area is shown. The face and lips appear red and the conjunctivae appear injected. There is bilateral cervical lymphadenopathy. The lungs are clear to auscultation. Cardiac examination shows an S3 gallop with no murmur. There is edema and erythema of the hands and feet. Which of the following pathologic findings is most likely in this patient? O A) Acute arteritis with aneurysms in coronary arteries B) Epidermal hyperplasia with epidermal microabscesses and parakeratosis O C) Granulomas with caseous necrosis in cervical lymph nodes O D) Granulomatous arteritis in cer

A. Kawasaki disease is a multisystemic vasculitic disease resulting in acute arteritis with a risk for the development of aneurysms in coronary arteries. In children, the disease is characterized by fever for at least 5 days, bilateral nonexudative conjunctivitis, a polymorphous rash, lymphadenopathy, erythema and edema of the hands and feet, and erythema affecting the mucous membranes with dry, cracked lips. To diagnose Kawasaki disease, fever for 5 days or more plus four of the remaining criteria are required. Laboratory evaluation may show thrombocytosis and increased erythrocyte sedimentation rate. Complications of Kawasaki disease include coronary artery aneurysms, and if untreated, the potential for aneurysmal expansion, rupture, or myocardial infarction. Therefore, children suspected of having Kawasaki disease require immediate evaluation with ECG and echocardiography, immediate treatment, and monitoring for progression or complications. Treatment of Kawasaki disease involves high-dose aspirin and intravenous immunoglobulin, which is the only widely accepted indication for aspirin use in febrile children due to the risk of Reye syndrome. Kawasaki disease is also known as mucocutaneous lymph node syndrome, is one of the most common childhood vasculitides, and is a common cause of pediatric heart disease. Incorrect Answers: B, C, D, and E. Psoriasis would demonstrate epidermal hyperplasia with epidermal microabscesses and parakeratosis (Choice B) on histopathology. While psoriasis can present with an inverse subtype in which shiny, pink patches form in the inguinal folds, it would not explain the patient's fever, mucosal findings, or cervical lymphadenopathy. Psoriasis is associated with cardiovascular disease in adults, but it is not a cause of pediatric heart disease. Granulomas with caseous necrosis in cervical

134 ---------- Exam Section 3: Item 34 of 50 Natjonal, Board of Medical Examinersment V 34. A 6-year-old boy is brought to the emergency department because of coughing, wheezing, and rapid breathing for 6 hours. He was diagnosed with an upper respiratory tract infection 2 days ago. His temperature is 37.2°C (99°F), pulse is 120/min, respirations are 44/min, and blood pressure is 90/60 mm Hg. Inspiratory and expiratory wheezes are heard throughout the lung fields. There is decreased tactile fremitus. Which of the following is the most likely diagnosis? A) Asthma O B) Atelectasis O C) Bronchitis O Đ}Left-sided heart failure O E) Pneumococcal pneumonia

A. The patient is likely experiencing an asthma exacerbation. Asthma is characterized by reversible obstruction of the bronchi secondary to hyperreactivity and airway inflammation. Patients present with episodes of wheezing, dry cough, and dyspnea. Physical examination often reveals tachycardia, tachypnea, diffuse wheezes (rhonchi), and prolonged expiration relative to inspiration. Decreased tactile fremitus may be noted due to air trapping which decreases lung density (leading to reduced transmission of vibrations through the lung parenchyma to the body waill). Chest radiographs are typically normal. Pulmonary function tests may be normal between symptomatic exacerbations, but during such episodes demonstrate an obstructive pattern with increased residual volume and total lung capacity. Viral upper respiratory tract infection is a common trigger of acute asthma exacerbation in patients under 12 years of age. Acute management includes supplemental oxygen, inhaled short-acting B-adrenergic agonists, and systemic corticosteroids. Incorrect Answers: B, C, D, and E. Atelectasis (Choice B) refers to the partial or complete collapse or incomplete expansion of pulmonary parenchyma. There are multiple types of atelectasis, including obstructive, passive, compressive, cicatricial, and adhesive. It is most common in patients who are immobile and/or in the postsurgical state, who are unable to produce adequate tidal volume to expand the lungs. Signs include decreased breath sounds and dullness to percussion in the affected region. Acute bronchitis (Choice C) is a respiratory tract infection involving inflammation of the bronchi. It is most commonly caused by viruses such as influenza A and B, parainfluenza, respiratory syncytial virus, coronavirus, adenovirus, rhinovirus, and human metapneumovirus. It typically presents with coug

144 ---------- Exam Section 3: Item 44 of 50 Natjonal, Board of Medical Examinersment 44. A 42-year-old man comes to the physician because of a 1-week history of a diffuse, painless rash on his back. He first noticed the rash after he spent a week sunbathing at the beach. He says the rash does not itch. A photograph of the rash is shown. Which of the following is the most likely causal organism? A) Malassezia furfur O B} StaphylocOGGUs aureus O G} StreptocoEGUS pyogenes (groupA) O Đ} Treponema pallidum O E) Trichophyton rubrum

A. Tinea versicolor is a superficial infection of the epidermis caused by Malassezia furfur. Clinically, it is characterized by hypopigmented and hyperpigmented macules and patches with a fine scale. The infection has a predilection for sites with increased sebum production such as the chest and back. Patients commonly notice the rash after spending time outdoors, as Malassezia furfur inhibits tanning of the affected areas, making them more noticeable when compared to the unaffected neighboring skin. A potassium hydroxide preparation of a skin scraping can be helpful and will show fungal spores and short hyphae along the edges of epidermal cells. Tinea versicolor may be treated using topical antifungal preparations including topical ketoconazole or selenium sulfide preparations. Incorrect Answers: B, C, D, and E. Staphylococcus aureus (Choice B) is a common cause of cutaneous infections including impetigo, cellulitis, folliculitis, and furunculosis. It can also produce an exotoxin leading to toxic shock syndrome. While toxic shock syndrome can cause an erythematous rash, the patient also classically exhibits fever, vital sign instability, and signs of end-organ dysfunction related to the underlying systemic shock. Streptococcus pyogenes (group A) (Choice C) is a Gram-positive bacterium that can result in cellulitis and pharyngitis. Additional conditions related to Streptococcus pyogenes include scarlet fever, necrotizing fasciitis, glomerulonephritis, and rheumatic fever. Treponema pallidum (Choice D) is the causative spirochete of syphilis, which presents in multiple stages with varying symptoms, including primary with a painless chancre, secondary with fever, lymphadenopathy, and condylomata lata, and tertiary with tabes dorsalis, aortitis, and gummas. Trichophyton rubrum (Choice E) is a dermatophyte (skin fungus) th

113 ---------- Exam Section 3: Item 13 of 50 Natjonal, Board of Medical Examinersment 13. A 47-year-old woman comes to the physician because she wants to "do something about my wrinkles." Physical examination shows fine wrinkles in photodamaged skin on the face. Treatment with tretinoin is started. Collagen synthesis will most likely be increased in this patient by which of the following mechanisms? A) Activation of nuclear gene transcription O B) Decreased CAMP production O C) Displaced vitamin A from cellular stores O D) Increased sebum production E) Protection of keratinocytes from UVB irradiation

A. Tretinoin, a Vitamin A analog, exerts its effects by binding to retinoic acid receptors, which are transcription factors in the nucleus of the fibroblast. All-trans-retinoic acid (ATRA) is another name for tretinoin. This medication is also used systemically for the treatment of acute promyelocytic leukemia. In the case of tretinoin, binding to its receptor causes activation of the nuclear gene transcription of collagen mRNA. Once the mRNA exits the nucleus, it is transported to the rough endoplasmic reticulum where it undergoes translation into preprocollagen and post-translational processing. The proline and lysine residues of preprocollagen are hydroxylated, which requires vitamin C as a cofactor, and then glycosylated. They form a triple helix called procollagen which is exocytosed into the intracellular space. There, each end of procollagen is trimmed creating tropocollagen. Many strands of tropocollagen then crosslink, which requires copper as a cofactor, and form the final collagen fiber. An increase in collagen leads to a plumper dermis, which is visibly perceived as a more youthful and less wrinkled appearance. Incorrect Answers: B, C, D, and E. CAMP is a signaling molecule involved in many intracellular pathways. However, decreased CAMP production (Choice B) does not play a role in the mechanism by which tretinoin increases collagen production. While tretinoin and other retinoids are vitamin A derivatives, they do not displace vitamin A from cellular stores (Choice C). Increased sebum production (Choice D) occurs in response to androgen stimulation of sebaceous glands. Sebum, an oily substance, does not have any effect on collagen production. Sebum decreases the dryness of the skin but does not affect the presence of wrinkles or photodamage. Protection of keratinocytes from UVB irradiation (Choice E) will

170 ---------- aive f Medical Examinersrment Exam Section 4: Item 20 of 50 Natjonal, Board * 20. A 42-year-old woman comes to the physician because of a 3-week history of numbness of the fingers on the left hand. Neurologic examination shows loss of touch graphesthesia and loss of two-point discrimination in the left hand. This patient most likely has a lesion in which of the following locations of the hemispheres shown? E F O A) O B) C) O E) O F) O H) O 1) J)

B. Choice B reflects the right anterolateral parietal lobe, representing the primary sensory cortex in the postcentral gyrus. This brain region mediates sensation in the left (contralateral) distal upper extremity. Lesions of this area result in deficits in all sensory domains including the loss of sensation of fine touch, pressure, proprioception, vibration, and pain. These deficits can also manifest on physical examination as loss of two-point discrimination, agraphesthesia, and astereognosia. This brain area is supplied by the middle cerebral artery (MCA). Cerebral infarctions (strokes) occur due to ischemic or hemorrhagic loss of blood supply to the brain. Approximately 80% to 85% of strokes are ischemic, commonly arising from thromboembolic disease (eg, middle cerebral artery occlusion from a thrombus), whereas 15% to 20% of strokes are hemorrhagic due to blood vessel rupture (eg, hypertension-related intraparenchymal hemorrhage from a perforating artery). Risk factors for stroke include smoking, hypertension, diabetes, carotid or intracranial atherosclerotic disease, history of hypercoagulability, atrial fibrillation, and advanced age. Classically, strokes manifest as a neurologic deficit related to the affected part of the brain. Incorrect Answers: A, C, D, E, F, G, H, I, and J. Choices A and J identify the posterior, superior temporal gyrus, an area that in the dominant hemisphere makes up one portion of the Wernicke area, a brain region involved in the understanding of language. In a stroke, lesions involving this area can result in Wernicke aphasia. In the non-dominant hemisphere, this area is involved in discriminating the pitch of sound. This region is supplied by the MCA. Choices C and H reflect the superolateral aspect of the posterior frontal lobes (precentral gyrus). This region contains the primary mot

199 ---------- Exam Section 4: Item 49 of 50 Natjonal, Board of Medical Examinersment aive B A tance * 49. A 62-year-old man with hypertension comes to the physician because of a 10-day history of fatigue and shortness of breath. His temperature is 37.4°C (99.4°F), pulse is 102/min, respirations are 18/min, and blood pressure is 110/70 mm Hg. Pulmonary examination shows diffusely decreased breath sounds and a prolonged expiratory phase. Cardiac examination shows distant heart sounds. There is 1+ pretibial edema of the lower extremities. Pulmonary function testing shows: 65% of predicted 90% of predicted FEV1/FVC60% of predicted FEV1 FVC Which of the following is the most likely diagnosis? A) Cardiac tamponade O B) Chronic obstructive pulmonary disease O G} Gongestive heart failure O Đ} Mitral stenosis O E) Pleural effusion O F}Sarcoidesis

B. Chronic obstructive pulmonary disease (COPD) is an obstructive lung disease characterized by decreased lung function from a combination of chronic bronchitis and/or emphysema, resulting in airflow obstruction on expiration. Physical examination reveals decreased breath sounds due to reduced airflow and a prolonged expiratory phase as the pathologically narrowed airways are further compressed by positive intrathoracic pressure during expiration. Heart sounds may be distant secondary to emphysematous lung changes (sound is transmitted poorly through air relative to normal lung tissue). Pulmonary abnormalities can be evaluated by pulmonary function testing, which includes spirometry, full-body plethysmography, single-breath diffusing capacity, arterial blood gas analysis, and pulse oximetry. Spirometry is the most useful in diagnosing obstructive lung disease. The patient breathes into an external device (spirometer) which records the volume and rate of airflow during inspiration and expiration to generate a flow-volume loop. Obstruction is defined by a decrease in the ratio of the volume of air forcefully expelled in the first second of expiration to the total volume of air expelled during the expiratory phase (FEV1/FVC). The severity of obstruction is graded by the amount of FEV1 reduction compared to predicted values. In this patient, the decreased FEV1/FVC (less than 80% of predicted defines obstruction) and the decreased FEV1 suggest moderate obstructive lung disease. Incorrect Answers: A, C, D, E, and F. Cardiac tamponade (Choice A) presents with tachycardia, hypotension, jugular venous distention, pulsus paradoxus, and muffled heart sounds, and results from rapid and/or excessive accumulation of fluid within the pericardial space impairing right ventricular diastolic filling. Congestive heart failure (Choice C)

118 ---------- Exam Section 3: Item 18 of 50 National Board of Medical Examiners nent * 18. A 43-year-old woman comes to the physician because of a 6-month history of altered consciousness. During the interview, she stops talking in mid-sentence, turns her head to the right, and extends and stiffens her right upper extremity. She has a blank look and does not respond to any questions. She then has some repetitive lip smacking and picking movements of the hands. The episode lasts approximately 30 seconds. She slowly returns to her normal state during the next 4 to 5 minutes. The most likely diagnosis is which of the following types of seizures? A) Absence O B) Complex partial O C) Generalized tonic-clonic O D) Simple partial motor O E) Simple partial sensory

B. Complex partial seizures are also called focal onset impaired awareness seizures. As this name suggests, complex partial seizures affect a specific (focal) region of the brain and therefore produce motor or sensory symptoms that are localized to one or a few areas of the body. Oral and manual automatisms, patient was unresponsive during the seizure (impaired awareness) and confused for a few minutes postictally, which is typical for complex partial seizures (as opposed to intact consciousness during simple seizures and more prolonged postictal confusion after generalized seizures). Complex partial seizures demonstrate localized rhythmic activity with generalized slowing on electroencephalogram (EEG). Treatment requires anti-epileptic medication. in this pati are suggestive of a temporal lobe origin. Upper limb dystonia as in this patient is also com mon in temporal lobe seizures. Patients may experience visual, auditory, or sensory auras preceding the seizure. This Incorrect Answers: A, C, D, and E. Absence seizures (Choice A) are characterized by a transient blank stare and 3-Hz spike-and-wave complexes on EEG without postictal confusion. This patient had postictal confusion as well as motor symptoms, making absence seizures an unlikely explanation. Generalized tonic-clonic seizures (Choice C) are characterized by a sudden loss of consciousness followed by a stiffening all of the muscles of the arms, legs, chest, and back (tonic phase), progressing to jerking or twitching of all of the muscles (clonic phase). After the seizure, postictal confusion is common. This patient had focal rather than generalized motor symptoms. Simple partial motor seizures (Choice D) can present with similar motor symptoms localized to a particular area of the body as in this patient. However, impaired consciousness and postictal confusio

175 ---------- Exam Section 4: Item 25 of 50 National Board of Medical Examiners ansive Basic Saance smont V 25. A 59-year-old man comes to the physician because of a 3-month history of progressive difficulty swallowing; he has had a 6.8-kg (15-lb) weight loss during this period. He has smoked 1 pack of cigarettes daily for 40 years and consumes four to six beers daily. He is cachectic. Bilateral wheezes are heard on auscultation of the chest. A chest x-ray shows enlargement of the paratracheal and hilar lymph nodes. Subsequent endoscopy shows a 6-cm mass in the mid esophagus. The mass is centrally ulcerated with an elevation of the surrounding mucosal rim. Examination of a biopsy specimen of the mass is most likely to show neoplastic cells with which of the following? O A) Acinar formation O B) Foci of keratinization O C) Intracytoplasmic pigment O D) Neuroendocrine granules O E) Smooth muscle differentiation

B. Esophageal cancer typically presents with progressive dysphagia due to the mass effect of the tumor. Initially, the mass is small enough to only obstruct the passage of large, solid foods. As the mass enlarges, passage of all solids becomes affected, along with some or all liquids. Nonspecific signs of malignancy, such as weight loss, are also seen. Direct extension of the malignancy lymph nodes may cause wheezing through airway narrowing, and involvement of the recurrent laryngeal nerve may cause hoarseness. The two most common subtypes of esophageal cancer are squamous cell carcinoma and adenocarcinoma. Řisk factors for squamous cell carcinoma including smoking, alcohol use, hot liquids, and the presence of strictures. The upper two thirds of the esophagus are more likely to be affected. In contrast, chronic gastroesophageal reflux disease, Barrett esophagus, and obesity increase the risk of adenocarcinoma, which is more likely to affect the distal third of the esophagus. In this patient, squamous cell carcinoma is more likely given his significant alcohol and smoking history and location of the mass in the mid esophagus. Squamous cell carcinoma, regardless of the organ involved, is derived from cells which produce keratin. When squamous cells become aberrant, they continue producing keratin but in small foci called keratin pearls. In contrast, adenocarcinoma is derived from glandular epithelium and thus forms gland-like structures, even when malignant. reactive lymphadenopathy surrounding the ne boring structures may also cause symptoms. For example, en rgeme of paratracheal and hilar Incorrect Answers: A, C, D, and E. Acinar formation (Choice A) is a feature of adenocarcinoma. An acinus is a small cavity within a gland. Because adenocarcinomas are derived from glandular epithelium, in malignant form the cells m

122 ---------- Exam Section 3: Item 22 of 50 Natjonal, Board of Medical Examinersment 22. A 22-year-old woman of Irish descent is brought to the emergency department because of a 1-hour history of severe pain and coldness of her left leg. She has an IQ of 80, and she dropped out of high school in the tenth grade. Her parents and two older siblings have normal intelligence and have no history of these symptoms. Ophthalmologic examination shows a partially dislocated lens in the right eye. Physical examination shows mottling and loss of pulses in the left lower extremity and foot. Arteriography of the left lower extremity shows thrombosis of the femoral artery. This patient most likely has a metabolic disorder involving which of the following amino acids? O A) Glycine B) Homocyst(e)ine O C) Leucine O D) Phenylalanine O E) Tyrosine

B. Homocystinuria is an autosomal recessive disorder that may vary in etiology and pathophysiology but results in excess homocysteine. Homocysteine is normally metabolized by cystathionine synthase to become cystathionine or methionine synthase to become methionine. Cystathionine synthase deficiency, methionine synthase deficiency, or abnormal affinity of cysta urine, intellectual disability, lens subluxation or dislocation (typically downward), osteoporosis, marfanoid habitus, thrombosis, and atherosclerosis that increases the risk of peripheral arterial disease, ischemic stroke, and coronary artery disease. ionine synthase for pyrie kine may all cause excess hom ysteine accumulation. Homocystinuria is characterized by increased levels of homocysteine in the Incorrect Answers: A, C, D, and E. Glycine (Choice A) accumulation can result from the autosomal recessive disorder glycine encephalopathy and is characterized by seizures, intellectual disability, lethargy, and hypotonia. Thrombosis and lens subluxation are not characteristic of glycine encephalopathy. Maple syrup urine disease is caused by the abnormal degradation of isoleucine, leucine, and valine (Choice C) and is characterized by severe neurologic defects, intellectual disability, vomiting, and poor feeding. The urine characteristically has an odor similar to maple syrup. It is not associated with thrombosis or atherosclerosis. Phenylketonuria is an autosomal recessive disorder that occurs secondary to a deficiency in phenylalanine hydroxylase, which results in an excess of phenylalanine (Choice D). It can also present with intellectual disability, but is additionally characterized by seizures, eczema, and a musty body odor. Alkaptonuria is an autosomal recessive congenital deficiency that prevents the degradation of tyrosine (Choice E) to fumarate via homoge

112 ---------- Exam Section 3: Item 12 of 50 Natjonal, Board of Medical Examinersment 12. A 62-year-old man has had petechiae and easy bruising for the past month. He has a 2-year history of alcoholic cirrhosis with portal hypertension. Laboratory studies show: 10 g/dL Hemoglobin Hematocrit 30% 4000/mm3 65% Leukocyte count Neutrophils Lymphocytes Monocytes Eosinophils Basophils Reticulocyte count Mean corpuscular volume Platelet count 30% 4% 1% 0% 8% of red cells 102 um3 68,000/mm3 Prothrombin time 16 sec Partial thromboplastin time (activated) Serum 28 sec Fibrinogen Fibrin split products 200 mg/dL (N=200-400 mg/dL) <10 (N < 10) Which of the following is the most likely cause of the pancytopenia? A) Disseminated intravascular coagulation B) Hypersplenism O C) Iron deficiency O D) Vitamin B1 (thiamine) deficiency O E) Vitamin C deficiency

B. Hypersplenism is defined as the presence of one or more cytopenias in the setting of splenomegaly, which is a common complication of cirrhosis and portal hypertension. Other causes of splenomegaly and potential hypersplenism include lymphoma, splenic sequestration from sickle cell disease, malaria, mononucleosis, and connective tissue disorders. Splenomegaly is associated with increased splenic activity and removal of cells from the blood with the potential development of decreased cell counts in one or more cell lines. Pancytopenia, as seen on this patient's laboratory studies, refers to a decrease in all three cell lines (red blood cells, white blood cells, and platelets). Symptoms of thrombocytopenia include abnormal bleeding or bruising, petechiae, ecchymoses, and mucosal bleeding (eg, uterine bleeding, epistaxis), along with persistent "oozing" at puncture and operative sites. Anemia may present with fatigue, light-headedness, and dyspnea. Leukopenia increases susceptibility to infection. Treatment for hypersplenism requires management of the underlying cause (eg, transjugular intrahepatic portosystemic shunt [TIPS] for portal hypertension), supportive therapy with potential transfusions for the cytopenias, and, if severe or refractory, splenectomy and/or liver transplantation. Incorrect Answers: A, C, D, and E. Disseminated intravascular coagulation (Choice A) is a systemic disorder characterized by the widespread activation of the clotting cascade resulting in microthrombi formation, thrombocytopenia, and bleeding complications due to consumption of platelets and clotting factors. It can be provoked by trauma, sepsis, malignancy, pancreatitis, exposure to toxins, and multiple other etiologies. Laboratory studies reveal thrombocytopenia, prolonged prothrombin time (PT) and activated partial thromboplastin time

130 ---------- Exam Section 3: Item 30 of 50 Natjonal, Board pf Medical Examinersment 30. A healthy 70-year-old woman has participated in a longitudinal study of the effects of aging on performance during pulmonary function tests for the past 50 years. She has undergone a complete set of tests, including arterial blood gas analyses, every 5 years. Which of the following sets of changes best represents this woman now compared with her results at the age of 20 years? Alveolar-arterial Ро2 Residual Arterial Po2 Difference Volume A) ↑ B) ↑ C) No change ↑ ↑ O D) No change O E) O F)

B. Normal lung function declines with age, with an expected increase in residual volume, decreased arterial PO2, and an increased alveolar-arterial PO2 difference (A-a gradient). These changes result from structural, physiological, and immunological alterations that occur with aging. Osteoporosis and kyphosis of the vertebral column alter the mechanics of respiratory function, and calcification of the rib cage results in a stiffened chest wall with decreased compliance. Residual volume (volume of air remaining at the end of forced expiration) increases with age. The expansion of the lungs at the end of expiration is a balance between an outward force from the chest wall and an inward force from the lungs. Decreased chest wall compliance results in a greater outward force, while loss of lung elastic recoil results in a reduced inward force. Arterial PO2 decreases with age due to a decreased responsiveness of the respiratory control center to hypoxemia and hypercarbia, which would normally stimulate an increase in minute ventilation. The A-a gradient has been also shown to increase with age, which may be due to the loss of alveolar surface area for gas exchange due to persistent wear and progressive damage to the alveolar-capillary interface along with reduced regeneration secondary to pneumocyte cell senescence, and chronic damage to the small airways over time. Incorrect Answers: A, C, D, E, and F. Choices A, D, E, and F are incorrect as the A-a gradient increases with age, which is secondary to alveolar surface dysfunction from progressive damage over time, along with small airways degradation, and reduced repair mechanisms. Choices C, D, E, and F are incorrect as residual volume commonly increases with age. The most common cause of a decreased residual volume is a restrictive lung disease, which includes diagnoses su

131 ---------- Exam Section 3: Item 31 of 50 National Board of Medical Examiners nent * 31. A study is conducted to assess the relationship between angiotensin gene polymorphisms (I versus D alleles) and the development of progressive renal disease due to IgA nephropathy. A total of 250 patients are enrolled: 125 patients with progressive renal disease due to IgA nephropathy (Group X) and 125 patients with IgA nephropathy with stable renal disease (Group Y). Results show that 67 patients in Group X have the D allele, whereas 12 patients in Group Y have the D allele. Which of the following tests is most appropriate to determine whether there is a significant difference in the prevalence of the D allele between these two groups? A) Analysis of variance O B) Chi-square test O C) Linear regression O D) Nonparametric regression E) Student's t-test

B. Statistical tests can be employed to assess whether a difference in values exists. Often, these values are means, though they need not be. Many tests have been derived to assess differences in numerical, quantitative, categorical, and qualitative datasets. Choosing the appropriate test is important in order to appropriately draw conclusions from data. In this case, comparing allele square test. Chi-square tests assess for statistically significant differences between two or more categorical values (eg, proportions, percentages). It is commonly used for larger samples and is also able to compare multiple values (eg, three different experimental groups). In this study, assessing the difference between the percentage of patients having the allele between the two arms is a categorical comparison. equency across two groups is a comparison of categorical values (the presence or absence of the allele in quest which is best assessed by a chi- Incorrect Answers: A, C, D, and E. Analysis of variance (Choice A) is a statistical test that compares means between three or more groups. In this case, the comparison is of a percentage, not a mean, and only two groups are analyzed. Linear regression (Choice C) and nonparametric regression (Choice D) model the relationship between dependent and independent variables in a linear fashion or without beginning with a predetermined form but instead deriving the trend from the data, respectively. It is not used to compare means. It is more frequently used to predict the dependent variable from an independent variable assumed to behave in a similar way to the data used to construct the regression. Student's t-test (Choice E) also simply referred to as a t-test, calculates differences between two means. For example, comparing the resting heart rates between athletes and non-athletes to assess

106 ---------- Exam Section 3: Item 6 of 50 National, Board pf Medical Examiners mont 6. A 65-year-old man with diabetes mellitus develops a neuropathy of the cranial nerve indicated by the arrow in the normal brain stem. Which of the following is the most likely associated functional deficit? O A) Complete ptosis of the ipsilateral eye O B) Inability to abduct the ipsilateral eye O C) Ipsilateral sensorineural deafness D) Lack of tear formation in the ipsilateral eye O E) Paralysis of ipsilateral jaw muscles

B. The abducens nerve (cranial nerve VI) arises from the mid-pons and exits the brainstem ventrally at the pontomedullary junction. It ascends steeply through the subarachnoid space across the clivus before proceeding through Dorello's canal to the cavernous sinus. It exits the cavernous sinus through the superior orbital fissure before reaching its target, the lateral rectus muscle, in the lateral orbit. The most common etiologies of acute abducens nerve palsy in adults include microvascular ischemia (secondary to diabetes, hypertension, and atherosclerotic cardiovascular disease), idiopathic etiologies, and trauma. The abducens nerve is fragile and vulnerable to injury as it crosses the clivus by any mechanism causing downward traction, such as brain herniation. Abducens nerve palsy presents with binocular horizontal diplopia and an inability to abduct the ipsilateral eye. Treatment includes systemic workup for vasculopathic risk factors and patching to reduce diplopia. Abducens nerve palsy secondary to microvascular disease may resolve spontaneously over several months. Incorrect Answers: A, C, D, and E. Complete ptosis of the ipsilateral eye (Choice A) can occur as a component of oculomotor nerve (cranial nerve III) palsy, or in association with Horner syndrome, myasthenia gravis, trauma, or age-related dehiscence of the aponeurosis of the levator palpebrae superioris muscle. The abducens nerve is not responsible for innervation of the muscles of the upper eyelid. Ipsilateral sensorineural deafness (Choice C) suggests pathology of the vestibulocochlear nerve (cranial nerve VIII), which may be caused by microvascular ischemia, multiple sclerosis, compression secondary to acoustic neuroma, trauma, or infection. Lack of tear formation in the ipsilateral eye (Choice D) may be a feature of facial nerve palsy, otherwise

154 ---------- Exam Section 4: Item 4 of 50 Natjonal, Board of Medical Examinersent 4. A 55-year-old man has a cough, hemoptysis, and dyspnea. He has smoked 2 packs of cigarettes daily for the past 30 years. Examination shows bilateral conjunctival edema, distention of sublingual veins, and edema of the upper extremities. Which of the following additional findings is most likely to be present on examination? O A) Bisferiens carotid pulsation B) Increased jugular venous pressure O C) Pansystolic apical murmur O D) S3 O E) S4

B. The patient is presenting with physical examination findings suggestive of superior vena cava (SVC) syndrome, which is caused by obstruction of the SVC and subsequent venous congestion of the head, neck, and upper extremities. Obstruction of the SVC is often due to extrinsic compression by a malignant mediastinal mass, especially lung cancer, non-Hodgkin lymphoma, and metastatic disease. Nonmalignant etiologies include venous thrombus formation, which may be more prevalent in patients with indwelling intravascular devices (eg, central venous catheters, PICC lines, port systems, pacemakers, defibrillator leads). Physical examination findings include swelling of the face and neck, increased jugular venous pressure, upper extremity swelling, distended chest vein collaterals, and conjunctival edema. Treatment depends on the underlying etiology. Options for bypassing the obstruction include endovascular recanalization and open surgical repair. Incorrect Answers: A, C, D, and E. Bisferiens carotid pulsation (Choice A) is associated with aortic regurgitation, combined aortic regurgitation and stenosis, and less commonly hypertrophic cardiomyopathy. The arterial waveform has a double systolic peak, with an initial sharp, shortened peak followed by a lower amplitude, broader peak. The examination findings in this case are more suggestive of SVC syndrome. A pansystolic apical murmur (Choice C) is most characteristic of mitral regurgitation, which is best heard in the left fourth or fifth intercostal space along the midclavicular line and radiates to the left axilla. It is commonly associated with mitral valve prolapse and prior myocardial infarction. SVC syndrome does not cause mitral regurgitation. S3 gallop (Choice D) is a heart sound that occurs in early diastole and is associated with increased ventricular filling pressur

182 ---------- Exam Section 4: Item 32 of 50 Natjonal, Board of Medical Examinersment ancive Rasic Salance * 32. A 55-year-old man comes to the physician because of a 2-month history of hoarseness and difficulty swallowing. He underwent an operation to resect a lobe of his lung because of lung cancer 1 year ago. Physical examination shows ptosis of the left eyelid and miosis of the left pupil. There is diminished flushing and sweating on the left half of the face. This patient most likely has disordered function of neurons at which of the following labeled locations in the photograph of a cross section of the spinal cord? B Left Right A) B) C) D) E) O F)

B. This patient likely has a lesion of the ciliospinal center of Budge (labeled "B"), which includes spinal cord levels C8 to T2 of the intermediolateral cell column (also known as the intermediate horn). This patient demonstrates Horner syndrome, which classically presents with the triad of ipsilateral miosis, ptosis, and anhidrosis. Horner Syndrome can arise from lesions along a sympathetic pathway. This pathway starts in the hypothalamus and descends ipsilaterally to the first synapse in the ciliospinal center of Budge. Second-order neurons exit the spinal cord and enter the cervical sympathetic chain (in close proximity to the pulmonary apex and subclavian artery), where they then ascend to the superior cervical ganglion. Some third-order neurons branch from this ganglion to innervate the sweat glands and vasculature of the face, while others ascend along the internal carotid artery, eventually joining with the oculomotor nerve to enter the orbit and innervate the pupil and eyelid smooth muscle. Miosis and ptosis occur as a result of denervation of the sympathetic supply to the pupil and superior tarsal muscle respectively, while anhidrosis happens due to denervation of the sweat glands. Lesions distal to the branching of the nerves innervating the sweat glands and vasculature of the face may exclude facial anhidrosis. Etiologies of Horner syndrome can include intracranial pathology (eg, stroke, trauma, multiple sclerosis) as well as diseases that compress the nerves along the sympathetic pathway such as malignancies involving the lung apex, subclavian artery aneurysm, and internal carotid artery dissection. Diagnostic workup should include head, neck, and/or chest cross-sectional imaging (eg, CT scan, MRI) depending on the suspected etiology. Appropriate therapeutic management is based on the underlying etiology.

107 ---------- Exam Section 3: Item 7 of 50 National, Board pf Medical Examiners ment 7. A 2-year-old boy has had recurrent bacterial infections with markedly increased absolute neutrophil counts in the blood (30,000-150,000/mm3). Studies of neutrophils show: Patient (% Control) Test Substance Bacterial killing opsonized patient Normal 80±20 serum opsonized normal Patient 70±15 serum Phagocytosis opsonized patient Normal 85±20 serum opsonized normal Patient 60±20 serum Chemotaxis patient C5a patient C5a Normal 90±15 Patient 10+5 Which of the following is the most likely diagnosis? O A) Chronic granulomatous disease O B) Leukocyte adhesion deficiency O C) Myeloperoxidase deficiency O D) NADPH oxidase deficiency O E) Neutrophil-specific granule deficiency

B. defect in the attachment of leukocytes to the vascular endothelium, which consequently results in the impaired recruitment and migration to sites of extravascular inflammation or infection. LAD type 1 is due to a Leukocyte adhesion deficiency (LAD) results from dysfunctional LFA-1 integrin (ČD18) protein on the leukocyte surface, which does not allow for the normal attachment of neutrophils to the vascular endothelium. It is typically characterized by recurrent bacterial infections, impaired wound healing, a delayed detachment of the umbilical cord after birth, and complete lack of neutrophils at sites of infections with no pus. LAD type 3 is due to dysfunctional integrin response to chemoattractants (such as C5a, IL-8, leukotriene B4), and presents with a similar clinical syndrome to LAD type 1. The actions of leukocyte phagocytosis and bacterial killing are not impaired. Laboratory studies in patients with LAD will show increased absolute neutrophil counts in the blood due to failure to marginate. Neutrophils from the patient's serum will perform similarly to controls with respect to phagocytosis and bacterial killing. In this case, the neutrophil response to C5a is diminished relative to the control, which suggests an impaired chemotactic and adhesive response and a diagnosis of LAD. Incorrect Answers: A, C, D, and E. Chronic granulomatous disease (Choice A) results from a defect or deficiency in the NADPH oxidase complex (Choice D). This impairs the production of reactive oxygen species and the respiratory burst in neutrophils, increasing a patient's susceptibility to catalase positive pathogens. Diagnosis is made by an abnormal dihydrorhodamine test, or a nitroblue tetrazolium reduction test. Decreased bacterial killing compared to control would be expected with opsonized patient serum. Myeloperoxidase deficie

197 ---------- Exam Section 4: Item 47 of 50 National Board OPT Medical Examiners® nent * 47. An investigator is conducting a study of the efficacy of certain vitamins in helping to prevent heart disease in women. Eight thousand women, 45 years of age or older, are enrolled in the study. Two thousand women receive vitamin C only, 2000 receive vitamin E only, 2000 receive both vitamins C and E, and 2000 receive a placebo. The investigator finds that the vitamins are ineffective in helping to prevent heart disease. Which of the following best describes the study design used in this clinical trial? A) Crossover B) Cross-sectional O C) Factorial O D) Nested O E) Simple

C. A factorial study design can be used to investigate the interactions between two or more treatments. In this study design, combinations of the treatments are given to different patient groups. It is a form of true experimental design. Its advantages include that researchers may evaluate multiple independent variables simultaneously. Researchers can also evaluate interactions between variables. In this way, factorial design may be more comprehensive in understanding a phenomenon than a study that focuses on a single factor. In this study design, independent variables are called factors while different values or doses of a factor are referred to as levels. Incorrect Answers: A, B, D, and E. In a crossover study design (Choice A) subjects receive multiple treatments in sequence. Crossover designs can be randomized. Crossover studies diminish confounding variables because each patient serves as their own control. Because of this, imbalances in the randomization are theoretically impossible. Crossover designs also increase statistical power as the number of control patients and the number of treatment patients is increased simultaneously. A cross-sectional study (Choice B) seeks to identify the prevalence of the condition at a particular point in time. An example of a cross-sectional study would be a single survey of a population inquiring whether patients have coronary artery disease and concurrently inquiring about activity levels and diet. Thus, the risk factor and the outcomes are measured simultaneously. The study does not follow patients over time. All information is coilected at a single time point. Nested (Choice D) case-control studies are a variation of a standard case-control study in which the cases and controls are drawn from a larger cohort study. In many cases, the nested case-control offers increased stat

188 ---------- Exam Section 4: Item 38 of 50 National, Board pf Medical Examiners diveB A iance ment * 38. A 68-year-old woman with a 20-year history of hypertension is brought to the emergency department 30 minutes after the sudden onset of right-sided weakness. Her speech is dysarthric. Physical examination shows drooping of the right side of the face. Visual field testing shows no deficits. Muscle strength is 4/5 in the right upper extremity and 3/5 in the right lower extremity. Deep tendon reflexes are 2+ in the upper extremities and left lower extremity and 3+ in the right lower extremity. Sensation is intact throughout. Babinski sign is present on the right. Mental status examination shows no abnormalities. Which of the following is the most likely cause of this patient's condition? O A) Border zone hypoperfusion O B) Embolism of the right anterior cerebral artery O C) Lacunar infarct of the internal capsule D)

C. A lacunar infarct of the left internal capsule would explain this patient's contralateral hemiparesis. Lacunar infarcts are small non-cortical infarcts that occur due to the blockage of small penetrating arteries that branch from the large arteries of the Circle of Willis. Chronic hypertension and diabetes lead to endothelial damage with subsequent hyaline thickening and sclerosis of the arterial wall, resulting in blockage and ischemic damage. Lacunar infarcts most commonly affect the internal capsule or basal ganglia. The internal capsule primarily contains the descending corticospinal tract, though also contains axons of various additional pathways, including third-order sensory neurons traveling from the thalamus to the primary sensory cortex. Internal capsule lacunar infarcts can present with purely motor deficits that cause contralateral hemiparesis, ataxic hemiparesis (contralateral hemiparesis plus ataxia), or clumsy hand-dysarthria (hand weakness plus articulation difficulties). Occasionally, sensory symptoms can accompany the motor weakness. Incorrect Answers: A, B, D, and E. Border zone hypoperfusion (Choice A), which refers to infarcts of brain areas between the major cerebral arteries (eg, anterior, middle, and posterior cerebral arteries), typically occurs in states of systemic hypoperfusion such as hypovolemic shock. Symptoms may include cortical blindness, stupor, and weakness of the bilateral proximal upper and lower extremities. Embolism of the right anterior cerebral artery (Choice B) would likely present with left-sided sensory and motor deficits of the lower extremity due to an infarct of the medial primary motor and sensory cortices. In this patient, sensation is intact, and the upper extremities and face demonstrate weakness, which is inconsistent with an anterior cerebral artery embolism. Rup

158 ---------- Exam Section 4: Item 8 of 50 National, Board pf Medical Examiners ament V 8. A 40-year-old woman comes to the physician because of a 2-month history of increasing redness around her nose and cheeks, and pimples around her mouth. She appears very upset and tells the physician, "I look like a teenaged Santa Claus." Which of the following initial responses by the physician is most appropriate? O A) "Can you tell me how often you go out in the sun and what kind of protection you use?" O B) "Don't worry. Let me assure you this is not a serious condition." C) "It's upsetting when you have a skin problem on your face." O D) "Maybe a little. At least you are keeping your sense of humor about the situation." E) "You really do look rather odd. I can understand why you are so upset." %3D

C. Emotional validation can strengthen the therapeutic alliance and relieve patients' distress. There are methods of emotional validation: being present (actively listening and indicating understanding via nonverbal cues such as nodding), accurate reflection (restating a patient's words), mindreading (inferring the emotional content behind the patient's words), and normalizing (recognizing the patient's reaction as a response any other person would have). This physician chose to infer this patient's emotion from her statement and nonverbal cues. Incorrect Answers: A, B, D, and E. Asking about the patient's clinical risk factors for this skin condition (Choice A) fails to validate the patient's emotions. This response may be perceived as emotionally distant and would not strengthen the physician-patient relationship or relieve patient distress. Reassuring the patient that her skin condition is not serious (Choice B) may be appropriate later in the conversation. However, this patient is less concerned now about the seriousness of the condition and more concerned about how others will perceive the condition. The physician should initially focus on the patient's concerns that are causing the most distress. Physicians should not critique a patient's aesthetic appearance (Choices D and E), which is unrelated to the physician's role to treat medical illness and would likely hurt the patient's feelings. Commenting on a patient's appearance in general, even if the physician has made a positive observation, can be perceived as unprofessional and outside of a physician's scope. The praise and validation in the second parts of Choices D and E, respectively, may be appropriate at some point in the conversation. Educational Objective: Emotional validation, including inferring a patient's underlying emotion, can strengthen the therap

124 ---------- Exam Section 3: Item 24 of 50 Natjonal, Board of Medical Examinersmen' ancive Rasic Sahnce 24. Following blockage of a cerebral artery, the neurons that die from hypoxia release a neurotransmitter that promotes excessive calcium influx, lysis, and cell death in nearby undamaged neurons. Which of the following substances is the primary cause of this cascade of excitotoxicity? O A) Acetylcholine O B) y-Aminobutyric acid C) Glutamate O D) Glycine O E) Norepinephrine

C. Excitotoxicity describes the phenomenon in which neurons endure additional damage or cell death beyond an initial inciting insult such as a traumatic brain injury, stroke, spinal cord injury, neurodegenerative disease, or withdrawal from y-aminobutyric acid (GABA) receptor agonists (eg, alcohol, benzodiazepines). In response to damage to the central nervous system (CNS), glutamate may be released rapidly into the synaptic clefts. When such concentrations persist at high levels, neuronal apoptosis ensues. Unchecked excitatory stimulus from glutamate results in excessive stimulation of the neuron, frequent firing of action potentials, and calcium influx into the cell with resultant activation of phospholipases and proteases. Calcium influx is believed to play a key role in neuronal apoptosis through the activation of intracellular enzymes including calpain and through increased mitochondrial membrane permeability. Such dysregulation can lead to activation of caspases leading to apoptosis. Damage to subcellular organelles, membrane lipids, and DNA can also trigger the intrinsic apoptotic cascade. Excitotoxicity can occur from excess glutamate, as in this example, or from N-methyl-D-aspartic acid (NMDA). Excitotoxicity can be further exacerbated by concomitant dysregulation of glucose; hypoglycemia in the setting of glutamate excitotoxicity can further damage the neurons and worsen the extent of neuronal loss. Incorrect Answers: A, B, D, and E. Acetylcholine (Choice A) is a neurotransmitter that has mixed excitatory and inhibitory activities. Active in the neuromuscular junction, autonomic nervous system, and brain, acetylcholine is an excitatory neurotransmitter in the brain, where it mediates attention, arousal, memory, and motivation. Acetylcholine deficits underlie dementia. Acetylcholine is not involved in excitoto

105 ---------- Exam Section 3: Item 5 of 50 Natjonal, Board of Medical Examinersment amprebensive Racic S nceS S2 S, Time 5. The diagram depicts a cardiac murmur heard in a healthy 92-year-old man. Which of the following age-related processes is responsible for the murmur? A) Calcification of the mitral valve O B) Enlargement of the mitral valve annulus C) Fibrosis and calcification of the aortic valve O D) Myxomatous degeneration of the aortic valve cusp E) Myxomatous degeneration of the mitral valve

C. Fibrosis and calcification of the aortic valve is responsible for the murmur depicted, which shows a systolic crescendo-decrescendo murmur of aortic stenosis. Calcification of the aortic valve is a pathologic consequence of mechanical stresses on heart valves, and results from repetitive microtrauma from the opening and closing of valve leaflets with associated chronic inflammation. Many people will develop some degree of aortic valve stenosis (AS) over time, but structural abnormalities of the valve, such as a bicuspid aortic valve, alter the biomechanics of valve opening and closing and increase the likelihood of earlier calcification and resultant stenosis. The severity of AS is graded on echocardiography through the measurement of valve area, peak velocity, and mean pressure gradient across the valve. While many patients may be asymptomatic, those with severe AS may complain of fatigue, shortness of breath, cough, diminished exercise tolerance, angina, or syncope with exertion. Examination findings include a crescendo-decrescendo systolic murmur best heard at the upper right sternal border, and pulsus parvus et tardus (weak and delayed) may be noted on examination of peripheral pulses. Due to the chronic increased afterload from a fixed obstruction by the valve, left ventricular hypertrophy and resultant diastolic dysfunction can occur. Severe symptomatic aortic stenosis is an indication for valve replacement. Incorrect Answers: A, B, D, and E. Calcification of the mitral valve (Choice A) can result in mitral stenosis, classically heard as an opening snap, followed by a diastolic rumble that is loudest over the cardiac apex and radiates to the axilla. Mitral stenosis is commonly seen as a manifestation of rheumatic heart disease. Enlargement of the mitral valve annulus (Choice B) causes mitral regurgitation, whi

193 ---------- Exam Section 4: Item 43 of 50 Natjonal, Board of Medical Examinersment aiveB A lance 43. A 15-year-old girl with acute myelogenous leukemia undergoes a bone marrow transplant procedure. Her 19-year-old brother is the donor. Blood group and human leukocyte antigen (HLA) typing of patient and donor leukocytes are: Patient Donor АВ, Rh- positive 2, 3 5, 7 2, 5 Blood Group A, Rh-positive HLA-A 2, 3 5, 19 2, 5 HLA-B HLA-DR Three weeks later, she develops a temperature of 37.4°C (99.3°F); a diffuse, erythematous rash over most of the body, including the palmar and plantar surfaces; and watery diarrhea. Serum studies show an AST activity of 120 U/L (N=5-55) and ALT activity of 180 U/L (N=5-45). Which of the following is the most likely diagnosis? O A) Acute rejection O B) Cytomegalovirus infection C) Graft-versus-host disease O D) Hepatitis C O E) Hyperacute rejection O F) Leukemia recurrence O G) Serum s

C. Graft-versus-host disease (GVHD) is the most likely cause of this patient's fever, diarrhea, rash, and increased liver enzymes following a bone marrow transplant. Prior to the grafting of allogeneic stem cells in a bone marrow transplant, recipient native marrow must be ablated. It is replaced with donor marrow, which recolonizes the marrow cavity. Successful transplantation results in reconstitution of the immune system with the donor's immune cells. GVHD occurs when the donor's immune system recognizes the host's tissues as foreign and mounts an immune response against them. Manifestations of acute GVHD include rash, ranging from a maculopapular to blistering rash, diarrhea and abdominal pain, and hepatitis with hyperbilirubinemia. Diagnosis should be suspected in patients with any of these symptoms in the setting of a recent bone marrow transplant. Ideally, donors and recipients should match at all HLA loci. Those receiving transplants from siblings where there is a single mismatch, as in this patient, should ideally receive high resolution typing of HLA-DRB1, and should match at seven out of the eight loci. Mismatches of any kind predispose to GVHD, although HLA-A, B, and DR appear to be the most important. Medications used in the treatment of GVHD act either by directly inhibiting production of T lymphocytes or by suppressing the normal function of these cells. Examples include mycophenolate mofetil, which inhibits the synthesis of nucleotides needed for lymphocyte proliferation, etanercept, a TNF-alpha receptor inhibitor, pentostatin, a purine analog that inhibits T lymphocyte proliferation, sirolimus, which inhibits mTOR to suppress T lymphocyte proliferation, and anti-thymocyte globulin, which directly targets T lymphocytes. Incorrect Answers: A, B, D, E, F, and G. Acute rejection (Choice A) describes failur

169 ---------- Exam Section 4: Item 19 of 50 Natjonal, Board of Medical Examinersment 19. A researcher in a pharmaceutical company designs a new protease inhibitor that inhibits replication of HIV in T lymphocytes in culture. In subsequent assays, which of the following findings is most likely to indicate that the compound is working specifically as a protease inhibitor? O A) The drug prevents integration of proviral DNA into the host genome O B) The RNA is partially reverse transcribed into proviral DNA C) There is a lack of a mature core O D) Transcription from the HIV promoter is blocked E) The virus does not bind to CD4 in the presence of the drug

C. Lack of a mature core would indicate that the new protease inhibitor is specific to that function. HIV drugs are separated into several classes and each of them work via a different mechanism to prevent viral replication. Drugs of disparate mechanisms are combined to inhibit HIV replication at multiple steps, thereby preventing the development of resistance. HIV infection and replication can be broken into seven primary steps: binding, fusion, reverse transcription, integration, replication, assembly, and budding. After HIV binds to the surface of CD4 cells via interaction of its specific glycoproteins with CD4 receptors such as CCR5, the virus fuses with the cell membrane and releases its contents into the cytoplasm. HIV RNÁ is reverse transcribed by reverse transcriptase to create HIV DNA, followed by incorporation of this DNA into the CD4 cell genome. Once integrated, HIV hijacks the cellular machinery to produce the HIV genome and all the requisite proteins to form a new HIV viral particle. The new virus then buds off of the CD4 cell and is released to infect other lymphocytes. However, the viral particles are immature as they contain long proteins that have not yet been cleaved, a task accomplished by HIV protease. Following protein cleavage, the HIV viral particle core has reached maturity. Protease inhibitors block this step but have no effect on the preceding steps of HIV replication. Ás such, a pure protease inhibitor would allow for normal replication and budding of the HIV virus, but the virus would remain immature and non-infectious as a result of failure to undergo the final step of protein cleavage. Incorrect Answers: A, B, D, and E. The drug prevents integration of proviral DNA into the host genome (Choice A) is not correct, as protease inhibitors do not inhibit this step of HIV replication. Inhibit

183 ---------- Exam Section 4: Item 33 of 50 Natjonal, Board of Medical Examinersment 33. A 28-year-old woman, gravida 3, para 2, comes to the physician to request an elective abortion 2 days after she received a positive home pregnancy test result. Her last menstrual period was 6 weeks ago. Ultrasonography shows a uterus consistent in size with a 6-week gestation. After counseling, she decides to undergo a medication-induced abortion rather than a surgical abortion. An oral medication is administered, and she is instructed to return to the office in 2 days to receive a second medication. The first drug administered most likely belongs to which of the following classes? O A) Human chorionic gonadotropin antibodies O B) Oxytocics C) Progesterone antagonists D) Prostaglandin synthesis inhibitors O E) Selective estrogen receptor modulators

C. Mifepristone is a competitive inhibitor of progesterone and is commonly used along with misoprostol for the elective termination of pregnancy. Mifepristone blocks the effects of progesterone on the uterus, causing dilation of the cervix. It also causes the placenta to separate from the uterus due to changes in the endometrium and primes the uterine muscles for the action of misoprostol, which is a age, with mildly less efficacy after 10 weeks. Side effects of this medication regimen include uterine cramping, vaginal bleeding, nausea, vomiting, joint pain, incomplete termination, and rarely pelvic inflammatory disease. Mifepristone should not be used in adrenal failure, chronic steroid use, porphyria, bleeding diatheses, or ectopic pregnancy. tic pros aglandin that causes uterine contractions. This combination of medications can be used up to 77 days of gestational Incorrect Answers: A, B, D, and E. Human chorionic gonadotropin (hCG) antibodies (Choice A) would also lead to abortion, as the lack of hCG would cause the corpus luteum to degenerate, resulting in the subsequent loss of progesterone and disruption of the placental-endometrial interface. However, antibodies are not used to induce medical abortions. Oxytocics (Choice B) are medications that work similarly to endogenous oxytocin, causing uterine contraction, particularly in the setting of uterine atony. They are commonly used to induce delivery but are not used to induce the elective termination of pregnancy. Prostaglandin synthesis inhibitors (Choice D) include non-steroidal anti-inflammatory drugs (NSAIDS), COX-2 selective inhibitors, and corticosteroids. Decreased levels of prostaglandins would cause a decrease in uterine contraction, making these medications an inappropriate choice for medical abortion. Selective estrogen receptor modulators (Choice E) i

165 ---------- Exam Section 4: Item 15 of 50 Natjonal, Board of Medical Examinersment. ivR ance * 15. A 31-year-old woman is brought to the emergency department by her husband because of difficulty breathing and severe muscle weakness for 10 minutes. She has a 3-year history of myasthenia gravis treated with neostigmine. The husband reports that she doubled her dosage 2 days ago because she was feeling extraordinarily weak, but her weakness has increased since then. Which of the following events is the most likely cause of the increased muscle weakness in this patient? O A) Autoimmune hyperactivation of nicotinic receptors O B) Autoimmune inactivation of muscarinic receptors O C) Desensitization of nicotinic receptors O D) Excessive degradation of acetylcholine O E) Hypersensitization of muscarinic receptors F) Insufficient release of acetylcholine

C. Myasthenia gravis is an autoimmune disorder in which the body makes antibodies against nicotinic acetylcholine receptors at the postsynaptic membrane of the neuromuscular junction, rendering them unresponsive to acetylcholine. It commonly presents with ptosis, diplopia, dysphagia, and muscle weakness, which are worse with repetitive activity and later in the day. Neostigmine is an acetylcholinesterase inhibitor, which decreases the breakdown of acetylcholine in the neuromuscular synaptic junction and increases its duration of action. It is often used in the treatment of myasthenia gravis, as it allows the concentration of acetylcholine in the neuromuscular junction to increase, maximizing the receptor binding and increasing the likelihood of muscular depolarization with a neural impulse. Complications of neostigmine include bradyarrhythmias and bronchoconstriction. Overdoses of neostigmine can lead to a cholinergic crisis with respiratory muscle weakness, paralysis, diarrhea, and changes in vision due to desensitization of the nicotinic receptors by prolonged exposure to acetylcholine. Treatment includes the administration of atropine. Incorrect Answers: A, B, D, E, and F. Autoimmune hyperactivation of nicotinic receptors (Choice A) would lead to muscle contraction followed by weakness, hypertension, tachycardia, mydriasis, urinary retention, and diaphoresis. She has no other symptoms associated with this condition, and a secondary primary autoimmune disorder is a less likely cause of her symptoms than desensitization of nicotinic receptors in the setting of high doses of neostigmine. Autoimmune inactivation of muscarinic receptors (Choice B) would lead to mydriasis, bronchodilation, dry skin, increased body temperature, tachycardia, urinary retention, and constipation. Muscarinic receptors do not play a role in ske

110 ---------- Exam Section 3: Item 10 of 50 Natjonal, Board pf Medical Examinersment V 10. A 56-year-old man comes to the physician for a routine examination. His stool is positive for occult blood. Colonoscopy shows a pedunculated growth that is coarsely lobulated and 1.4 cm in diameter. The excised lesion is covered by colonic epithelium and composed of branching glands lined by tall hyperchromatic cells. Some areas appear mildly dysplastic, but there is no evidence of invasion of the stalk. Which of the following oncogenes is most likely to be activated in this lesion? O A) erb O B) fos С) К-ras OD) p53 (TР53) O E) Rb O F) src

C. Neoplasms of the colon (benign or malignant) have their molecular pathogenetic roots in the progressive accumulation of mutations that lead to disordered or dysregulated cell growth, loss of control of the cell cycle, evasion of immune detection or destruction, angiogenesis, and loss of DNĀ damage repair capability. Colonic neoplasia often begins with the formation of a polyp, a small growth that may be flat, sessile, or pedunculated (as in this case). Histologically, polyps may be adenomatous (neoplastic, glandular, tubular, tubulovillous, or villous as in this case), serrated (premalignant), hamartomatous (disorganized growth of native tissue), or hyperplastic. Adenomatous polyps typically occur due to chromosomal instability; most precursor mutations involve the APC gene, a tumor suppressor. When both copies of APC are mutated, increased cellular proliferation may occur. Inherited mutations of ÁPC are common in familial adenomatous polyposis syndrome. A second mutation that follows often involves the K-ras oncogene. K-ras codes for a signaling protein involved in MAP-kinase and PI3-K pathways that promote cell growth, cell differentiation, and cell survival. Only a single mutated allele in oncogenes (in contrast to the requirement for two mutated alleles in tumor suppressor genes) is needed to promote neoplasia. When constitutively active, uncontrolled growth and division can result, reflected grossly as an adenomatous polyp. Additional mutations in p53 (a tumor suppressor) that follow mutations in K-ras may permit sufficient loss of cell cycle control resulting in malignant transformation. This pathway (APC, K-ras, p53 mutations generally in sequence) accounts for 80% to 85% of sporadic cases of colorectal carcinoma. The other 15% of cases are due to microsatellite instability from DNA mismatch repair, includi

166 ---------- Exam Section 4: Item 16 of 50 Natjonal, Board pf Medical Examinersment 16. A 25-year-old woman at 38 weeks' gestation undergoes an oxytocin stress test to evaluate the integrity of the placental circulation. Based on the results, a full-scale induction of labor with oxytocin is started. Treatment with oxytocin is most likely to activate which of the following substances in this patient's uterine smooth muscle? O A) Adrenergic nerve terminals O B) Cholinergic nerve terminals C) Ligand-gated calcium channels O D) Na+-K+ ATPase O E) Voltage-gated chloride channels

C. Oxytocin is commonly used to promote uterine contractions during labor. Oxytocin receptors are expressed by the uterine myometrial smooth muscle cells and myoepithelial cells of the mammary gland. The oxytocin receptor (OTR) belongs to the rhodopsin-type group of G-protein-coupled receptors. Binding of oxytocin to the OTR triggers a signaling cascade which results in activation of ligand-gated calcium channels, leading to increased calcium influx into the uterine smooth muscle cells. The resultant increase in intracellular calcium leads to increased calcium-calmodulin complex activation of myosin light chain kinase (MLCK). Myosin light chains in muscle cells are then phosphorylated by MLCK, leading to cross-bridge formation between myosin heads and actin filaments to generate smooth muscle contraction. Incorrect Answers: A, B, D, and E. Adrenergic nerve terminals (Choice A) in smooth muscle cells are coupled to signal transduction pathways that activate adenylyl cyclase and result in increased intracellular CAMP concentrations. Cyclic AMP inhibits the activity of myosin light chain kinase, leading to smooth muscle relaxation. Cholinergic nerve terminals (Choice B), also called muscarinic receptors, are found on uterine smooth muscle cells. Multiple subtypes of the receptor are present with different signal responses. Oxytocin does not activate these receptor types. + + Na*-K* ATPase (Choice D) is an essential transmembrane enzyme that transports three Na* ions out of the cell in exchange for two K* ions into the cell. Increased Na*-K* ATPase activation indirectly leads to an increased activity of the Na*-Ca-t antiporter, which removes Ca-* from the cell. Decreased intracellular Ca-* results in decreased smooth muscle contractility. Voltage-gated chloride channels (Choice E) are involved in the regulation of the tran

115 ---------- Exam Section 3: Item 15 of 50 National, Board pf Medical Examiners ment NONSMOKERS SMOKERS 92+ 245+ 221-244 203-220 245+ 221-244 203-220 182-202 86-91 86-91 81-85 81-85 76-80 76-80 182-202 <76 <182 <76 <182 Diastolic blood pressure quintile, (mm Hg) Cholesterol quintile, (mg/dL) Diastolic blood pressure quintile, (mm Hg) Cholesterol quintile, (mg/dL) 15. The graphs show the combined effect of total serum cholesterol concentration and diastolic blood pressure on age-adjusted coronary artery disease death rates for both smokers and nonsmokers. Two persons, X and Y, have the following data: Diastolic Blood Total Serum Smokes Pressure Cholesterol 72 mm Hg 95 mm Hg 180 mg/dL 250 mg/dL no Y yes Compared with X, which of the following is the relative risk of dying from coronary artery disease for Y? O A) 2 O B) 6 о С) 12 D) 30 O E) 55

C. Relative risk (RR) describes the difference in likelihood of the occurrence of a particular disease outcome between two groups of patients with or without a particular exposure. In this example, the combined risk of death from coronary artery disease as evidenced by diastolic blood pressure and cholesterol is compared between groups of smokers and nonsmokers. Calculations of relative risk are commonly performed in cohort studies. RR is calculated by dividing the fraction of patients with a positive exposure and who developed disease (a) amongst all patients who were exposed (a + b) by the fraction of patients with a negative exposure and who developed disease (c) amongst all patients who were not exposed (c + d). RR thus equals (a i (a + b)) / (c / (c + d)). In this case, the data is presented as the fraction of the cohort (per 10,000), meaning that the sums a + b and c + d are already accounted for. The numerator in this case (deaths from coronary artery disease in smokers with diastolic blood pressure of 92+ mm Hg and total serum cholesterol of 245+ mg/dL) is 60 deaths per 10,000 persons. The denominator in this case (deaths from coronary artery disease in nonsmokers with diastolic blood pressure of less than 76 mm Hg and total serum cholesterol less than182 mg/dL) is 5 deaths per 10,000 persons. The RR is calculated as follows: RR = (60 deaths) / (5 deaths) = 12. RR values greater than 1.0 indicate an increased risk of developing disease in association with the exposure, whereas values less than 1.0 indicate a reduced risk of developing disease, and RR equal to 1.0 indicates that the disease outcome and the exposure are not related.Incorrect Answers: A, B, D, and E. 2 (Choice A) reflects the RR of death from coronary artery disease among smokers with diastolic blood pressure of 92+ mm Hg and total serum cholester

161 ---------- Exam Section 4: Item 11 of 50 National, Board pf Medical Examiners nsive ent 11. A 60-year-old man develops palpitations over a 6-week period during which he has been drinking approximately 8 cups of coffee per day. A rhythm strip from his ECG is shown. Which of the following is most likely responsible for the palpitations in this man? O A} Secend-degree atrioventricular block O B) Sinoatrial exit block C) Supraventricular premature beats O D) Third-degree atrioventricular block O E}Ventricular tachycardia

C. Supraventricular premature beats account for this patient's palpitations and are apparent on the rhythm strip at beat three and beat seven, followed by compensatory pauses and resumption of sinus rhythm. Normal cardiac conduction starts with an electrical impulse in the sinoatrial (ŠA) node located in the upper right atrium, which is depicted on the rhythm strip as the P wave. The electrical impulse is conducted throughout the atria, causing coordinated contraction of the right and left atria with active filling of the ventricles. The electrical impulse reaches the atrioventricular (AV) node located in the interatrial septum. Conduction is temporarily slowed in the AV node to allow for complete active and passive filling of the ventricles (PR interval), followed by conduction of the electrical impulse down the His-Purkinje system in the interventricular septum, then to the right and left bundles, and finally to the myocardial cells of the ventricles. This results in contraction of the right and left ventricles (QRS complex) with expulsion of blood into the pulmonary and systemic circulations, respectively. The ventricles subsequently repolarize (T wave) and relax to allow filling for the next cardiac cycle. A premature supraventricular beat, also called a premature atrial contraction (PAC), occurs when an electrical impulse is generated at a location in the atria outside the SA node. If a P wave is visualized. it will have an abnormal appearance, but typically the P wave cannot be seen as it is hidden in the T wave from the prior beat. The impulse travels through the AV node and conduction system, so the QRS complex appears similar to the other sinus beats. There is a compensatory pause following a PAC because the ectopic beat causes the SA node to become temporarily refractory. PACS are often seen in patients with

129 ---------- Exam Section 3: Item 29 of 50 Natjonal, Board of Medical Examinersment * 29. A 20-year-old woman comes to the physician because her menstrual period is 1 week late. Menses had occurred at regular 28-day intervals. She had unprotected sexual intercourse with her boyfriend 20 days ago. A urine pregnancy test result is positive. Which of the following best describes the stage of development of the embryo at this time? A) The cytotrophoblast is present, but the syncytiotrophoblast has not yet formed B) Gastrulation is complete, but there are only two germ layers O C) The neural plate is present, but the neural tube is not yet complete O D) Placental development is complete, but the embryo is resistant to teratogens O E) The sclerotome cells have begun migrating, but the somites have not yet formed

C. This patient's embryo has been developing for at most 20 days. At this time, the embryo has progressed through the stages of a blastocyst (week one), bilaminar disc (week two), trilaminar disc, and gastrulation has occurred or is occurring. Three germ layers are present or are being formed, and the primitive streak (epiblast invagination), notochord, and neural plate are present. Progressing forward through the fourth to eighth weeks of embryogenesis, development will include formation of the neural tube, neuroectoderm, primitive organs, and limb buds. Incorrect Answers: A, B, D, and E. The cytotrophoblast is present but the syncytiotrophoblast has not yet formed (Choice A) is incorrect as by the third to fourth week of gestation, the syncytiotrophoblast has formed. The syncytiotrophoblast begins to form initially during blastocyst implantation which occurs in week one. Gastrulation is complete, but there are only two germ layers (Choice B) is incorrect, as gastrulation is recognized as the process that generates three germ layers (the ectoderm, mesoderm, and endoderm). It occurs during the third week. Placental development is complete, but the embryo is resistant to teratogens (Choice D) is incorrect, as the placenta develops over several weeks to months. The embryo is highly sensitive to teratogens at this time. Once organogenesis and cell differentiation are complete in the second and third trimester, sensitivity to teratogens decreases. The early first trimester is when the embryo is most sensitive. The sclerotome cells have begun migrating, but the somites have not yet formed (Choice E) is incorrect as a sclerotome is a subdivision of a somite. Somites are segmental mesodermal derivatives oriented axially and adjacent to the neural tube. They subdivide into sclerotomes, myotomes, and dermatomes that migrate, gi

114 ---------- Exam Section 3: Item 14 of 50 Natjonal, Board of Medical Examinersmen' 14. An investigator attempting to develop a vaccine for gonorrhea produces an attenuated organism that he administers to female rats as an inhaled aerosol. Swabs of vaginal secretions show a significant increase in antibodies specific for gonorrheal antigens. Which of the following mechanisms best accounts for this immune response in the genital tract? O A) Fecal-oral transfer of attenuated organisms O B) Synthesis of IgG by circulating lymphocytes exposed to antigens in the lung C) Trafficking of IgA-producing lymphocytes to seed all mucosal sites O D) Transport of attenuated Neisseria gonorhoeae from the lung to the vaginal mucosa E) Transport of circulating IgM across the vaginal epithelium

C. Trafficking of IgA-producing lymphocytes to seed all mucosal sites is the most likely mechanism accounting for the immune response to mucosal exposure to attenuated N. gonorrhoeae. Attenuated organisms are non-viable organisms that still possess the antigens required to induce the formation of specific antibodies against the pathogen via the actions of the adaptive immune system. Plasma cells, which are terminally differentiated B lymphocytes, produce immunoglobulins in large quantities after antigenic stimulation. Each B lymphocyte possesses a unique B lymphocyte receptor created during V(D)J rearrangement in the bone marrow that will bind to a specific antigen. Antigens binding to the B'lymphocyte receptors activate B lymphocytes to recruit helper T lymphocytes, which aid in the proliferation and differentiation of B lymphocytes into immunoglobulin-producing plasma cells. Additionally, B lymphocytes present antigens via major histocompatibility complex-Il to CD4+ Ť lymphocytes, which aid in adaptive immunity. Pathogens that are primarily introduced into mucosal sites are more likely to stimulate the production of IgA immunoglobulins as these are the primary immunoglobulins involved with mucosal immunity. In this study, gonorrhea antigens introduced to the respiratory tract through an inhaled aerosol stimulate the activation of IgA-producing plasma cells. These IgA-producing plasma cells, along with memory B lymphocytes, are then trafficked to other mucosal sites such as the genital tract, which explains the presence of gonorrheal antibodies present in vaginal secretions. Incorrect Answers: A, B, D, and E. Fecal-oral transfer of attenuated organisms (Choice A) would be unlikely to induce such a robust immune response in this instance. While there are some vaccines that are given orally, including the rotavirus, ch

186 ---------- Exam Section 4: Item 36 of 50 Natjonal, Board of Medical Examinersment 36. A 53-year-old man comes to the physician for a follow-up examination. He began antibiotic therapy 1 month ago for acute bacterial pneumonia. At the time of diagnosis, a chest x-ray showed patchy pneumonic consolidation and a 2.5-cm peripheral nodular lesion in the right upper lobe. His symptoms have since resolved, and an x-ray today shows absence of the pneumonic consolidation. However, the lesion is still present. A PPD skin test is nonreactive. A wedge resection is done by thoracoscopy. Histologic examination of the resected tissue shows a solid, well-circumscribed nodule composed predominantly of mature cartilage admixed with small amounts of fibrous tissue and occasional clefts lined by benign respiratory epithelium. Which of the following best describes this patient's pulmonary lesion? O A) Abscess O B) Adenoma O C) Choris

D. A hamartoma is a benign, disorganized overgrowth of histologically normal tissue in its native location within the body. In the lungs, they may consist of hyaline cartilage, adipose, respiratory epithelium, and/or smooth muscle cells. Pulmonary hamartomas are common benign nodules (often presenting as a solitary lung nodule) that are typically asymptomatic and discovered incidentally on imaging. They may occasionally present with hemoptysis, bronchial obstruction, and cough. Radiographic features include a well-circumscribed nodule with either smooth or lobulated borders. Calcification may be present. These lesions should be further evaluated with CT or PET imaging, with consideration of biopsy to rule out malignancy. Histology will show normal, mature cells in a disorganized pattern. Surgical resection is curative. Patients without known risk factors for malignancy (eg, smoking history) and without high-risk features on CT or PET imaging may be followed with surveillance CT scans to monitor for growth instead of undergoing resection. Incorrect Answers: A, B, C, and E. Pulmonary abscess (Choice A) typically appears as a cavitary lesion on chest imaging with an air-fluid level. Patients will present with fever, malaise, and cough with sputum production (which is often foul-smelling). Histologic examination, if indicated, would reveal a necrotic mass of leukocytes and microbes. Adenoma (Choice B) describes a benign neoplasm of epithelial cell origin. Adenomas of the respiratory tract and lungs are rare lesions that may have potential for malignant transformation. They are less common than hamartomas and may present with airway obstruction or post-obstructive pneumonia. Choristoma (Choice C) refers to a mass of histologically normal tissue that has developed in an abnormal location. Examples include Meckel diverticulum

t disease) O B) Osteogenesis imperfecta O G)Rheumatic fever O D) Tuberculosis O E}Vitamin B4-(thiamine) deficiency F) Vitamin D deficiency

F. Childhood vitamin D deficiency (Rickets) presents with multiple skeletal deformities including frontal bossing, nodules at the costochondral junctions of the anterior chest, and delayed closure of the growth plates due to impaired bone mineralization. Children will demonstrate varus bowing of the tibia as well as bowing of the femur. Children will have hypoplastic teeth and multiple caries as mineralization of dentin is impaired. Vitamin D deficiency in children commonly comes from low exposure to UV radiation and low dietary vitamin D intake (particularly from low vitamin D in breast milk). Fat malabsorption syndromes such as celiac disease or cystic fibrosis can impair absorption of vitamin D in the gastrointestinal tract. Incorrect Answers: A, B, C, D, and E. Osteitis deformans (Choice A), also known as

153 ---------- Exam Section 4: Item 3 of 50 National Board of Medical Examiners mont * 3. Which of the following best explains why deoxygenated blood can carry more carbon dioxide for a given Pco2 than oxygenated blood? O A) Deoxyhemoglobin does not bind to 2,3-bisphosphoglycerate as efficiently as oxyhemoglobin O B) Deoxyhemoglobin has a lower capacity to form carbamino compounds than oxyhemoglobin O C) Deoxyhemoglobin has a lower pKa than oxyhemoglobin D) Deoxyhemoglobin is a better buffer of hydrogen ions than oxyhemoglobin E) Oxygen and carbon dioxide compete for the same binding site in hemoglobin O F) Oxyhemoglobin binds nitric oxide with a higher affinity than deoxyhemoglobin

D. Deoxyhemoglobin is a better buffer of hydrogen ions than oxyhemoglobin, which explains why deoxygenated blood can carry more carbon dioxide for a given Pco2 than oxygenated blood. This is known as the Haldane effect. The hemoglobin tetramer exists in two primary conformations, the taut form, which has a reduced affinity for oxygen, and the relaxed form, which has an increased affinity for oxygen. As hemoglobin molecules accept oxygen from the lungs and deliver it to tissues, they undergo repeated conformational change between the taut (when unloaded) and relaxed (when loaded) form. At the level of the alveoli, there is a high partial pressure of oxygen that favors the loading of oxygen onto hemoglobin and unloading of carbon dioxide, while in the periphery there is a lower partial pressure of oxygen that favors the unloading of oxygen from hemoglobin. Carbon dioxide produced by cellular respiration must be transported back to the alveoli for removal. It can travel in the blood dissolved in plasma (~10%), bound to NH2 groups on hemoglobin (~10%), or as bicarbonate (~80%). When hemoglobin unloads oxygen in the tissues and undergoes conformational change, it is more capable of accepting hydrogen ions. The cytosolic increase in pH from the removal of hydrogen ions that are now bound to hemoglobin favors the formation of bicarbonate from carbon dioxide and water. Bicarbonate is a highly soluble form of carbon dioxide and therefore, in the setting of deoxyhemoglobin, more carbon dioxide can be carried in the blood for any given Pco2. Incorrect Answers: A, B, C, E, and F. Deoxyhemoglobin does not bind to 2,3-bisphosphoglycerate (2,3-BPG) as efficiently as oxyhemoglobin (Choice A) is incorrect. 2,3-BPG binds to deoxyhemoglobin with high affinity and promotes further oxygen unloading in the tissues. Deoxyhemoglobin has a low

174 ---------- Exam Section 4: Item 24 of 50 Natjonal, Board of Medical Examinersment 24. A 50-year-old man is brought to the physician 2 days after the onset of double vision and drowsiness. Three weeks ago, he began using over-the-counter cimetidine for dyspepsia. He has epilepsy treated with phenytoin for the past 2 years. Physical examination shows nystagmus and an ataxic gait. Which of the following effects of cimetidine on phenytoin is the most likely cause of the adverse effects in this patient? O A) Altered binding to plasma proteins O B) Altered binding to tissue receptors O C) Altered distribution to active receptors D) Inhibition of metabolism E) Inhibition of renal excretion

D. The cytochrome P-450 (CYP450) superfamily of oxidative enzymes is responsible for the majority of phase I metabolism of drugs. Cimetidine is a potent CYP450 inhibitor, which results in the inhibition of metabolism of multiple drugs, including phenytoin. Phenytoin is an anticonvulsant agent with neurotoxic effects at increased serum levels. The neurotoxic effects can range from mild nystagmus to ataxia, slurred speech, lethargy, coma, and death. Other potent CYP450 inhibitors include tricyclic antidepressants, fluoroquinolones, ketoconazole, amiodarone, quinidine, proton pump inhibitors, diltiazem, isoniazid, chloramphenicol, erythromycin, sulfonamides, metronidazole, ritonavir, and grapefruit juice. In the presence of a CYP450 inhibitor, phenytoin is unable to be metabolized appropriately, resulting in a progressively increasing concentration of phenytoin, which results in the development of neurotoxic symptoms. Incorrect Answers: A, B, C, and E. Altered binding to plasma proteins (Choice A) affects the concentration of physiologically active phenytoin in the serum as the majority of drug is bound to albumin. Patients with hypoalbuminemia have a higher risk of toxicity due to a decreased concentration of bound phenytoin and increased free, physiologically active drug. This is not the mechanism by which cimetidine increases the activity of phenytoin. Altered binding to tissue receptors (Choice B) is not associated with cimetidine use. Phenytoin acts on voltage-gated sodium channels. It binds preferentially to channels in the inactive state, blocking them from converting to the active state. Altered distribution to active receptors (Choice C) occurs with activated charcoal administration in cases of known toxic phenytoin ingestion. Activated charcoal binds phenytoin in the gastrointestinal tract to prevent absorption

103 ---------- Exam Section 3: Item 3 of 50 National Board of Medical Examiners nent * 3. A 2-month-old boy is brought to the physician because of a 7-day history of hoarseness. His mother has AIDS and used cocaine throughout her pregnancy. Physical examination shows no abnormalities. A fiberoptic laryngoscopic examination shows a nodule on the left vocal cord. Which of the following viruses is the most likely cause of this nodule? A) Cytomegalovirus B) Herpes simplex virus O C) HIV O D) Human papillomavirus E) Varicella-zoster virus

D. Human papillomavirus (HPV) is a common human pathogen that infects epithelial cells. There are multiple strains which may be low- or high-risk for the development of squamous cell carcinoma, which result in the development of a variety of lesions. Anogenital manifestations are common (eg, cervical intraepithelial neoplasia, condylomata acuminata). Non-anogenital manifestations include verruca vulgaris, plantar warts, and mucosal lesions of the oropharynx, respiratory epithelium, and larynx. Laryngeal papillomatosis frequently affects the vocal cords and presents as a vocal cord nodule. Symptoms include dysphonia (hoarseness) and may progress to dyspnea, chronic cough, dysphagia, globus sensation, and airway obstruction if the lesion becomes large enough. Risk factors for clinically significant HPV infection include immunodeficiency and injury to the epithelium from another cause (eg, trauma, coinfection). HPV is primarily transmitted through genital contact. In children, the most common vector for HPV infection is transmission from an affected mother during labor and transit through the birth canal, though the presence of these lesions may also raise concern for sexual assault in the correct clinical context. Treatment focuses on maintaining a patent airway, surgical removal of symptomatic lesions, and preserving voice quality. Incorrect Answers: A, B, C, and E. Cytomegalovirus (Choice A), also known as human herpesvirus-5 (HHV-5), can be transmitted through multiple modes, including sexual contact, urine, respiratory droplets, and across the placenta. It can cause a variety of presentations, including mononucleosis in immunocompetent patients, along with retinitis, infectious esophagitis, and pneumonia in immunocompromised patients. It is not associated with laryngeal papillomatosis. Herpes simplex virus (Choice B)

123 ---------- Exam Section 3: Item 23 of 50 Natjonal, Board of Medical Examinersment ansive Basic Saance 23. A 68-year-old man comes to the physician because of a 1-month history of not being able to sustain an erection for sexual intercourse, although he has no difficulty during masturbation. His wife of 40 years died 2 years ago after a prolonged illness. He began dating recently. He has a multinodular goiter but takes no medications. His serum thyroid-stimulating hormone concentration is 4.0 µU/mL, and serum testosterone concentration is within the reference range. He does not have any symptoms of major depressive disorder. Which of the following pairs of additional findings is most likely to be found on history taking? Nocturnal Erections Libido A) Decreased decreased O B) Decreased normal O C) Normal decreased D) Normal normal

D. Male erectile disorder, or erectile dysfunction, is a common condition, especially in older males. Erectile dysfunction can be caused by vascular, neurologic, hormonal, and/or psychological dysfunction. Physicians should ask about the patient's erectile function during masturbation and while sleeping; if the patient continues to have erections while masturbating or sleeping (nocturnal), the erectile dysfunction is more likely due to a psychogenic etiology rather than a vascular, neurologic, or hormonal etiology. This patient had likely adapted psychologically to having sexual intercourse with his late wife such that sexual dysfunction with new partners may be related to adapting to the new sexual partner or to performance anxiety. In the absence of depression, low testosterone, or thyroid dysfunction, patients are likely to possess a normal libido. The treatment depends on the cause. Patients with a psychological etiology respond best to psychotherapy such as cognitive-behavioral therapy. Incorrect Answers: A, B, and C. Libido and nocturnal erections can both be decreased (Choice A) in patients with hormonal causes of erectile dysfunction. Hypogonadism and hypothyroidism are common culprits. Decreased libido with normal nocturnal erections (Choice B) can result from multiple problems: medications (eg, selective serotonin reuptake inhibitors, antiandrogens), alcoholism, recreational drugs, depression, relationship stress, fatigue, and systemic illness. This patient does not possess these risk factors for low libido. Decreased nocturnal erections with normal libido (Choice C) would likely be caused by neurological or cardiovascular problems due to nerve damage or endothelial dysfunction respectively. Neurological causes include stroke, trauma, and neuropathy (eg, from longstanding diabetes). This patient continues to

202 ---------- Exam Section 1: Item 1 of 50 National Board of Medical Examiners Comprehensive Basic Science Self-Assessment 1. A 12-year-old boy is admitted to the hospital because of lethargy, hip pain, and a temperature of 39.4°C (103°F). He has been hospitalized several other times because of pneumonia. His neonatal period was normal. Complete blood counts are within normal limits, and a test for HIV antibody is negative. Blood cultures grow Staphylococcus aureus. Serum electrophoresis is most likely to show which of the following patterns? A E Albumin Y B2 A) B) C) D) E)

D. The electrophoresis in Choice D demonstrates a deficiency in immune globulins. This young patient with S. aureus bacteremia and a history of multiple prior infections has a clinical syndrome consistent with hypogammaglobulinemia, an immunodeficiency reflected by absence of the gamma fraction on protein electrophoresis. Protein electrophoresis separates the major proteins found in the serum by size and polarity. Larger proteins migrate slowly, and proteins with negative charge (eg, albumin) migrate toward the positive pole just as those with neutral or positive charge (eg, weakly negative, neutral, or positive immune globulins) migrate toward the negative pole. The large peak on the left of each of these diagrams is representative of albumin. The alpha fraction is next and is comprised primarily of a 1-antitrypsin, thyroid-binding globulin, and transcortin. The beta fraction contains transferrin and B-lipoproteins in addition to complement. The gamma fraction contains the immunoglobulins, including IgM, IgG, IgD, and IgE. A reduced number of circulating immunoglobulins as is seen in hypogammaglobulinemia, results in a marked reduction or absence of this spike. Incorrect Answers: A, B, C, and E. Choice A shows a narrow monoclonal spike in the gamma region. In the appropriate clinical context, this is suggestive of multiple myeloma, although other syndromes such as Waldenstrom macroglobulinemia, smoldering myeloma, monoclonal gammopathy of undetermined significance, and amyloidosis can demonstrate spikes in this region. Choice B shows a blurring of the beta-2 and gamma regions, sometimes refe rheumatoid arthritis, or cirrhosis. either by increased C-reactive protein or by ed to as beta/gamma fusion. This ac ted ased IgA, which can seen in patients with severe respiratory infections, Choice C shows a large but wide spik

192 ---------- Exam Section 4: Item 42 of 50 National Board of Medical Examiners mont 42. A 60-year-old woman dies of a myocardial infarction. Incidental findings on examination at autopsy include multiple 0.5- to 1-cm stool-filled outpouchings of the sigmoid colon alongside the taeniae coli. The outpouchings protrude through the muscularis propria. Microscopic examination of a lesion shows a thin wall consisting of attenuated colonic mucosa and submucosa. Which of the following mechanisms best explains the formation of these lesions in the colon? O AJAbnormal -development of the museularis propria O B}Atrophy of the muscularis propria O C) Hyperplasia of the columnar epithelium D) Increased intraluminal pressure E) Neoplastic proliferation of the columnar epithelium

D. The layers of the gastrointestinal wall, from the luminal surface to the exterior of the bowel, include the mucosa, submucosa, muscularis propria, and serosa. Diverticulosis is characterized by sac-like protrusions of the colonic wall which are hypothesized to develop secondary to increased intraluminal pressure that predisposes the herniation of mucosa and submucosa through the muscularis propria of the colonic wall. Risk factors for the development of diverticulosis include abnormal colonic motility, high dietary intake of red meat and low dietary fiber, physical inactivity, and obesity. Complications of diverticular disease include bleeding and diverticulitis. Incorrect Answers: A, B, C, and E. Abnormal development (Choice A) or atrophy (Choice B) of the muscularis propria is rare but may result in diminished or abnormal peristalsis of the gastrointestinal system. Diverticulosis is a common phenomenon and is not related to intrinsic abnormalities of the muscularis propria. Hyperplasia of the columnar epithelium (Choice C) results in hyperplastic polyps of the colon, which are most commonly located in the rectosigmoid colon. They are not associated with diverticulosis. They are benign and do not confer an increased risk of malignancy. Contrarily, neoplastic proliferation of the columnar epithelium (Choice E) results in the formation of adenomatous polyps, which may progress to malignancy. Diverticulosis is not associated with colonic polyps or malignancy. Educational Objective: The layers of the gastrointestinal wall, include the mucosa, submucosa, muscularis propria, and serosa. Diverticulosis is characterized by sac-like protrusions of the colonic wall, which are hypothesized to develop secondary to increased intraluminal pressure that predisposes the herniation of mucosa and submucosa through the muscularis pro

145 ---------- Exam Section 3: Item 45 of 50 National, Board pf Medical Examiners mont 45. A 60-year-old woman is receiving cisplatin therapy for advanced transitional cell bladder cancer. She develops paresthesias, and the medication is stopped. The medication likely damaged the largest cells in the region labeled "B" in the drawing of the spinal cord shown. The damage to these cells would most likely lead to which of the following motor signs? O A) Babinski sign O B) Clonus O C) Fibrillations O D) Hyporeflexia E) Muscle atrophy

D. This patient's dorsal root ganglion cells (labeled "B") have likely been damaged by cisplatin, resulting in hyporeflexia due to decreased sensation. Dorsal root ganglia are present at each spinal cord level and contain the cell bodies of first-order sensory neurons. Each sensory neuron begins with sensory receptors in the skin, tendon, joint, or muscle. The axon travels medially, with the cell body extending into the dorsal root ganglion, and the axon continuing on to enter the spinal cord, ultimately synapsing with the second- order neuron in the gray matter of the spinal cord. Symptoms resulting from dorsal root ganglion damage include paresthesias (as in this patient), numbness (resulting in this patient's hyporeflexia), loss of proprioception, abnormal temperature sensation, and decreased vibration sensation. Patients with decreased sensation may be unable to sense and therefore respond to a reflex hammer, representing a disruption in the reflex pathway and manifesting as hyporeflexia. Cisplatin and other platinum compounds used for chemotherapy typically cause symmetrical, predominantly sensory peripheral neuropathy. Treatment is symptomatic, and the neuropathy is sometimes reversible after stopping cisplatin. Incorrect Answers: A, B, C, and E. The Babinski sign (Choice A), or extensor plantar response, is an upper motor neuron sign that indicates damage to the corticospinal tract, a descending motor tract located in the central nervous system. The Babinski sign is present when the lateral part of the plantar surface of the foot is firmly stroked and the hallux extends (moves rostrally) while the other toes abduct. This patient has peripheral sensory nervous system damage rather than central motor nervous system damage. Clonus (Choice B) is a rhythmic stretch reflex that occurs with extreme hyperreflexia, repre

117 ---------- Exam Section 3: Item 17 of 50 National Board of Medical Examiners ancive Rasic Saance S ement 17. During a genome project, an experimental animal is found to have a DNA sequence with similarities to a growth hormone receptor. Which of the following peptide segments shown is the transmembrane segment? A Ala Leu HisAsp Asn Tyr Lys Pro Glu Phe TyrAsn Asp Asp Ser Trp Val Glu Phe Glu Leu Thr Glu Glu Ser Asp Glu Tyr Glu Leu Asp His Gin Lys Ser Leu Gly Ala Lys Asp B Asp Asp Ser Gly Arg Thr Ser Cys Gly Cys Glu Ala Cys Thr Ser Leu CysAla Glu Leu Lys His Ala Ser Lys Gly Ser Thr Arg His Thr Gly Gin Ala Asn Arg Arg Arg Lys His Leu Asp TyrAsn Asp Asp Glu lle Asp lle Asp Asp ThrAsp Glu Thr Glu Glu Ser Asp Thr E Leu Val Leu lle Phe Gly Met lle Gly Val Val lle Val Val Leu Leu Val Leu Leu lle Phe Gly Asp Glu Lys Leu Ala Leu Leu Leu Asp Gin Pro Glu Ser Leu Thr Ala Ala Ala Gly F Lye Ser Gly Leu Ala Glu Gin Arg Leu Ser Cy

E. Amino acids contain an amino group, carboxy group, and side chain that may be nonpolar, polar, acidic, or basic. Nonpolar amino acids include glycine, alanine, valine, leucine, isoleucine, methionine, phenylalanine, proline, and tryptophan. Polar amino acids include serine, threonine, asparagine, cysteine, tyrosine, and glutamine. These amino acids have a polar side chain without a net charge. Amino acids with basic side chains include lysine, arginine, and histidine. Amino acids with an acidic side chain include aspartate and glutamate. Transmembrane proteins such as peptide hormone receptors (eg, growth hormone receptor) have hydrophilic extracellular and intracellular domains, and hydrophobic transmembrane domains. In turn, the structure of transmembrane domains is composed of multiple nonpolar amino acids. At times, these domains may fold to form alpha helices. In the peptide sequence shown, sequence E consists of primarily nonpolar amino acids, which would demonstrate affinity with the hydrophobic phospholipid bilayer, likely comprising the transmembrane domain. Incorrect Answers: A, B, C, and D. Sequences A through D (Choices A, B, C, and D) each include a mixture of polar, acidic, and basic amino acids in addition to occasional (but few) nonpolar amino acids. Each of these domains would have strong hydrophilic or cross-linking characteristics and would demonstrate considerably less affinity for the hydrophobic phospholipid bilayer. These domains are more likely extracellular or cytosolic. Educational Objective: Transmembrane proteins such as peptide hormone receptors (eg, growth hormone receptor) have hydrophilic extracellular and intracellular domains, and hydrophobic transmembrane domains. In turn, the structure of transmembrane domains in humans is composed of multiple nonpolar amino acids that have affini

172 ---------- Exam Section 4: Item 22 of 50 National, Board pf Medical Examiners nent 22. A 68-year-old man with dementia, Alzheimer type, has improvement in memory and cognition with donepezil therapy. Which of the following is the most likely mechanism of this beneficial effect? O A) Blockade of choline reuptake into presynaptic terminals O B) Blockade of effects on the nicotinic receptors on muscle O C) Enhanced influx of calcium into the presynaptic terminal O D) Prevention of acetylcholine release from subsequent action potentials E) Prolongation of acetylcholine effects in the synaptic cleft

E. Donepezil is a cholinesterase inhibitor, which prolongs acetylcholine effects in the synaptic cleft. Alzheimer dementia is the most prevalent dementia, presenting with progressive cognitive decline that begins with short-term memory impairment, progresses to apraxia and language abnormalities, and culminates in behavioral and personality changes preventing the patient from performing basic activities of daily living. Alzheimer dementia is postulated to result from decreased cholinergic signaling in the cortex and basal forebrain. Acetylcholinesterase normally hydrolyzes acetylcholine in synaptic clefts. Donepezil non-competitively and reversibly inhibits acetylcholinesterase activity and thereby increases the amount of synaptic acetylcholine available for neurotransmission. Though donepezil may slow the rate of cognitive decline and modestly improve functionality, donepezil and the other cholinesterase inhibitors are not curative. Incorrect Answers: A, B, C, and D. Blockade of choline reuptake into presynaptic terminals (Choice A) will lead to decreased synthesis of acetylcholine, as reuptake of choline is the rate-limiting step in acetylcholine synthesis. Therefore, this mechanism of action may exacerbate acetylcholine deficits and would not be helpful as an Alzheimer dementia medication. Blockade of effects on the nicotinic receptors on muscle (Choice B) is the mechanism of action of muscle relaxants used in anesthesia such as succinylcholine and atracurium. Alzheimer dementia arises from central-not peripheral-acetylcholine deficiency and would not be affected by medications with this mechanism of action. Enhanced influx of calcium into the presynaptic terminal (Choice C) would increase presynaptic neurotransmitter release from vesicles via exocytosis into the synaptic cleft. Botulism and tetanus cause dysfunctio

121 ---------- Exam Section 3: Item 21 of 50 National Board of Medical Examiners ment V 21. An 8-year-old girl is brought to the physician by her parents because of abdominal pain for 3 days. She is at the 90th percentile for height and the 80th percentile for weight. Breast, pubic hair, and axillary hair development is Tanner stage 3. Pelvic examination shows a firm, smooth, right ovarian mass. Ultrasonography of the abdomen confirms the presence of an 8-cm-diameter ovarian mass. Which of the following types of ovarian lesions is most likely in this patient? O A Gystadenocarcinoma O B}Dermoideyst O G) Embryonal rhabdemyosarcema O Đ} Fibroma E) Granulosa cell tumor O F}Krukenberg-tumor O G) Papillary carcinoma

E. Granulosa cell tumors are a type of malignant sex-cord stromal tumor. They generally occur in women in their sixth decade of life, though can occur at any age, including during childhood. Their presentation is marked by the effect of their functional production of hormones (eg, estrogen). Thus, it may present with precocious puberty, as in this patient, or vaginal bleeding in younger or premenstrual women. Precocious puberty is suspected when girls younger than eight years old or boys younger than nine years old develop secondary sexual characteristics. Tanner stage 3 characteristics, including thickening of pubic and axillary hair and breast enlargement, are typically seen in females ages 11 to 13. The presence of these findings at a significantly earlier age suggests precocious puberty and warrants evaluation. Granulosa cell tumors are typically indolent and may not be detected until large or advanced. On histology, granulosa cell tumors demonstrate Call-Exner bodies, which are granulosa cells arranged around eosinophilic fluid, resembling ovarian follicles. Treatment is through surgical excision, and if the tumor is early stage, prognosis is generally favorable. In the postmenopausal patient, granulosa cell tumors often present with postmenopausal vaginal bleeding, which should prompt investigation. Incorrect Answers: A, B, C, D, F, and G. Cystadenocarcinoma (Choice A) is the most common ovarian malignant neoplasm. They are generally seen in women of advanced age and those having a genetic predisposition to malignancy (eg, BRCA, hereditary nonpolyposis colorectal malignancy). Dermoid cyst (Choice B), also known as a mature cystic teratoma, is a benign tumor of the ovary. They are common germ-cell tumors often seen in women age 20 to 40 years. They contain tissue derived from all three germ layers and as a result,

149 ---------- Exam Section 3: Item 49 of 50 Natjonal, Board of Medical Examinersment andive Rasic Science V 49. A 56-year-old man comes to the physician because of a 3-month history of intermittent regurgitation of material with an acidic taste. He is unable to sleep and has missed 3 days of work during the past month because of the symptom. Various non- pharmacologic measures, including elevating the head of his bed and decreasing his dietary fat intake and portion size, have resulted in only mild improvement. Physical examination shows no abnormalities. Esophageal endoscopy shows severe erosions; there are no other structural abnormalities. The most appropriate pharmacotherapy for this patient is a drug with which of the following mechanisms of action on the parietal cells? O A) Agonism of cholecystokinin B (CCK2) receptor O B) Agonism of prostaglandin E receptor 3 (EP3) receptor O C) Antagonism of histamine-2 (H2

E. Irreversible inhibition of H+-K+ ATPase is the mechanism by which proton-pump inhibitors (PPIS) exert their effect on gastric acid production. Acid reflux in the esophagus causes mucosal irritation and inflammation, which can present as mucosal erythema and multip malignancy marked by intestinal metaplasia of the distal esophagus. Untreated, this can lead to esophageal adenocarcinoma. Parietal cells are the acid secreting cells of the stomach and are found primarily in the gastric body. These cells utilize water and carbon dioxide to create carbonic acid, which dissociates into bicarbonate and hydrogen ions. Bicarbonate is exchanged with chloride and enters the blood stream while hydrogen ions are secreted into the gastric lumen in exchange for potassium via the H*-K* ATPase pump. Chloride is also secreted into the lumen. The accumulation of hydrogen ions results in an osmotic gradient that pulls water into the lumen to help make acidic gastric chyme. PPIS irreversibly inhibit the H*-K* ATPase in gastric parietal cells and thereby limit the secretion of hydrogen ions, which results in an increased gastric pH. The reduction of stomach acid production allows for the neutralization of stomach contents, limiting the inflammatory and erosive sequelae of GERD in the distal esophagus, and allowing for the appropriate healing of previous esophagitis and/or Barrett esophagus. erosions (reflux esophagitis). The entire clinical of disease is known as roesophage reflux disease (GER Over time, if GERD remains untreated, metaplasia can occur leading to the development of Barrett esophag a state of pre- Incorrect Answers: A, B, C, and D. Agonism of cholecystokinin B (CCK2) receptor (Choice A) is normally achieved by the action of gastrin from G cells and CCK from the duodenum. Stimulation of this receptor leads to activation of th

140 ---------- Exam Section 3: Item 40 of 50 National, Board pf Medical Examiners ement V 40. A 64-year-old man with a 25-year history of alcoholism is brought to the emergency department by his wife because of a 1-day history of confusion. He is disoriented and disheveled. His pulse is 110/min, respirations are 20/min, and blood pressure is 100/64 mm Hg. Physical examination shows signs of dehydration and jaundice, and spider angiomata over the face and chest. There is a flapping, up-and-down motion of the hands when the upper extremities are outstretched horizontally. Abdominal examination shows distention with bulging flanks and shifting dullness. In addition to other appropriate pharmacotherapy, administration of oral neomycin is begun. Which of the following primary mechanisms of action is most likely to occur in this patient as a result of this drug treatment? O A) Binding of ammonia and other hepatically clear

E. Killing of bacteria in the gut that generate ammonia is the mechanism of action by which neomycin helps to treat hepatic encephalopathy (HE). Neomycin is an aminoglycoside antibiotic with activity against Gram-negative aerobes that was used frequently in the past for the treatment of HE. It works by inhibiting the 30S subunit of bacterial ribosomes to prevent protein translation. Ammonia is a waste product generated by bacteria in the gut. In its nonionized form, ammonia is readily absorbed by the gut and into the systemic circulation. In cirrhosis, normal mechanisms of detoxification and processing of ammonia via the urea cycle are impaired, and high levels of ammonia play a role in the dysfunction of neurons seen in HE. Killing Gram-negative gut flora reduces the amount of ammonia produced. Neomycin is no longer a preferred therapy for HE as it is absorbed readily and has side effects including acute kidney injury and ototoxicity. Rifaximin is currently the antibiotic of choice for the adjunctive treatment of hepatic encephalopathy, which also targets colonic ammoniagenic bacteria. Incorrect Answers: A, B, C, and D. Binding of ammonia and other hepatically cleared toxins in the gut (Choice A) is accomplished by the action of AST-120, an oral microspherical carbon that binds ammonia directly. It is not currently in clinical use. Blockade of new protein synthesis by the liver (Choice B) is not a mechanism of action of currently used medications for the treatment of hepatic encephalopathy. Many proteins are synthesized in the liver, including coagulation factors and albumin. These are not thought to play a role in HÉ. Blockade of two successive steps in the metabolism of folic acid (Choice C) is the function of trimethoprim-sulfamethoxazole (TMP-SMX). This pathway is required for the synthesis of nucleic acids, whic

196 ---------- Exam Section 4: Item 46 of 50 National Board of Medical Examiners aive B A tance mont 46. A healthy 28-year-old woman participates in an exercise study. Several physiologic variables are measured as she runs on a treadmill. The ambient room temperature is 75°F. She reaches a steady state that increases her oxygen consumption threefold. She continues to exercise at that level for 20 minutes. During the first 5 minutes of exercise, the vascular resistance of which of the following is likely to show the greatest increase from resting values in this volunteer? O A-Goronary O B}Gutaneous O G} Exercising-musele O Đ) Pulmonary E) Splanchnic O F} Systemie

E. Splanchnic vascular resistance is likely to show the greatest increase when compared to its resting state. The splanchnic circulation includes the celiac trunk and the superior and inferior mesenteric arteries, which supply blood to the gastrointestinal system. Adequate splanchnic blood supply is critical to digestion and maintenance of the gastrointestinal mucosal barrier. In the resting state, splanchnic circulation utilizes 25% to 30% of all cardiac output, but this can change drastically after meals (increased blood flow) or during periods of physiologic stress such as exercise or critical illness (decreased blood flow). Changes in splanchnic blood flow are regulated by multiple mechanisms that are grouped into intrinsic, extrinsic, and humoral control. Intrinsic regulatory mechanisms include vasodilation (increased blood flow) in response to tissue hypoxia or acidosis. Extrinsic control is exerted by the sympathetic and parasympathetic nervous systems, with sympathetic innervation leading to vasoconstriction and parasympathetic innervation leading to vasodilation. Humoral mechanisms describe a response to the presence of hyperosmolar contents within the lumen of the gastrointestinal tract and stimulate release of nitric oxide to cause vasodilation and aid in digestion. During exercise, a greater degree of cardiac output is required to supply oxygenated blood to the coronary arteries and the skeletal muscles. Action of the sympathetic nervous system leads to increased cardiac output via augmentation of the heart rate, increased splanchnic vascular resistance, and increased pulmonary, skeletal muscle, coronary, and cutaneous blood flow. Incorrect Answers: A, B, C, D, and F. Coronary (Choice A) vascular resistance decreases during intensive exercise leading to increased perfusion of the heart, enabling maintenance

136 ---------- Exam Section 3: Item 36 of 50 National Board of Medical Examiners mont * 36. A 50-year-old man comes to the physician because of a 2-week history of progressive shortness of breath while climbing stairs to his office every morning. He reports no other problems, but he is concerned because his father had a major myocardial infarction at the age of 52 years. His pulse is 110/min and regular, respirations are 16/min, and blood pressure is 135/95 mm Hg. The lungs are clear to auscultation. Cardiac examination shows normal heart sounds with a physiologic split of S2. Stress echocardiography shows hypokinesis of the posterior left ventricle with increasing activity levels. Which of the following is the most likely cause of the posterior left ventricular findings in this patient? A) Disruption of the sympathetic nerves to the left ventricle O B) Extravascular compression of the coronary arteries O C) Increase

E. Stenosis of the right coronary artery (RCA) most likely accounts for this patient's posterior left ventricle hypokinesis on stress echocardiography. The coronary arteries supply oxygenated blood to the myocardium and originate just distal to the aortic valve at the right and left coronary ostia, which give rise to the RCA and left coronary artery (LCA), respectively. The RCA in right-dominant individuals gives rise to several bra branch. In left-dominant individuals, the PDA arises from the left circumflex artery, which is a branch of the LCA. The RCA descends along the right side of the heart in the coronary groove between the right atrium and right ventricle. It wraps around the base of the heart, moving posteriorly and inferiorly. It supplies blood to the right atrium, right ventricle, and the inferior aspect of the heart apex. Stenosis of one or more parts of the coronary arteries is termed coronary artery disease (CAD) and is usually a result of atherosclerotic plaque buildup over many years. Depending on the degree of stenosis, the myocardium may not receive sufficient oxygen to meet its metabolic demands during times of increased work, such as exercise. This mismatch in supply and demand of oxygen results in myocardial ischemia and can cause angina. If myocardial ischemia is severe enough and not reversed, which occurs in cases of plaque rupture, it results in myocardial infarction. Many individuals with CAD will have anginal chest pain when they exercise, but symptoms abate with adequate rest or the use of nitroglycerin, which decreases the myocardial oxygen demand and decreases the supply-demand mismatch. Exercise stress tests, including stress echocardiogram, are employed in an attempt to identify areas of decreased perfusion. In this instance, hypokinesis on echocardiogram indicates a flow-limiting stenos

152 ---------- Exam Section 4: Item 2 of 50 Natjonal, Board pf Medical Ęxaminersment. 2. A 14-year-old boy is brought to the physician's office because of decreased appetite and abdominal pain over the past 3 weeks. His mother says that during the same time he has withdrawn from everyone and sleeps constantly. He describes his symptoms vaguely. Physical examination is normal. During further history-taking, it is most critical for the physician to obtain information about which of the following? O A) Developmental history O B) Family history of affective disorders O C) Orientation to time, place, and person O D) School history E) Suicidal ideation or attempts

E. The physician should conduct a thorough suicide risk assessment including questions about suicidal ideation or attempts in this adolescent patient demonstrating neurovegetative symptoms concerning for depression. Symptoms of major depressive disorder (MDD) typically include 2 or more weeks of depressed mood, anhedonia, guilt or worthlessness, difficulty concentrating, suicidal thoughts, and/or neurovegetative symptoms (decreased energy, sleep disturbance, appetite disturbance). These symptoms disrupt everyday functioning. Compared to adults, children are more likely to present with vague somatic symptoms such as abdominal pain (as in this patient) and irritable mood rather than depressed mood. Suicide is the most serious and acute risk of patients with MDD, so physicians should ask detailed questions about patients' suicidal ideation and recent or remote suicide attempts. Incorrect Answers: A, B, C, and D. Developmental history (Choice A), family history of affective disorders (Choice B), and school history (Choice D) are important parts of a thorough psychiatric history in children but are not as important as doing a thorough suicide risk assessment. Some behavioral disorders can be related to developmental delays (as revealed by a developmental history). Gathering a family history of affective disorders can assist in assessing a patient's risk of developing an affective disorder and guide treatment decisions (eg, choosing an antidepressant that was effective in the patient's mother). Obtaining a school history can inform the physician about the child's intellectual and social functioning and screen for mental disorders such as intellectual disability and attention-deficit/hyperactivity disorder. None of these histories directly influence a patient's acute suicide risk. Orientation to time, place, and person (Choic

126 ---------- Exam Section 3: Item 26 of 50 Natjonal, Board of Medical Examinersment andive Rasic Saance 26. A previously healthy 42-year-old Asian woman is brought to the emergency department because of a 24-hour history of nausea, vomiting, and progressive lethargy. She has smoked 1 pack of cigarettes daily for 25 years and drinks four glasses of wine daily. She uses high-dose acetaminophen daily for headaches. She does not use illicit drugs. She is 155 cm (5 ft 1 in) tall and weighs 50 kg (110 Ib); BMI is 21 kg/m2. She is responsive to painful stimuli. Initial laboratory studies show increased hepatic aminotransferase. Which of the following effects of alcohol most likely contributed to this patient's condition? O A) Decreased generation of N-acetyl-p-benzoquinoneimine O B) Increased glucuronidation O C) Increased hepatic glutathione stores D) Increased sulfation E) Induction of cytochrome P450

E. This patient presents with acetaminophen overdose in the setting of chronic alcohol consumption. When taken at therapeutic doses, acetaminophen is safely metabolized through phase II conjugations, including glucuronidation and sulfation. At high doses, saturation of phase II metabolic pathways leads to excess acetaminophen being metabolized by cytochrome P450-mediated reactions to N-acetyl-p to the induction of cytochrome P450. Chronic alcoholics are therefore at an increased risk of hepatotoxicity when consuming high doses of acetaminophen due to the increased production of NAPQI. There is no definite increased risk of hepatotoxicity in association with either acute alcohol intoxication or in patients with chronic alcohol consumption who take normal therapeutic doses of acetaminophen. enzoquinoneimine (NAPQI), which has strong oxidizing properties and is directly hepatotoxic. Chronic ethanol ingestion leads Incorrect Answers: A, B, C, and D. Decreased generation of N-acetyl-p-benzoquinoneimine (NAPQI) (Choice A) would not be observed in the setting of acetaminophen toxicity. NAPQI is a toxic metabolic byproduct of acetaminophen. Through the induction of cytochrome P450, chronic alcohol ingestion leads to an increased generation of NAPQI. Increased glucuronidation and sulfation (Choices B and D) are observed in the setting of acute acetaminophen overdose due to the large quantity of acetaminophen ingested, but this is not a direct effect of alcohol. Glucuronidation and sulfation reactions convert acetaminophen into harmless metabolites. Once glucuronidation and sulfation pathways are overwhelmed, cytochrome P450 converts excess acetaminophen into the toxic metabolite NAPQI. Ethanol does not directly lead to the increased glucuronidation or sulfation of acetaminophen. Increased hepatic glutathione stores (Choice C) a

128 ---------- Exam Section 3: Item 28 of 50 Natjonal, Board of Medical Examinersent 28. A 32-year-old woman comes to the physician because of a 3-month history of fevers, weakness, and night sweats. Physical examination shows enlarged axillary lymph nodes. A photomicrograph of tissue obtained on biopsy of an affected node is shown. A stain for acid-fast bacilli is negative. Which of the following is most likely involved in the pathogenesis of these findings? O A) Activation of alternate complement pathway O B) Streptococcal infection of the region drained by the lymph node O C) Type I (immediate) hypersensitivity reaction O D) Type II (immune complex-mediated) hypersensitivity reaction E) Type IV (delayed) hypersensitivity reaction

E. Type IV (delayed) hypersensitivity reaction is the correct answer. This patient with fevers, night sweats, and lymphadenopathy with a lymph node biopsy showing noncaseating granulomas likely has sarcoidosis, a disease with myriad manifestations that most commonly involves the lungs, eyes, skin, and central nervous system. It is characterized by granuloma formation in the absence of pathogenic microbes such as fungi or mycobacterium. The noncaseating epithelioid granulomas, which are the hallmark of sarcoidosis, are thought to contain an antigen at their core, with surrounding macrophages that form multinucleated giant cells. The dense lymphocytic inflammatory infiltrate that surrounds these granulomas contains many helper CD4+T lymphocytes, cytotoxic CD8+ T lymphocytes, B lymphocytes, and fibroblasts. Granuloma formation is often a consequence of type IV hypersensitivity reactions, which involve the exposure of a sensitized individual to an antigen, followed 24 to 72 hours later by an immune reaction to the antigen. The response is primarily cellular rather than mediated by antibodies, and the cells involved are macrophages and Th1 helper T cells. Thus, a type IV hypersensitivity reaction is consistent with granuloma formation in sarcoidosis. Incorrect Answers: A, B, C, and D. Activation of alternate complement pathway (Choice A) does not occur in sarcoidosis and has no role in the formation of granulomas. The alternate complement pathway has been implicated in the pathogenesis of atypical hemolytic uremic syndrome, among other diseases such as lupus nephritis. Streptococcal infection of the region drained by the lymph node (Choice B) would not likely present with granulomas on lymph node biopsy. Bacterial infections commonly lead to a reactive lymph node profile with infiltration of the node by neutrophils. Type I

109 ---------- Exam Section 3: Item 9 of 50 National, Board pf Medical Examiners ment 9. In biosynthesis of thyroid hormones, iodide is concentrated by the thyroid gland and oxidized by thyroidal peroxidase to iodine. lodine iodinates amino acid residues within the thyroglobulin molecule. Which of the following amino acids undergoes iodination? O A) Histidine O B) Phenylalanine O C) Proline O D) Tryptophan E) Tyrosine

E. Thyroid hormone is produced in the follicular cells of the thyroid gland. Thyroid-stimulating hormone is produced in the anterior pituitary and signals the production of thyroid hormone by the thyroid. Tyrosine molecules are assembled into protein-containing chains known as thyroglobulin. Concurrently, iodine is transported into the follicular cells and is linked to the tyrosine residues in thyroglobulin by thyroid peroxidase, forming thyroxine (T4) or triiodothyronine (T3). The T4 and T3 molecules are then cleaved from thyroglobulin. These thyroid hormones are released into the bloodstream, where they act upon target tissues in the periphery. Incorrect Answers: A, B, C, and D. Histidine (Choice A) is an amino acid that is a precursor to the biologically important molecule histamine. Histamine is released from mast cells and contributes to vascular permeability. It is key in allergic responses. It also functions in the central nervous system to promote wakefulness and causes release of gastric acid in the stomach. Phenylalanine (Choice B) is an amino acid that can be converted into multiple neurotransmitters. Inability to metabolize phenylalanine due to genetic defects in the enzyme phenylalanine hydroxylase leads to deposition in central nervous system tissues resulting in intellectual disability and seizures. Proline (Choice C) is an amino acid that is important for the formation of collagen. It is essential for maintaining the connective tissue integrity. Vitamin C deficiency prevents hydroxylation of proline, leading to scurvy. Tryptophan (Choice D) is an amino acid that cannot be synthesized in humans, making it an essential amino acid. It is a biochemical precursor for serotonin, melatonin, and niacin. Educational Objective: Thyroid hormone is produced via the iodination of the tyrosine residues within thyrogl

168 ---------- Exam Section 4: Item 18 of 50 National, Board pf Medical Examiners ment 18. A 17-year-old boy with chronic renal insufficiency undergoes a unilateral nephrectomy. A photograph of the resected kidney is shown. Which of the following pathologic processes is most likely present in the kidney? O A) Acute glomerulonephritis O B) Acute tubular necrosis O C) Angiomyolipoma D) Autosomal recessive polycystic kidney disease O E) Fibromuscular dysplasia O F) Hydronephrosis O G) Nephroblastoma SPECIMEN. DATE O H) Papillary necrosis O I) Renal vein thrombosis O J) Staghorn calculus

F. Congenital urinary tract abnormalities include, but are not limited to, unilateral renal agenesis, fused kidneys, ureteral stricture and stenosis, duplex collecting system, posterior urethral valves, bladder agenesis, bladder exstrophy, hypo- or epispadias, and urethral strictures. Regardless of the etiology of malformation, ureteral obstruction and vesicoureteral reflux are common complications. Chronic reflux can lead to hydroureter and hydronephrosis, characterized by dilation of the renal pelvis and calyces resulting in compression atrophy of the renal parenchyma and eventual renal insufficiency and failure. Hydronephrosis can be graded in severity, in which the most severe cases compress the parenchyma resulting in gross distortion of renal architecture. Incorrect Answers: A, B, C, D, E, G, H, I, and J. Acute glomerulonephritis (Choice A) refers to a variety of glomerular diseases, including nephritic and nephrotic syndromes. Nephritic syndromes typically present with acute renal failure associated with hematuria, red blood cell urine casts, and hypertension. Nephrotic syndrome typically presents with excessive proteinuria, hyperlipidemia, hypoalbuminemia, and edema. Diagnosis is usually made with renal biopsy; gross dilation of the renal pelvis and calyces are inconsistent with the diagnosis. Acute tubular necrosis (Choice B) occurs following an ischemic or nephrotoxic insult to the kidneys, which results in necrosis of the tubular epithelium. Granular, muddy brown casts are typical on urinalysis. It would not cause gross distortion of the pelvic and calyceal architecture. Angiomyolipomas (Choice C) are tumors derived from perivascular epithelioid cells. While typically benign, they can be associated with tuberous sclerosis. They present as a mass, which can be composed of smooth muscle, adipocyte, and epithel

200 ---------- Exam Section 4: Item 50 of 50 Natjonal, Board of Medical Examinersment 50. A 55-year-old man comes to the office because of a 7-year history of red, dry, itchy skin over his back and buttocks. Use of emollients and topical corticosteroids has not improved his symptoms. He has no other history of major medical illness. His vital signs are within normal limits. Physical examination shows the findings in the photograph. Laboratory studies show eosinophilia and increased serum concentrations of IgA and IgE. Malignant transformation of which of the following cell types is the most likely cause of the findings in this patient? O A) Blastocytes O B) Eosinophils о С) Масrophages D) Mastoid cells O E) Monocytes F) T lymphocytes

F. The early stage of mycosis fungoides, the most common form of cutaneous T-cell lymphoma, is characterized by erythematous, wrinkled, scaly patches. If the disease remains untreated, the patches will thicken into plaques and then tumors. The patches classically involve areas protected from the sun such as the axillae and buttocks. As skin biopsy can be inconclusive early in the disease, it often takes several years for a diagnosis of mycosis fungoides to be made. It is an indolent lymphoma and has a good prognosis. It is caused by the infiltration of malignant T lymphocytes into the epidermis. Histopathologic evaluation with immunohistochemical staining for T lymphocyte specific markers is critical to the diagnosis. Incorrect Answers: A, B, C, D, and E. Blastocytes (Choice A) are undifferentiated cells present early in the first stages of embryonic development. They do not play a role in the pathogenesis of mycosis fungoides. Eosinophils (Choice B) may be increased in primary hypereosinophilia, cutaneous eosinophilic vasculitis (eg, eosinophilic granulomatosis with polyangiitis), and multiple causes of eosinophilic dermatosis (eg, urticaria, drug eruptions, eczema, atopic dermatitis). They are not the causative cell type in mycosis fungoides. Macrophages (Choice C) and monocytes (Choice E) are cells of the innate immune system that serve to phagocytose foreign proteins or organisms and also serve as antigen-presenting cells to T-lymphocytes. Macrophages that reside in the skin are called Langerhans cells; malignant transformation of Langerhans cells causes Langerhans cell histiocytosis, not mycosis fungoides. Mastoid cells (Choice D) describe the air-lined spaces contained within the mastoid region of the temporal bone and are not involved in skin rashes. A similarly named cell, the mast cell, is increased in mastocy

67 ---------- Exam Section 2: Item 17 of 50 Natjonal, Board of Medical Examinersment 17. To investigate the association between cellular telephone use and the development of brain tumors, 782 patients with brain tumors are compared with 799 age- and sex-matched patients who do not have a malignancy. The results of the study find that there was no increase in the odds ratio for cellular telephone use in patients who had brain cancer compared with those who did not have cancer. Which of the following best describes this study design? A) Case-control study O B) Case series O C) Cross-sectional study O D) Prospective cohort study O E) Randomized clinical trial O F) Retrospective cohort study

A. A case-control study investigates an association between exposure and an outcome. In this study design, a group of patients with the disease (cases) are identified. A group of patients without the disease (controls) are matched on baseline characteristics to the cases. Exposure data for the two groups is collected, and these data are compared to determine association with the outcome (disease) in question. An odds ratio may be calculated to compare exposures between groups. Incorrect Answers: B, C, D, E, and F. A case series (Choice B) is a descriptive study design in which a number of consecutive or nonconsecutive cases of a disease and/or treatment are described in detail, with information about exposure, demographics, and comorbidities. Case series do not imply a cause-and-effect relationship. They do not test a hypothesis nor are they randomized. They are useful in characterizing the natural history of a disease or response to treatment. They are also useful in describing rare diseases, as small amounts of patients may not permit conduction of larger case-control, cohort, or randomized trials with sufficient power. A cross-sectional study (Choice C) seeks to identify the prevalence of the condition at a particular point in time. An example of a cross-sectional study would be a single survey of a population inquiring whether patients have coronary artery disease and concurrently inquiring about activity levels and diet. Thus, the risk factor and the outcomes are measured simultaneously. The study does not follow patients over time. All information is collected at a single time point. A prospective cohort study (Choice D) identifies a group of patients and follows them over time seeking to identify whether an exposure is associated with an outcome of interest. In a prospective design, the hypothesis and analysis p

41 ---------- Exam Section 1: Item 42 of 50 National, Board of Medical Examiners Comprehensive Basic Science Self-Assessment 42. A previously healthy 20-year-old woman comes to the emergency department because of a 3-day history of fever, shaking chills, headaches, fatigue, and joint and muscle pain. She recently spent the summer working as a lifeguard on Long Island, New York. She has never traveled outside the USA. She underwent splenectomy for injuries sustained in a motor vehicle collision at the age of 6 years. Her temperature is 39.1°C (102.4°F). Physical examination shows no other abnormalities. A peripheral blood smear shows small intraerythrocytic rings; the result of a polymerase chain reaction test for Plasmodium species is negative. Which of the following is the most likely causal organism? A) Babesia microti O B) Bartonella bacilliformis C) Bordetella bronchiseptica D) Borrelia burgdorferi E) Brucella

A. Babesiosis is a disease presenting similar to malaria that is caused by the protozoa Babesia microti. It is predominantly found in the northeastern United States and transmitted via the Ixodes tick. It classically presents with fever, chills, myalgias, weakness, headache, and an associated hemolytic anemia. Asplenia is a risk factor for severe disease. Diagnosis is made by direct visualization of organisms on a Giemsa-stained peripheral blood smear, which may appear in ring forms or in a tetrad formation resembling a cross within erythrocytes. Treatment is with atovaquone plus azithromycin. Incorrect Answers: B, C, D, and E. Bartonella bacilliformis (Choice B) is a Gram-negative, coccobacillus found in Peru, Ecuador, Colombia, and some parts of southern Florida that causes bartonellosis (Carrión disease) or Oroya fever in the acute phase of infection and verruga peruana in the chronic phase. It is transmitted by sandflies. Bordetella bronchiseptica (Choice C) is a Gram-negative bacillus and is a common cause of respiratory disease in dogs and cats, but rarely causes disease in humans. Borrelia burgdorferi (Choice D) is a spirochete common in the northeastern United States that causes Lyme disease. It is transmitted by the Ixodes tick. Babesia microti may be co-transmitted with Borrelia burgdorferi in this manner. Brucella melitensis (Choice E) is one of the species of Gram-negative coccobacilli that cause brucellosis, also known as undulant fever. The other species that infect humans are B. abortus, B. canis, and B. suis. Brucellosis is commonly contracted by the consumption of unpasteurized milk. Educational Objective: Babesiosis is a malaria-like disease caused by the protozoa Babesia microti. It can be recognized by the presence of intraerythrocytic ring forms or a tetrad formation on a peripheral blood smear.

91 ---------- Exam Section 2: Item 41 of 50 National Board of Medical Examiners andive Rasic Science ment * 41. A 75-year-old woman with non-small cell carcinoma of the lung is brought to the physician because of a 2-day history of lethargy and cognitive impairment. One month ago, she completed a 1-year regimen of chemotherapy and cranial irradiation. One week ago, she received the diagnosis of major depressive disorder and began therapy with a selective serotonin reuptake inhibitor. She speaks with a blunted affect. Physical examination shows no abnormalities. Her Mini-Mental State Examination score is 20/30. Serum studies show a sodium concentration of 122 mEg/L. The patient appears confused and falls asleep during the examination. Which of the following is the most likely diagnosis? O A) Delirium O B) Dementia, Alzheimer type O C) Dementia caused by Pick disease O D) Serotonin syndrome O E) Vascular dementia

A. Delirium is an acute confusional state typically associated with acute medical illness in older patients. The pathogenesis of delirium is poorly understood but may be related to deficient acetylcholine signaling in the brain. Delirium presents with acute disturbances in awareness, attention, and baseline cognition that fluctuate in severity over the course of the day and appear related to a substance or acute medical condition. The cognitive changes can include memory or language deficits, disorientation, or perceptual disturbances (eg, visual hallucinations or paranoia). This patient demonstrates clear disturbances in awareness and cognition, as well as severe hyponatremia (likely due to syndrome of inappropriate antidiuretic hormone secretion associated with brain irradiation and selective serotonin reuptake inhibitor initiation). Delirium prevention with cognitive stimulation and avoidance of cognition-impairing medications is key. Delirium is managed by addressing the underlying medical problem and supportive management such as frequent reorientation and promoting sleep only at night to re-establish the appropriate circadian rhythm. Antipsychotic medication can manage symptoms of agitation or distressing hallucinations but does not shorten the duration of delirium. Incorrect Answers: B, C, D, and E. Dementia, Alzheimer type (Choice B) or caused by Pick disease (Choice C) would present with insidious cognitive decline instead of acute confusion. Fluctuations in the severity of cognitive deficits over the course of the day would be atypical, unless a concomitant delirium was also present. Baseline dementia is a risk factor for developing acute delirium. Pick disease (also known as frontotemporal dementia) typically develops when patients are in their 50s and features personality changes in addition to cognitive de

39 ---------- Exam Section 1: Item 40 of 50 National Board of Medical Examiners Comprehensive Basic Science Self-Assessment 40. A 52-year-old woman undergoes surgical excision of a large multinodular goiter. During the operation, the superior thyroid artery is used as a landmark to avoid damage to a nerve proximal to that vessel. Which of the following best describes this nerve? A) External branch of the superior laryngeal nerve B) Inferior root of the ansa cervicalis C) Nerve to the mylohyoid D) Phrenic nerve E) Transverse cervical nerve

A. During thyroid surgery, there are a number of imp laryngeal nerve arises from the vagus nerve and courses along the superior thyroid artery, supplying innervation to the cricothyroid muscles. Injury to this nerve will cause alterations in the pitch of the voice. The recurrent laryngeal nerve is also important for vocalization, as it provides motor innervation for all of the intrinsic muscles of the larynx except for the cricothyroid muscle. It provides the sensory innervation for the laryngeal structures inferior to the vocal cords. The recurrent laryngeal nerves additionally have unique anatomy as they branch from the vagus nerves distally in the thorax. The left recurrent laryngeal nerve loops inferior and posterior to the aortic arch, while the right recurrent laryngeal nerve loops inferior and posterior to the right subclavian artery. Both of these nerves then progress cranially, rather than caudally. ant structures at risk. These structures include the parathyroid glands, the superior and inferior thyroid arteries, the recurrent laryngeal nerve, and the external branch of the superior laryngeal nerve. The superior Incorrect Answers: B, C, D, and E. The ansa cervicalis (Choice B) is a loop of nerves in the anterior neck that originate from the spinal nerve roots of C1-C3. The inferior root is formed by the fibers of C2 and C3 and provides branches to the inferior portion of the omohyoid, sternothyroid, and sternohyoid muscles. The nerve to the mylohyoid (Choice C) arises from the mandibular division of the trigeminal nerve and supplies motor innervation for the mylohyoid muscle and the anterior belly of the digastric muscle. The phrenic nerve (Choice D) arises from the cervical roots C3-C5 and provides motor innervation to the diaphragm. It also provides sensory innervation to the mediastinal pleura and the peri

65 ---------- Exam Section 2: Item 15 of 50 Natjonal, Board of Medical Examinersment ancive Rasic Saance 15. Histone acetyltransferases catalyze the acetylation of lysine residues in the amino-terminal tails of histones. Which of the following is the most likely effect of this covalent modification on chromatin structure? O A) Decreases the affinity of histones for DNA O B) Decreases the nucleosome content of the nucleus O C) Increases the affinity of histones for DNA O D) Increases the nucleosome content of the nucleus O E) Removes histone H1 from DNA O F) Removes histone H4 from DNA

A. Histone acetyltransferases decrease the affinity of histones for DNA by adding acetyl groups to histones lysine residues. Histones are proteinaceous cores, rich in lysine and arginine (positively charged amino acids). Without modification (eg, acetylation), the positively charged histones attract negatively charged DŇA, which loops around it, forming a chromatin nucleosome. Condensed heterochromatin is transcriptionally inactive. Acetylation of a histone is important in gene transcription as it permits relaxation of a DNA-histone complex by reducing the molecular affinity between DNA and histone. When DNA is tightly bound to a histone, gene transcription does not occur, since binding sites for RNA polymerase |I (which transcribes messenger RNA) will be occupied by the presence of the histone. When the affinity between the histone and DNA is reduced, the unpacked (euchromatin) DNA can be bound by transcriptional enzymes, promoting gene expression. Incorrect Answers: B, C, D, E, and F. Decreasing the nucleosome content of the nucleus (Choice B) implies dissolution of the nucleosome structure, a discrete length of DNA (usually 126 base pairs, or nearly two turns of DNA) that is wrapped around a set of eight histones (H2A, H2B, H3, and H4) forming a histone octamer. During transcription or DNA replication for cell division, the nucleosome is transiently disassembled, but then rapidly reassembled. Conversely, increasing the nucleosome content of the nucleus (Choice D) implies the formation of more nucleosomes. Adding additional nucleosomes to existing DNA would indicate winding the chromatin tighter, but the tight regulation of DNA folding prevents this from occurring. The addition of an extra chromosome, however, such as in trisomy 21, would technically increase the nucleosome content of the nucleus. Increasing the aff

77 ---------- Exam Section 2: Item 27 of 50 National Board of Medical Examiners nent V 27. A 52-year-old man who is in the hospital with malignant hypertension, heart failure, and a mild renal impairment is being treated with captopril, nitroprusside, and furosemide. After 48 hours of therapy, his blood pressure is controlled, but he develops metabolic acidosis, lethargy, and difficulty breathing. Which of the following substances most likely caused these adverse effects in this patient? A) Cyanide O B) Iron O C) Lead O D) Nitric oxide E) Selenium

A. Sodium nitroprusside is an intravenous, titratable vasodilator that can be used for the treatment of hypertensive emergencies. It breaks down in circulation to release nitric oxide, which in turn activates guanylate cyclase in vascular smooth muscle to result in vascular smooth muscle relaxation and vasodilation via a cyclic guanosine monophosphate (CGMP) pathway. A byproduct of sodium nitroprusside is cyanide, and in rare cases, infusion of nitroprusside can cause cyanide toxicity. Cyanide poisoning occurs due to its action of inhibiting the electron transport chain, leading to metabolic lactic acidosis. Cyanide inhibits the mitochondrial enzyme cytochrome c oxidase, preventing the synthesis of adenosine triphosphate, leading to lactic acidosis. The presentation of cyanide poisoning is characterized by tachypnea due to compensatory respiratory drive, hypotension, confusion, lethargy, and bradycardia. The management of cyanide poisoning includes supportive care with intravenous fluids and vasopressors, sodium thiosulfate which increases the conversion of cyanide to thiocyanate for renal excretion, amyl nitrites to induce methemoglobinemia which binds with cyanide and prevents it from acting on the electron transport chain, and hydroxocobalamin which combines with cyanide to create nontoxic cyanocobalamin. Incorrect Answers: B, C, D, and E. Iron toxicity (Choice B) is characterized by gastrointestinal irritation, and later, anion gap metabolic acidosis, hepatic toxicity, acute kidney injury, coagulopathy, and fulminant hepatic failure. Due to the severity of toxicity, early management may include whole bowel irrigation with polyethylene glycol or gastric lavage. Lead toxicity (Choice C) can occur for example from workplace exposure, lead paint in a residence, or contaminated drinking water. Symptoms of lead exposure

6 ---------- Exam Section 1: Item 7 of 50 National Board of Medical Examiners Comprehensive Basic Science Self-Assessment 7. A 65-year-old man who has chronic lymphocytic leukemia has the sudden onset of fatigue and shortness of breath. Laboratory studies show: Hemoglobin Hematocrit Leukocyte count Platelet count 6.5 g/dL 19% 50,000/mm3 170,000/mm3 8% Reticulocyte count Serum bilirubin 4 mg/dL 0.5 mg/dL Total Conjugated A peripheral blood smear is shown. Which of the following is the most likely cause of the anemia? A) Aplastic anemia B) Autoimmune hemolysis C) Hereditary spherocytosis D) Microangiopathic hemolysis E}Monoclonal gammopathy

B. Autoimmune hemolysis explains this patient's anemia, unconjugated hyperbilirubinemia, reticulocytosis (large blue-colored erythrocytes), and spherocytes on the peripheral smear. Autoimmune hemolytic anemia (AIHA) occurs due to production of antibodies targeting circulating red blood cell (RBC) surface antigens, marking them for removal in the reticuloendothelial system (RES) or fixing complement leading to intravascular hemolysis. IgG antibodies are active at the physiologic temperature of the human body (called "warm AIHA"); these do not routinely activate complement. Instead, they mark the cells for removal. Phagocytosis of the IgG antibody along with a component of the erythrocyte membrane creates sphere-shaped RBCS seen on peripheral smear. In contrast, IgM antibodies bind and fix complement at lower temperatures and result in intravascular hemolysis (called "cold AIHA"). IgG antibodies are known as "warm" antibodies, and IgM antibodies are known as "cold" antibodies. Regardless of the type of antibody, laboratory findings demonstrate a normocytic anemia, an increased reticulocyte count indicating hematopoiesis, and unconjugated hyperbilirubinemia from hemoglobin released from lysed erythrocytes. Diagnosis is made by laboratory analysis, a compatible clinical history, and positive direct antibody test (DAT, Coombs). In the DAT, the patient's erythrocytes are washed free of plasma, and incubated with Coombs reagent, an anti-lgG and anti-complement antibody. If an autoantibody or complement is bound to the surface of the erythrocytes, the Coombs reagent will bind to it and cause agglutination, a positive test. Chronic lymphocytic leukemia (CLL) predisposes to the development of warm AlHA because malignant CLL cells produce autoantibodies. Incorrect Answers: A, C, D, and E. Aplastic anemia (Choice A) results from b

1 ---------- Exam Section 1: Item 2 of 50 National, Board of Medical Examiners Comprehensive Basic Science Self-Assessment 2. Free purine and pyrimidine bases are reutilized in normal metabolism. In children with Lesch-Nyhan syndrome who have intellectual disability, poor muscle coordination, and self-mutilation tendencies, there is a defect in the salvage of which of the following pairs of bases? A) Adenine and thymine B) Guanine and hypoxanthine C) Guanine and uric acid D) Uracil and cytosine E) Xanthine and hypoxanthine

B. Lesch-Nyhan syndrome presents with intellectual disability, aggressive behavior, self-mutilation, gout, and dystonia. The disorder is due to inactivating mutations of hypoxanthine-guanine phosphoribosyltransferase (HGPRT), a key enzyme in the purine salvage pathway, and is inherited in an X-linked recessive fashion. HGPRT catalyzes the conversion of guanine to guanosine monophosphate and hypoxanthine to inosine monophosphate. Patients with deficient activity of HGPRT are unable to salvage guanine and hypoxanthine and develop resultant increased levels of xanthine and uric acid. Hyperuricemia in Lesch-Nyhan syndrome is treated with xanthine oxidase inhibitors, such as allopurinol or febuxostat, in order to reduce the synthesis of uric acid. Incorrect Answers: A, C, D and E. Adenine and thymine (Choice A) are purine and pyrimidine bases, respectively. Purine and pyrimidine salvage are handled through two distinct pathways that are not commonly involved in a single disease process. Guanine and uric acid (Choice C) accumulation may occur as part of Lesch-Nyhan syndrome, however, the accumulation of uric acid is also secondary to accumulation of hypoxanthine. Choice B more accurately describes defective salvage of guanine and hypoxanthine as the fundamental effect of HGPRT dysfunction. The accumulation of uric acid is secondary. Uracil and cytosine (Choice D) are pyrimidine nucleotides. Pyrimidine salvage is not affected by mutations of HGPRT. Defects of xanthine and hypoxanthine (Choice E) metabolism may result from defects in HGPRT. However, HGPRT dysfunction results in impaired hypoxanthine salvage with resultant excessive production of xanthine, rather than impaired xanthine salvage. Educational Objective: Lesch-Nyhan syndrome presents with intellectual disability, aggressive behavior, self-mutilation, gout, and dyst

70 ---------- ivA Medical Examinersment Exam Section 2: Item 20 of 50 National, Board 20. A 55-year-old woman comes to the physician because of a 3-year history of weakness, fatigue, decreased appetite, and constipation. She says that she did not receive medical treatment for these symptoms because she did not have health insurance until recently. She was admitted to the hospital 1 year ago because of renal calculi, but she could not afford to pay for follow-up care. X-rays at that time were indicative of osteitis fibrosa cystica. Vital signs are within normal limits. Physical examination shows bony tenderness and muscle strength of 4/5 with decreased tone. Serum studies show an increased calcium concentration and a decreased phosphorus concentration. Which of the following is the most likely diagnosis? O A) Adult T-lymphocyte leukemia B) Chief cell adenoma of the parathyroid gland O C) Familial (benign) hypocalciuri

B. Primary hyperparathyroidism results in hypercalcemia and hypophosphatemia secondary to the aberrant increased production of parathyroid hormone (PTH). The most common cause of primary hyperparathyroidism is a chief cell adenoma of the parathyroid gland. PTH is produced by parathyroid chief cells and stimulates 1a-hydro phosphate. PTH also plays a role in the regulation of calcium and phosphate by stimulating osteoclastic bone reabsorption and distal convoluted tubular calcium reabsorption and phosphate excretion in the kidney. Common symptoms of primary hyperparathyroidism are secondary to hypercalcemia and include recurrent nephrolithiasis, bone pain from osseous resorption (osteitis fibrosa cystica), polyuria, dehydration, constipation, and psychiatric disturbances. kylase in the kidn which leads to the conversion of 25-hydroxycholecalciferol to 1,25-dihydroxycholecalciferol (active vitamin D). Vitamin Ď acts to increase the intestinal sorpt of calcium and Incorrect Answers: A, C, D, and E. Adult T-lymphocyte leukemia (Choice A) is a rare neoplasm of T lymphocytes that results as a sequela of infection with human T-lymphotropic virus type 1 (HTLV-1). While this disease produces lytic bone lesions and hypercalcemia, it is also accompanied by other findings including generalized lymphadenopathy, hepatosplenomegaly, and immunodeficiency. Familial (benign) hypocalciuric hypercalcemia (Choice C) arises from an abnormal calcium sensing receptor on parathyroid cells. As a result, PTH is not suppressed by normal or increased serum calcium. This leads to a mild hypercalcemia and decrease urine calcium concentration in the setting of increased PTH due to the normal mechanism of action of PTH on osteoclasts and renal tubules. Symptomatic hypercalcemia and renal calculi are uncommon in this condition. Medullary carcinoma of

95 ---------- Exam Section 2: Item 45 of 50 Natjonal, Board of Medical Examinersment andive Rasic Science * 45. A 10-year-old boy with spinal muscular atrophy is brought to the emergency department because of a 2-week history of nausea and vomiting. He also has had decreased appetite during this period. One week prior to his illness, his entire family had similar symptoms which improved after a few days. Physical examination shows decreased skin turgor, dry mucous membranes, and diffuse muscle weakness and atrophy. Laboratory studies show: Serum 140 mEq/L (N=138- 145) 4 mEq/L (N=3.4-4.7) 105 mEq/L (N=95- 105) 18 mEq/L (N=22-28) 18 mg/dL (N=5-18) 30 mg/dL (N=60-100) Na+ K+ CI- HCO3- Urea nitrogen Glucose Urine 5.0 (N=4.6-8.0) 1.020 (N=1.003- 1.030) pH Specific gravity Glucose none Protein trace Ketones 4+ RBC O/hpf O/hpf WBC Which of the following biochemical cycles in the liver is most likely providing this patient's

B. Spinal muscular atrophy is a group of autosomal recessive disorders that lead to the degeneration of motor neurons and result in progressive hypotonia and areflexia. Patients may exhibit difficulty with mobility and frequently experience respiratory distress and difficulty feeding. The brain primarily utilizes glucose for energy, and during the fasting state is highly dependent on glycogenolysis and hepatic gluconeogenesis. In the initial fasting state, glycogen is broken down in the liver to assist in the maintenance of blood glucose levels. Glycogen stores are typically depleted after 1 day of fasting. Following this, blood glucose is maintained through the breakdown of muscle and fat to provide substrates for hepatic gluconeogenesis. After approximately 3 days of fasting, fatty acids become the primary source for energy through the production of ketone bodies. This patient's laboratory values, including profound hypoglycemia, decreased bicarbonate, and ketonuria, are suggestive of starvation ketoacidosis in the setting of acute illness and prolonged fasting from persistent nausea and vomiting. Carnitine and acylcarnitine are components of the carnitine shuttle and serve to transport acyl-CoA into the mitochondrial matrix to allow for the beta-oxidation of fatty acids. During the initial step of the shuttle, carnitine palmitoyltransferase I transfers the acyl group from fatty acyl-CoA to carnitine, forming acylcarnitine. Acylcarnitine is then transported across the inner mitochondrial membrane by carnitine-acylcarnitine translocase, a step which is susceptible to negative feedback by increased malonyl-CoA. In the final step, carnitine and fatty acyl-CoA are liberated by carnitine palmitoyltransferase I| on the inner mitochondrial membrane. This allows beta-oxidation to proceed within the mitochondrial matrix, whic

37 ---------- Exam Section 1: Item 38 of 50 National Board of Medical Examiners Comprehensive Basic Science Self-Assessment 38. A 55-year-old woman who is a physician is admitted to the hospital because of a fractured femur sustained in a motor vehicle collision. Two days after admission, she develops tachycardia, restlessness, diaphoresis, and anxiety. She says that "vague shapes" are coming out of the walls. Which of the following is the most likely cause of this patient's current condition? A) Acute stress disorder B) Alcohol withdrawal C) Bipolar disorder D) Histrionic personality disorder E) Panic disorder

B. This patient is likely developing delirium tremens, a serious complication of alcohol withdrawal. Alcohol increases the central nervous system activity of gamma-aminobutyric acid (GABA), the main inhibitory neurotransmitter. Abrupt discontinuation of alcohol therefore leads to sympathetic excess. Uncomplicated alcohol withdrawal typically presents with tremors, anxiety, restlessness, headache, nausea, and diaphoresis, which can occur hours after the cessation of alcohol use. Delirium tremens, the most concerning and life-threatening complication of alcohol withdrawal, involves severe confusion and disorientation, fluctuations in consciousness, agitation, visual hallucinations, and autonomic instability (fluctuations in heart rate and blood pressure with hyperthermia). As in this patient, delirium tremens can begin 2 to 5 days after the patient's last alcoholic drink. Patients with delirium tremens may die from associated dysrhythmias, seizures, or respiratory failure. Delirium tremens is managed supportively with benzodiazepines to address agitation and prevent the symptoms of withdrawal, fluid repletion, nutritional supplementation, and frequent assessment, including vital sign checks. Other complications of alcohol withdrawal include alcoholic hallucinosis (auditory or visual hallucinations without confusion or autonomic instability). Incorrect Answers: A, C, D, and E. Acute stress disorder (Choice A) refers to symptoms of intrusive thoughts, hyperarousal, dissociation, avoidance, and/or depressed mood within 1 month of a traumatic event such as a life-threatening motor vehicle collision. This patient's tachycardia and diaphoresis could represent symptoms of hyperarousal, and hallucinations do occasionally occur during trauma-related dissociative experiences. However, hallucinations occur less commonly in acute st

85 ---------- Exam Section 2: Item 35 of 50 Natjonal, Board of Medical Examinersment ansive Rasic Saance 35. A study is conducted to assess 32 patients in a community of 1000 who have developed drug-resistant tuberculosis during a 1-year period. These patients are removed from the community for treatment. Assuming that the risk for infection and susceptibility to the disease is constant, which of the following best represents the number of individuals most likely to develop subsequent drug-resistant tuberculosis during the next year? O A) 27 O B) 29 C) 31 O D) 32 E) 33

C. The incidence rate of disease in a population equals the number of new cases of the disease divided by the number of persons at risk for contracting the disease. Incidence is a measure of risk and is the probability of a given condition occurring within a specified period of time. It is often expressed as a single number of new cases but is more appropriately expressed as a fraction of a population. Importantly, when individuals are added to or are removed from a population, the denominator must also change to reflect the new number of persons at risk for contracting the condition. The incidence rate is mathematically expressed as (the number of new cases) / (the number of persons at risk). In this study, 32 patients contracted drug-resistant tuberculosis over a 1-year period of time. 1,000 persons were at risk over that same period of time. The incidence rate of contracting drug-resistant tuberculosis is therefore 3.2% (32/1000). In the following year, once the 32 affected persons were removed from the community, with risk of infection and susceptibility remaining constant, 968 persons would be at risk (1000-32). The number of individuals likely to develop drug-resistant tuberculosis in the following year is therefore computed by: Incidence rate = number of new cases/number of persons at risk = 3.2% = number of new cases/968 individuals =3.2%*968 individuals = 30.97 individuals, or approximately 31 individuals. Incorrect Answers: A, B, D, and E. 27 (Choice A) and 29 (Choice B) would be potentially correct if a larger number of persons than 32 were removed from the population at risk. 32 (Choice D) reflects the number of persons who would be expected to contract drug-resistant tuberculosis if the population at-risk remained 1,000 individuals. The number of persons added to or removed from a population must be consid

11 ---------- Exam Section 1: Item 12 of 50 National, Board of Medical Examiners Comprehensive Basic Science Self-Assessment 12. A previously healthy 35-year-old woman develops hypoxemia 35 minutes after ingesting a near-lethal dose of barbiturates. She has not aspirated. Which of the following sets of arterial blood gas values (in mm Hg) is most likely in this patient? Po2 Pco2 (A-a)02 A) 40 50 35 B) 40 60 40 C) 50 25 10 D) 50 80 10 E) 60 35 25

D. Barbiturates are central nervous system depressants which act on the chloride pore of the GABA receptor, increasing the time the receptor is opened. Symptoms of overdose include encephalopathy and respiratory depression. Respiratory depression leads to hypoventilation as a decreased volume of air is moved to the alveolar-capillary interface for gas exchange. Hypoventilation can be identified by decreased PO2, increased CO-2, and a normal alveolar-arterial (A-a) gradient on arterial blood gas analysis. The alveolar-arterial (A-a) gradient, also written as (A-a)O2, is the difference between the partial pressure of oxygen in the alveoli (PAO 2) and the arterial blood (PaO2). The PaO2 is measured directly on the blood gas analysis. The PAO2 must be estimated using the alveolar gas equation, which is a function of the fraction of inspired oxygen (FiO2), the partial pressure of carbon dioxide (PaCO2), and the respiratory quotient (RQ), which relates the volume of CO2 produced by tissue to the volume of O2 consumed. If there is no disruption of the alveolar-capillary interface or impaired perfusion to ventilated lung, the PAO2 and PaO2 will equalize as oxygen diffuses readily across the membrane. A normal A-a gradient of~10 mm Hg exists based on physiologic shunt, which is a volume of blood that does not interact with ventilated lung. This occurs due to a normal ventilation-perfusion mismatch that exists due to gravity (increased ventilation in the apices of the lung, increased perfusion in the bases) as well as circulation that bypasses the alveolar-capillary interface entirely (eg, bronchial arteries to pulmonary veins, smallest cardiac veins that drain directly into the left ventricle). This deoxygenated blood mixes with freshly oxygenated blood returning from the lungs, leading to a decreased PaO2 relative to the PAO¬

29 ---------- Exam Section 1: Item 30 of 50 National, Board of Medical Examiners Comprehensive Basic Science Self-Assessment 30. A 55-year-old woman with systemic lupus erythematosus has an increased serum C-reactive protein (CRP) concentration. Which of the following is the most likely source of the increased serum CRP concentration in this patient? O A) Bone marrow O B) Brain C) Kidney D) Liver E) Lymph node F) Skin

D. C-reactive protein (CRP) is an acute phase reactant protein that is synthesized in the liver in response to the pro-inflammatory cytokine interleukin-6. It forms an important element of the innate immune response. CRP functions as an opsonin and aids in the targeting of cells for phagocytosis and in the fixation of complement. Clinically, serum measurement of CRP is used as an indicator of active inflammation. Serum CRP is a non-specific inflammatory marker and can indicate inflammation due to infectious, rheumatologic, or malignant etiologies. Incorrect Answers: A, B, C, E, and F. Bone marrow (Choice A) is the site of hematopoiesis and of the production of immune cells. B lymphocyte maturation occurs in the bone marrow. CRP is not synthesized in the bone marrow. Brain (Choice B) is an immune-privileged site and does not play a role in the synthesis of acute phase reactants. Kidney (Choice C) is the site of production of 1,25-dihydroxycholecalciferol, or vitamin D. There is evidence that vitamin D plays a role in regulation of the immune response. CRP is not synthesized in the kidney. Lymph nodes (Choice E) are the site of mature B and T cells and function as the site of antigen presentation and activation of the acquired immune response. ) are Skin (Choice F) has a principal function in immunity, in addition to thermoregulation and fluid balance. Antigen-presenting cells within skin identify antigens and present them to other immune cells, which may produce a host of cytokines including interleukin-6, which triggers the synthesis of CRP by the liver. Educational Objective: CRP is an acute phase reactant protein that is synthesized in the liver. CRP functions as an opsonin and aids in the targeting of cells for phagocytosis and in the fixation of complement. Serum CRP serves as a non-specific inflammatory marker and

60 ---------- Exam Section 2: Item 10 of 50 National, Board pf Medical Examiners ment V 10. A 34-year-old man has a herniated lumbar intervertebral disc. Laminectomy and removal of the herniated fragment are scheduled. Which of the following labeled sites on the normal lumbar axial CT scan shown is the most appropriate location for surgical entrance to the neural canal? FRONT A C O A) O B) D) E)

D. Disc herniations occur when the nucleus pulposus herniates through the outer fibrocartilaginous ring of the intervertebral disc, the annulus fibrosus. Disc herniations can occur from acute trauma, but often have an underlying degenerative component. Degeneration of intervertebral discs is characterized by decreased water content and flexibility of the spinal column. When the nucleus pulposus herniates through the annulus, it can cause direct compression of neural elements as well as irritation, cytokine release, and inflammatory cell infiltration in the area of the affected nerve root. Often, acute disc herniation occurs laterally in the spinal canal due to the integrity of the posterior longitudinal ligament. This can cause acute radiculopathy, which classically presents with pain in a dermatomal distribution, decreased sensation, and weakness in the corresponding myotome. Typically, the affected nerve root is that which exits the spinal canal one level below the disc herniation, because the nerve root is compressed as it traverses distally prior to exiting the neural foramen. Compression of the distal aspect of the cord, known as conus medullaris syndrome, will present with bilateral upper motor neuron type (eg, hyperreflexia, spastic paresis) lower extremity weakness, perineal numbness, and urinary retention leading to overflow incontinence. Cauda equina syndrome is the compression of the bundle of nerve roots that dwell in the spinal canal distal to the conus medullaris. Compression of the cauda equina can cause similar symptoms to conus medullaris syndrome, however, deficits are more likely to be asymmetric as the bilateral roots are seldom compressed in identical fashion, and it typically results in lower motor neuron type weakness (eg, hyporeflexia). Along with anti-inflammatory medications, surgical decompre

84 ---------- Exam Section 2: Item 34 of 50 National Board of Medical Examiners ement 34. A 63-year-old woman comes to the physician because of a 5-day history of shortness of breath and swollen legs. Her respirations are 25/min and labored, and blood pressure is 130/50 mm Hg. She has a large subclavian arteriovenous fistula caused by a stab wound to the left supraclavicular area 15 years ago. Physical examination shows 2+ edema of the lower extremities. Which of the following findings is most likely in this patient? O A) Decreased arterial oxygen saturation O B) Decreased mixed venous oxygen saturation O C) Decreased stroke volume D) Increased resting cardiac output E) Increased systemic vascular resistance

D. Increased resting cardiac output is an expected finding in this patient with clinical signs and symptoms of high-output heart failure in the setting of a large subclavian arteriovenous fistula. While most cases of heart failure are divided into heart failure with preserved (HFPEF) or reduced (HFREF) ejection fraction, high-output cardiac failure is a unique variety of cardiac failure. Causes of high-output cardiac failure include endocrine abnormalities such as hyperthyroidism, chronic lung disease, sepsis, cirrhosis, and large arteriovenous (AV) fistulas as seen in this patient. AV fistulas are either congenital or acquired and result in an abnormal connection between a high-pressure system (arterial) and a low-pressure system (venous), without the benefit of the stepwise reduction in pressure that occurs with blood transmission through the peripheral arterioles and capillaries. AV fistulas effectively bypass the arteriolar and capillary systems leading to significant shunting of blood directly from the arterial to the venous system. This results in increased venous return to the right side of the heart, with a subsequent increase in heart rate and stroke volume. As cardiac output (CO) is defined by the product of stroke volume and heart rate, this results in increased CO. Eventually, due to the persistent increased myocardial oxygen demand of the heart in the setting of high-output, patients will decompensate and present with signs and symptoms of heart failure, including dyspnea, increased jugular venous pressure, and peripheral edema. Incorrect Answers: A, B, C, and E. Decreased arterial oxygen saturation (Choice A) is not a feature of AV fistulas. Arterial oxygen saturation will be normal. Decreased mixed venous oxygen saturation (Choice B) is not a feature of AV fistulas. Highly oxygenated arterial blood passe

75 ---------- Exam Section 2: Item 25 of 50 National, Board pf Medical Examiners diveB A ance nent * 25. In a healthy 25-year-old man with a normal diet and fluid intake, which of the following substances filtered by the kidney has the greatest fraction excreted in urine? O A) Glucose B) Magnesium C) Sodium D) Urea E) Water

D. The fractional excretion of a solute measures the percentage of the original filtered amount that is present in the excreted urine. The calculations are made using the plasma and urine concentrations of the solute in question. The fractional excretion of a molecule is more reliable than the urine concentration of that molecule as urine concentration can vary with the amount of water present in the urine. Out of the molecules listed, urea has the highest fractional excretion. Urea is created by the urea cycle as a way to excrete nitrogen and prevent the accumulation of ammonia, which is toxic. Urea is filtered by the glomerulus and then reabsorbed in the proximal convoluted tubule at a rate that is lower than that of water. It is also an integral component of the medullary countercurrent system, allowing for the reabsorption of water in the loop of Henle. Approximately 40% of the initially filtered urea is found in the final excreted urine. Incorrect Answers: A, B, C, and E. Glucose (Choice A) has a fractional excretion of nearly 0% in healthy, nondiabetic patients with serum glucose levels less than 180 mg/dL. The proximal convoluted tubule is the primary site of glucose reabsorption. In diabetes mellitus, the filtered load of glucose exceeds the resorptive capacity of the proximal convoluted tubule and glucosuria ensues. Under physiologic conditions, the fractional excretion of magnesium (Choice B) is approximately 3% to 5%. Unlike most other solutes, the site of reabsorption of magnesium is the thick ascending loop of Henle, not the proximal convoluted tubule. The fractional excretion of sodium (Choice C) differs based on volume status but is usually less than 1%. When dehydration is present, increased sodium is reabsorbed to create an osmotic gradient for water to follow. In a well-hydrated patient, there is less

92 ---------- Exam Section 2: Item 42 of 50 National Board of Medical Examiners nent 42. An 18-year-old man is brought to the emergency department 30 minutes after sustaining injuries in a motor vehicle collision. He has abdominal pain. His pulse is 120/min, and blood pressure is 80/60 mm Hg. Physical examination shows multiple contusions over the trunk and abdominal tenderness. A chest x-ray shows multiple rib fractures. A CT scan of the abdomen is shown. Which of the following organs is most likely injured in this patient? O A) Bladder O B) Liver O C) Pancreas D) Spleen O E) Stomach

D. The spleen is one of the most common organs damaged during blunt abdominal trauma, sustaining injuries such as laceration and rupture. The CT image demonstrates the upper abdomen, with a normal-appearing liver in the right upper quadrant and a complex high- grade laceration and rupture of the spleen in the left upper quadrant surrounded by extravasated blood. Splenic rupture can cause large amounts of blood to leak into the peritoneal space, which can result in hemodynamic instability characterized by tachycardia and hypotension. Patients with splenic injuries may have left upper quadrant tenderness, peritonitis from blood products irritating the peritoneum, or referred pain to the left shoulder secondary to diaphragmatic irritation. In hemodynamically stable patients, minor splenic injuries may be treated nonoperatively with observation and reassessment. In unstable patients, exploratory laparotomy is required for repair of the injury or splenectomy, depending on the nature and severity of the injury. Incorrect Answers: A, B, C, and E. The bladder (Choice A) is not shown in this CT image. Blunt abdominal trauma to the lower abdomen or pelvis can cause bladder rupture and urine leakage into the abdominopelvic cavity. Bladder injuries are commonly associated with pelvic fractures. Patients with bladder injuries may present with gross hematuria, suprapubic tenderness, difficulty urinating, or peritonitis from irritation by intraperitoneal urine. Liver (Choice B) injuries in blunt abdominal trauma may cause hemoperitoneum, resulting in hemodynamic instability. However, the CT image reveals a normal appearing liver in the right upper quadrant without any evidence of injury. Pancreatic (Choice C) injuries in blunt trauma are less common. Clinical evaluation is nonspecific for pancreatic injury, and may present with abdom

74 ---------- Exam Section 2: Item 24 of 50 Natjonal, Board of Medical Examinersent V 24. A 49-year-old man comes to the physician because of a 1-month history of lower abdominal pain and increased urge to defecate. Capability to prevent spontaneous defecation is intact. He sustained nondisplaced multiple fractures of the pelvis 2 months ago in a motor vehicle collision. He initially was on bed rest with pelvic support, progressed to non-weight-bearing crutch walking, and now is partial weight bearing with crutches. Medications now include naprosyn three times daily. Physical examination shows an anal wink. Neurologic examination shows intact rectal sphincter tone. This patient is able to sustain cortical control over defecation via which of the following nerves? O A) Conus medullaris O B) Hypogastric nerves O C) Myenteric plexus D) Pelvic nerves E) Pudendal nerves

E. The pudendal nerve is formed from the sacral plexus by the S2, S3, and S4 nerve roots. The nerve carries both sensory and motor fibers and innervates the genitalia and the pelvic floor musculature, including the levator ani muscle and the external anal and urethral sphincters. Pudendal neuropathy can occur in Alcock canal syndrome with pelvic trauma or surgery, and in childbirth. The pudendal nerve courses just medially to the ischial tuberosity, making this an important landmark for regional anesthetic blocks used in vaginal delivery or anorectal surgery. Given this patient's history of pelvic trauma, there is likely disruption involving the complex interactions between sympathetic and parasympathetic nerves that provide autonomic control over urination, ejaculation, and defecation. However, his ability to maintain voluntary (cortical) control over defecation demonstrates an intact neural pathway involving the pudendal nerve, a somatic nerve, even if mild injury to this nerve was sustained. often due to impingement of the pudendal erve between the sacrotuberous or sacrospinous ligaments. This pathology can occur in cyclists, due to prolonged compression in this area, patients Incorrect Answers: A, B, C, and D. The conus medullaris (Choice A) is the distal termination of the spinal cord. Compression of the conus will lead to saddle anesthesia, overflow incontinence, and weakness or paralysis of the bilateral lower extremities. The hypogastric nerves (Choice B) arise from roots of T11-L4 and provide sympathetic innervation to portions of reproductive organs. They are responsible for emission of semen into the posterior urethra which is the first stage of male ejaculation. The myenteric plexus (Choice C) is part of the enteric nervous system located between inner and outer layers of the muscularis externa. Absence of

66 ---------- Exam Section 2: Item 16 of 50 Natjonal, Board of Medical Ęxaminersont * 16. A 57-year-old woman comes to the physician because of chronic shortness of breath and swollen legs for 2 months. Her respirations are 12/min. Physical examination shows edema of the lower extremities. Auscultation of the chest shows each cardiac systole associated with a thrusting impulse palpable along the left sternal border, most prominently in the third, fourth, and fifth intercostal spaces. Which of the following disorders is the most likely cause of this cardiac finding? O A) Aneurysm of the ascending aorta O B) Left atrial hypertrophy O C) Left ventricular hypertrophy D) Pericardial effusion O E) Right atrial hypertrophy F) Right ventricular hypertrophy

F. The physical examination findings of right ventricular hypertrophy may include a parasternal heave (also called a precordial heave), which is an impulse observed or palpated along the left sternal border. The right ventricle is the most anterior of the four heart chambers and lies deep to the third, fourth, and fifth intercostal spaces when in its usual anatomic location. Right ventricular hypertrophy is a consequence of increased right ventricle pressure, which most commonly occurs secondary to chronic lung disease resulting in increased pulmonary vascular resistance and resultant pulmonary hypertension. Chronic lung disease is a risk factor for the development of pulmonary hypertension due to diffuse hypoxia-induced vasoconstriction and pulmonary vascular remodeling. When an area of lung has low oxygen concentration, reflexive contraction of vascular smooth muscle diverts blood flow to a region with increased ventilation. This is called ventilation-perfusion matching. Chronic, severe lung disease leads to widespread pulmonary vasoconstriction, and the increased resistance to flow results in increased right ventricle pressures and development of right ventricular hypertrophy. Other etiologies of right ventricular hypertrophy include primary pulmonary hypertension, severe tricuspid regurgitation, high altitude, left to right cardiac shunts, atrial or ventricular septal defects, anomalous pulmonary venous return, Eisenmenger syndrome, pulmonic valve stenosis, and cardiomyopathy. Patients often present with signs and symptoms of right-sided heart failure, including dyspnea, dependent edema, jugular venous distention, and hepatic congestion. Incorrect Answers: A, B, C, D, and E. Aneurysm of the ascending aorta (Choice A) is a dilatation of the three layers of the aortic wall proximal to the aortic arch. They are often

190 ---------- Exam Section 4: Item 40 of 50 Natjonal, Board of Medical Examinersment * 40. A 3-year-old girl is brought to the physician because of a 1-year history of short stature. She has no history of major medical illness. She is below the 3rd percentile for height and at the 10th percentile for weight. Physical examination shows coarse facial features and contractures of the large joints. X-rays show dysostosis multiplex. Plasma lysosomal enzyme analysis shows increased B-hexosaminidase, B-glucuronidase, B-galactosidase, and a-fucosidase activities. Which of the following mechanisms is the most likely cause of the lysosomal enzyme findings in this patient? O A) Abnormal targeting of these enzymes to lysosomes O B) Deficiency of the other lysosomal enzymes in the cytoplasm O C) Degradation of these enzymes within the cytoplasm O D) Degradation of these enzymes within the lysosomes O E) Storage of these enzymes

A. Abnormal targeting of these enzymes to lysosomes accounts for this patient's findings, which are consistent with mucolipidosis II (I-cell disease). Mucolipidosis II, also known as l-cell (inclusion cell) disease, results from accumulation of oligosaccharides, lipids, and glycosaminoglycans such as heparan sulfate and dermatan sulfate within cells throughout the body. I-cell disease is a type of lysosom based post-translational modification of proteolytic enzymes that would typically be targeted to the lysosome. Proteases targeted to the lysosome are tagged with phosphate at the sixth carbon on their mannose residues, forming mannose 6-phosphate. Defective Golgi N-acetylglucosaminyl-1-phosphotransferase enzymes are unable to join phosphate onto mannose residues. This causes the synthesized enzymes to be abnormally routed into vesicles for exocytosis, instead of to the lysosome. In the absence of these proteins within the lysosome, normal cellular debris that requires lysosomal degradation accumulates within the cells, causing inclusions seen on light microscopy. The resulting accumulation of such products leads to widespread cellular and organ dysfunction. Signs and symptoms of l-cell disease include failure to thrive, developmental delay, coarse facial features, restricted skeletal development, hepatosplenomegaly, cardiac structural defects, corneal clouding, and dwarfism. I-cell disease demonstrates autosomal recessive inheritance; there is no treatment for the condition other than supportive and symptomatic care. storage disease. It is similar in phenotype and pathophysiology to Hurler sy rome. I-cell disease results from a failure of Golgi- Incorrect Answers: B, C, D, and E. Deficiency of the other lysosomal enzymes in the cytoplasm (Choice B) is not a feature of l-cell disease, which results from the failure to

132 ---------- Exam Section 3: Item 32 of 50 Natjonal, Board of Medical Examinersment ensive BasiS nce 32. A 44-year-old woman comes to the physician because of a 3-week history of severe headache. She is 157 cm (5 ft 2 in) tall and weighs 86 kg (190 Ib); BMI is 35 kg/m2. Funduscopic examination shows papilledema. An MRI of the brain shows no abnormalities. A lumbar puncture is done. Cerebrospinal fluid (CSF) analysis shows an opening pressure of 32 cm H20. In addition to recommendations regarding dietary changes and weight loss, treatment with acetazolamide is begun. This drug will most likely improve the patient's symptoms by which of the following mechanisms? A) Decreased CSF production by the choroid plexus O B) Decreased venous pressure around the arachnoid granulations O C) Increased permeability of tight junctions (zonulae occludentes) between endothelial cells O D) Increased sodium permeability of the ependym

A. Acetazolamide is a carbonic anhydrase inhibitor that acts as a diuretic and indirectly decreases CSF production by the choroid plexus. Pseudotumor cerebri, or idiopathic intracranial hypertension, is an idiopathic increase in intracranial pressure (ICP) that typically occurs in obese women of childbearing age and presents with headaches that may be postural, transient visual deficits due to pressure on the optic nerve, and nausea/vomiting (symptoms of increased ICP). Papilledema is comi is typically unremarkable, and lumbar puncture is diagnostic if increased opening pressure is demonstrated. Treatment includes weight loss and acetazolamide, serial lumbar punctures (which decrease ICP), or placement of a CSF shunt in refractory cases. on fundo opic examination. MRI Incorrect Answers: B, C, D, and E. Decreased venous pressure around the arachnoid granulations (Choice B) would lower ICP by promoting venous drainage down a hydrostatic pressure gradient. However, treatments for pseudotumor cerebri, which features an idiopathic etiology of increased ICP, do not utilize this mechanism. Increased permeability of tight junctions (zonulae occludentes) between endothelial cells (Choice C) would lead to increased diffusion of substances from the blood into the CSF, as these tight junctions form the blood-CSF barrier. However, this mechanism would not address increased ICP and might actually increase it. It is not a mechanism employed in pseudotumor cerebri treatment. Increased sodium permeability of the ependyma (Choice D) would cause increased sodium diffusion across the ependyma (the permeable lining of the ventricles that separates the CSF system and brain parenchyma). Increased sodium in the brain parenchyma would not decrease ICP (and may worsen headaches as increased brain sodium levels are proposed as a migraine mechani

102 ---------- Exam Section 3: Item 2 of 50 Natjonal, Board of Medical Examinersment 2. A 50-year-old man collapses and dies while running to catch an airplane. A photograph of his aortic valve is shown. Which of the following best describes the lesion? O A) Calcified congenital bicuspid valve O B) Infective endocarditis O C) Myxomatous degeneration D) Postinflammatory valve stenosis

A. Calcified congenital bicuspid valve describes the lesion depicted in the gross photograph. The normal aortic valve is a tri-leaflet valve, but a bicuspid valve is found in between 0.5% to 2% of the population. It can be inherited or sporadic. Bicuspid valves, as a result of their geometry and abnormal leaflet excursion, are subjected to more turbulent and shear forces, and become calcified much earlier than tri-leaflet valves as a result of chronic inflammation. Bicuspid valves are more prone to the development of early aortic stenosis or aortic regurgitation. Abnormal valve architecture may present a nidus for infection; hence, infective endocarditis may be more frequently seen. The symptoms of aortic stenosis generally include lightheadedness or syncope with exertion, angina pectoris, and dyspnea. Auscultation findings include a crescendo-decrescendo systolic murmur best heard at the upper right sternal border, and pulsus parvus et tardus (weak and delayed) may be noted on examination of peripheral pulses. Due to the chronic increased afterload from a fixed obstruction by the valve, left ventricular hypertrophy and resultant diastolic dysfunction can occur. If early aortic stenosis occurs, the patient may be a candidate for aortic valve replacement. Sudden cardiac death (SCD) may occur in patients with severe aortic stenosis or severe aortic regurgitation. Incorrect Answers: B, C, and D. Infective endocarditis (Choice B) occurs at higher rates in patients with bicuspid aortic valves, but this patient's presentation with SCD is not consistent with endocarditis, which is diagnosed using the Duke criteria. Criteria include positive blood cultures, vascular and immunologic phenomena such as Roth spots, Osler nodes, and Janeway lesions, and evidence of valvular vegetation on imaging. The photograph demonstrates a calci

187 ---------- Exam Section 4: Item 37 of 50 National Board of Medical Examiners mont 37. A previously healthy 32-year-old man is brought to the emergency department because of a 1-day history of intermittent flashing spots and blurred vision, vomiting, confusion, and difficulty walking. He is a painter and says that yesterday he cleaned up a spilled bottle of paint thinner. Ophthalmologic examination shows dilated pupils with hyperemia of the optic disc and retinal edema. Physical examination shows tachypnea. Serum studies show an anion gap metabolic acidosis. The most appropriate pharmacotherapy for this patient is most likely to inhibit the activity of which of the following? A) Alcohol dehydrogenase O B) Aldehyde dehydrogenase O C) Formaldehyde dehydrogenase O D) Lactate dehydrogenase O E) Pyruvate dehydrogenase

A. Methanol is commonly used as paint thinner or as an additive in paint thinner. Methanol can be absorbed into the body via ingestion, inhalation, and skin or eye contact. Once absorbed, methanol can be converted by alcohol dehydrogenase to formaldehyde, which is then converted by aldehyde dehydrogenase into formic acid, a toxic metabolite that depression (eg, confusion, ataxia). The production of formic acid results in the development of an anion gap metabolic acidosis with a compensatory respiratory alkalosis, manifesting as tachypnea. Treatment requires the administration of fomepizole, which inhibits alcohol dehydrogenase and blocks the conversion of methanol to its toxic metabolites. Severe methanol toxicity resulting in refractory metabolic acidosis, refractory electrolyte abnormalities, visual disturbances, or deteriorating vital signs may require dialysis. visual disturbances, retinal edema, otic disc hyperemia, gastrointestinal distress (eg, abdominal pain, nausea, vomiting), putaminal hemorrhage, and neurologic cau Incorrect Answers: B, C, D, and E. Aldehyde dehydrogenase (Choice B) and formaldehyde dehydrogenase (Choice C) are different subtypes of an enzyme catalyzing the second step in metabolic processing of alcohol, methanol, or ethylene glycol following the initial enzymatic conversion by alcohol dehydrogenase. Formaldehyde dehydrogenase converts formaldehyde into formic acid in methanol metabolism, while aldehyde dehydrogenase converts acetaldehyde into acetate in alcohol metabolism, and glycoaldehyde into glycolic acid in ethylene glycol metabolism. Disulfiram inhibits some isoforms of aldehyde dehydrogenase but is not used in the acute management of methanol toxicity. Lactate dehydrogenase (Choice D) is involved in anaerobic metabolism with the conversion of pyruvate to lactate. It has no direct rol

189 ---------- Exam Section 4: Item 39 of 50 National, Board pf Medical Examiners Comprehansive aniSaanca ament 39. A 57-year-old man is brought to the emergency department because of a 2-hour history of severe abdominal pain and vomiting. Abdominal examination shows diffuse tenderness. A CT scan of the abdomen and abdominal radiograph are shown; the arrows indicate an abnormality. The most likely cause of these findings is perforation of which of the following structures? A) Duodenum O B) Esophagus O C) lleum O Đ}Stomach E) Transverse colon

A. Peptic ulcer disease describes the presence of ulcers in the stomach or duodenum, which classically present with worsening abdominal pain related to consumption (gastric) or lack of consumption (duodenal) of food. Gastric ulcers can be caused by prolonged or excess usage of nonsteroidal anti-inflammatory drugs which inhibits cyclooxygenase enzymes in the gastrointestinal tract mucosa, leading to a reduction of prostaglandin secretion and decreased protection of the gastric mucosa. However, the most common cause of peptic ulcer disease is infection with the bacterium Helicobacter pylori, which accounts for nearly all duodenal ulcers. Persistent inflammation related to a peptic ulcer can result in complications including fibrosis, stricture, hemorrhage, and perforation. Anterior wall ulcers are associated with a higher risk of perforation, while posterior wall ulcers demonstrate an increased risk of hemorrhage given their position near the gastroduodenal artery. Perforation commonly presents with acute abdominal pain, peritonitis, fever, and free air on radiographs and CT. This patient's CT demonstrates air within the retroperitoneum, which indicates the perforation must have occurred in a portion of the bowel that is retroperitoneal. The duodenum's first segment is intraperitoneal, but the second through fourth segments are retroperitoneal, which is the likely location of the perforated ulcer. Incorrect Answers: B, C, D, and E. The esophagus (Choice B) is unlikely to be perforated in this patient. Esophageal perforation is rare and can be iatrogenic secondary to recent instrumentation, such as with an endoscopy, or secondary to forceful vomiting causing spontaneous esophageal rupture. It can present with dysphagia, sharp chest pain, hematemesis, and subcutaneous emphysema in the chest or neck. lleal (Choice C) perfor

50 ---------- Exam Section 2: Item 1 of 50 National, Board of Medical Examiners Comprehensive Basic Science Self-Assessment 1. A 35-year-old woman and her 35-year-old husband come to the physician for genetic counseling after their son is diagnosed with a rare metabolic disease. Assuming Hardy-Weinberg equilibrium, the physician tells the parents about the incidence and carrier frequency of this disorder. Which of the following is most likely to disturb the Hardy-Weinberg equilibrium of this disorder? A) Appreciable rate of gene mutation B) Random matings in the population C) Relatively large population D) No selection against a certain genotype E) No significant immigrant population

A. The Hardy-Weinberg principal, known as Hardy-Weinberg equilibrium, proposes that allele frequencies will remain constant across generations in the absence of evolutionary change. This model of population genetic stability requires a number of assumptions, including that natural selection does not occur, that there are no random mutations, that no movement in or out of the population occurs in equilibrium, and that mating is completely random. With these assumptions, the frequencies of alleles can be calculated as they will remain constant over time. If deviations from these assumptions occur, the allele frequencies within the population will change over time. The Hardy-Weinberg equilibrium equation is: p< + 2pq + q² = 1, where p is the frequency of the dominant allele and q is the frequency of the recessive allele. 2pq is the probability of heterozygosity, p- is the probability of dominant homozygote, and q- is the probability of being a recessive homozygote. If there is an appreciable rate of gene mutation occurring within the population, over time the allele frequency will change. ASsuming equivalent mutation rates in each gene, eventually all allele frequencies would change. Gene mutation prevents the stability or equilibrium of genotypes over time. %3D Incorrect Answers: B, C, D, and E. Random matings in the population (Choice B) are required for Hardy-Weinberg equilibrium. Nonrandom mating promotes imbalances in the equilibrium with resultant selection for or against certain alleles. A relatively large population (Choice C) is not necessary for Hardy-Weinberg equilibrium. However, small population sizes with small numbers of particular alleles are more subject to genetic drift. No selection against a certain genotype (Choice D) is required for Hardy-Weinberg equilibrium. If a certain gene is selected for, that

59 ---------- Exam Section 2: Item 9 of 50 National Board of Medical Examiners Comprehensive Basic Science Self-Assessment me diapnysis ana disrupuon or tne cortex WILN Tocal areas onincreased caiciication. Agiistening mass containng severa cysts is surgicany excIsed. A pnotomicrograpn or ussue Trom the mass is shown. Which of the following is the most likely diagnosis? A) Atypical stress fracture B) Chondrosarcoma C) Enchondroma D) Ewing sarcoma E) Giant cell carcinoma F) Multilobulated bone cyst G) Osteosarcoma

B. Chondrosarcoma is a malignant neoplasm that arises from the mesenchymal cells that produce cartilage. It commonly occurs in the axial skeleton, particularly in the pelvis, and often in patients in the fifth decade of life. It is a slow-growing tumor that is initially asymptomatic, permitting extensive growth before detection. Pelvic chondrosarcoma will often present with pelvic pain and compression of adjacent structures such as the lumbosacral plexus, leading to neurologic symptoms in the ipsilateral lower extremity. Chondrosarcoma can be primary or arise secondary to

52 ---------- Exam Section 2: Item 3 of 50 National, Board of Medical Examiners Comprehensive Basic Science Self-Assessment 3. A 68-year-old man comes to the physician's office because of the sudden onset of loss of vision in the left eye. There is no pain. He has a 15-year history of well controlled type 2 diabetes mellitus and hypertension. He had a myocardial infarction 2 years ago. Temperature is 36.7°C (98.1°F), pulse is 72/min, respirations are 16/min, and blood pressure is 200/100 mm Hg. Funduscopic examination shows a pale, white retina with retinal artery narrowing and decreased filling of the retinal veins. Which of the following is the most likely cause of the loss of vision? A) Malignant hypertension B) Retinal artery embolism C) Retinal detachment D) Retinal vein thrombosis E) Vitreous hemorrhage secondary to neovascularization

B. Central retinal artery occlusion (CRAO) presents with sudden, monocular vision loss, which is usually painless. Vision loss is typically profound, with severely reduced visual acuity, and occurs over seconds. There may be a history of preceding amaurosis fugax. Physical examination shows a relative afferent pupillary defect, a pale fundus with a cherry red spot, and narrowing of the retinal arterioles with arteriolar boxcarring. The most common cause of CRAO is a retinal artery embolism from a carotid artery atheroma or from a cardiac valvular vegetation. Embolic disease may also manifest with Hollenhorst plaques, which are smaller, refractile deposits that can be visualized at the bifurcations of retinal arterioles. CRAO is considered a form of embolic stroke affecting the eye, and suspicion of CRAO should therefore prompt an immediate stroke risk factor evaluation. ČRÃO may also occur secondary to giant cell arteritis. There is no proven effective treatment for CRAO and the prognosis for visual recovery is poor. Patients who have been affected by CRAO are at risk of secondary ocular neovascularization and glaucoma due to persistent retinal ischemia. Incorrect Answers: A, C, D and E. Malignant hypertension (Choice A) presents with optic disc edema, retinal arteriolar narrowing with arteriovenous nicking and silver wiring, retinal edema, cotton wool spots, and flame hemorrhages. It is typically a symmetric, bilateral process. Retinal detachment (Choice C) presents with painless, monocular vision loss but is typically preceded by photopsia and bursts of floaters. The detachment is identifiable on dilated fundus examination as a distinct region of retinal elevation and is not usually associated with retinal whitening, arteriolar narrowing, or reduced venous filling. Retinal vein thrombosis (Choice D), presents with

155 ---------- Exam Section 4: Item 5 of 50 Natjonal, Board of Medical Examinersment aiveB A lance S 5. A 57-year old man has a hemoglobin concentration of 18.5 g/dL. A peripheral blood smear is shown. These findings are most consistent with which of the following disorders? O AJAgnogenie myeloid metaplasia O B}Chronic ebstructive pulmonary disease O C) Hereditary hemochromatosis O D) Hypersplenism E) Myelodysplastic syndrome

B. Chronic obstructive pulmonary disease (COPD) is an obstructive lung disease characterized by decreased lung function from a combination of features of chronic bronchitis and/or emphysema, resulting in airway obstruction on expiration. COPD commonly presents with chronic hypoxemia, which triggers increased erythropoietin (EPO) production by the kidneys. EPO induces the proliferation and maturation of erythroid progenitor cells in the bone marrow and results in secondary polycythemia. Polycythemia refers to an increased absolute red blood cell mass in the body. It is reflected by an increased hemoglobin concentration and an increased density of normochromic, normocytic erythrocytes on peripheral blood smear. Other common causes of secondary polycythemia are obstructive sleep apnea, obesity hypoventilation syndrome, living at high altitude, and performance-enhancing drugs such as exogeneous erythropoietin, testosterone, and anabolic steroids. Incorrect Answers: A, C, D, and E. Agnogenic myeloid metaplasia (Choice A), also known as primary myelofibrosis, is a chronic myeloproliferative disorder characterized by the progressive fibrosis and ablation of the bone marrow. It is commonly associated with JAK2 mutations. Peripheral blood smear typically reveals leukoerythroblastosis (immature myeloid cells and nucleated red blood cells) along with teardrop-shaped erythrocytes. Hereditary hemochromatosis (Choice C) is an inherited disorder characterized by the abnormal accumulation of iron in the body. It typically presents with liver failure, diabetes, arthritis, heart failure, darkening of the skin, and gonadal atrophy. Diagnostic studies may include liver biopsy, commonly demonstrates excess iron deposition seen in hepatocytes on Prussia blu stain. Hypersplenism (Choice D) is defined as the presence of one or more cytopenias

185 ---------- Exam Section 4: Item 35 of 50 National Board of Medical Examiners ment * 35. A 21-year-old man comes to the physician because of a lump in his scrotum for 3 months. Physical examination shows a 1-cm nontender nodule in the right scrotum. Transillumination shows a solid, noncystic structure. An operation to remove the testis is done. After incision of the skin and subcutaneous tissue, the next fascial layer to be incised in this patient is a direct extension of which of the following abdominal layers? O A) Dartos muscle O B) External abdominal oblique muscle O G} Internalabdominal oblique muscle O Đ} Peritoneum E) Transversalis fascia

B. From superficial to deep, the layers of the scrotum include the skin, the Dartos muscle and fascia, the external spermatic fascia, the cremaster muscle and fascia, the internal spermatic fascia, and the tunica vaginalis. Each of these layers is derived from a layer of the abdominal wall because the fetal testes originated in the abdomen. Early in the third trimester the testes begin to migrate through the inguinal canal and into the scrotum, carrying with them the abdominal wall coverings. While the structures are contiguous, they are named differently in the abdomen and scrotum. In this case, the skin and next layer, the Dartos muscle and fascia (subcutaneous tissue), have been incised. The following layer is the external spermatic fascia which is derived from the external abdominal oblique muscle aponeurosis. Incorrect Answers: A, C, D, and E. The Dartos muscle (Choice A) is derived from Scarpa fascia and is the second layer that would be incised when performing an orchiectomy and has already been incised in this patient. The internal abdominal oblique muscle (Choice C) will produce the cremasteric muscle and fascia in the scrotum. This layer is deep to the external spermatic fascia. After migration, the abdominal peritoneum (Choice D) becomes the tunica vaginalis, which is the deepest layer of the scrotum and forms a protective sheath around the testes. The transversalis fascia (Choice E) will produce the internal spermatic fascia of the scrotum, which is the layer deep to the cremasteric muscle and fascia. Educational Objective: Each layer of the testes is derived from a layer of the abdominal wall (listed in parentheses). From superficial to deep, the layers of the scrotum include the skin (skin), the dartos muscle and fascia (Scarpa fascia), the external spermatic fascia (external abdominal oblique muscle and

150 ---------- Exam Section 3: Item 50 of 50 National Board of Medical Examiners ivB ience mont 50. A 46-year-old woman comes to the office because of a 3-month history of fever, malaise, and easy bruising. Her mother and maternal grandmother died of breast cancer at the ages of 49 and 52 years, respectively. Her temperature is 38°C (100.4°F), pulse is 115/min, respirations are 16/min, and blood pressure is 125/90 mm Hg. Physical examination shows numerous large ecchymoses over the trunk and lower extremities. Laboratory studies show an increased leukocyte count with immature segmented neutrophils. Genetic testing shows the presence of the Philadelphia (Ph1) chromosome. Which of the following best explains this patient's current condition? O AJABL activation of the p23-tumor suppresser gene B) Fusion protein influence on the cell cycle O C) Point mutation in the ABL oncogene O D) Point mutation in the BCR gene O EJ

B. Fusion protein influence on the cell cycle accounts for this patient's presentation, which is consistent with chronic myelogenous leukemia (CML). The Philadelphia (Ph) chromosome is created by a translocation between chromosomes 9 and 22, leading to constitutive activation of the ABL1 tyrosine kinase. Activation of the ABL1 tyrosine kinase leads to subsequent activation of the JAK/STAT and Ras/MAPK/ERK pathways, with the downstream consequences of increased cellular inhibition, and resistance to apoptosis. While the Ph chromosome is required for the diagnosis of CML, it can also be found in acute lymphoblastic leukemia (ALL) and acute myelocytic leukemia (AML). Typical laboratory findings in CML include hyperleukocytosis, with an increase in nearly all of the cell lines. Basophilia and eosinophilia are particularly characteristic. Leukostasis, a condition in which hyperleukocytosis leads to organ damage from occlusion of capillaries by malignant, non-distensible cells, is uncommon in CML because malignant cells are differentiated and pliable. This is in contrast to conditions such as AML and ALL in which circulating blasts, which are large and not particularly pliable, easily lodge in capillaries. Treatment of CML is with tyrosine kinase inhibitors such as imatinib or dasatinib. They are remarkably effective and often result in complete molecular response within several months of initiation. tion, inhibition of normal ckpoint Incorrect Answers: A, C, D, and E. ABL activation of the p23 tumor suppressor gene (Choice A) is not the defining feature of the Philadelphia chromosome. Activation of a tumor suppressor gene would result in inhibition of tumor growth, as opposed to cellular proliferation. Point mutation in the ABL oncogene (Choice C) does not define the Philadelphia chromosome. In CML, the ABL oncogene is cons

198 ---------- Exam Section 4: Item 48 of 50 National Board of Medical Examiners andive Rasic Science ment 48. A 26-year-old man comes to the physician because of a 1-week history of temperatures to 38.3°C (101°F), chills, and chest and back pain. Two weeks ago, he returned from a trip to Costa Rica, where he had been performing ecologic studies of bats in their cave habitats. He reports exposure to bat urine and feces during his studies. Today, his temperature is 38.1°C (100.6°F). Physical examination shows no other abnormalities. A complete blood count and serum chemistry studies are within the reference ranges. A chest x-ray shows bilateral reticulonodular infiltrates. The virulence of the most likely infectious agent in this patient depends on which of the following? O AJElaboration-of polyribitol-phosphate-capsule B) Growth inside nonactivated macrophages O G) Invasion of host tissues and vasculature O D) Ph

B. Histoplasmosis is an endemic mycosis secondary to Histoplasma capsulatum. Pulmonary histoplasmosis can present in acute, subacute, or chronic phases, generally characterized by fever, malaise, myalgia, cough, and hemoptysis with the associated development of calcified nodules and mediastinal or hilar lymphadenopathy. It is endemic to the capsulatum is dependent on its growth inside nonactivated macrophages, which protect the fungal cells from detection and elimination. Risk factors for histoplasmosis include exposure to bird or bat droppings. Diagnosis is typically made via the detection of histoplasma antigen in the serum and/or urine. sippi and Ohio River valleys in the United States, as well as parts of Central and South America, sub-Saharan Africa, and Southeast Asia. The virulence of Histoplasm Incorrect Answers: A, C, D, and E. Elaboration of polyribitol phosphate capsule (Choice A) is the major virulence factor of Haemophilus influenzae, which confers resistance to phagocytosis. Haemophilus influenzae is a Gram-negative bacterium that can result in mucosal infections such as conjunctivitis, otitis media, bronchitis, and pneumonia. It was the leading cause of epiglottitis in children prior to widespread immunization. Invasion of host tissues and vasculature (Choice C) is the common mechanism by which virulent microbes cause disease, resulting in localized and disseminated infection. Histoplasma capsulatum virulence, rather, is dependent on its replication within nonactivated macrophages, as this serves to protect the organism from the immune system. Phenotypic switching from yeast to hyphae in the host (Choice D) is a characteristic of Candida albicans. It is a dimorphic fungus that switches forms depending on temperature. It can cause systemic infections in immunocompromised patients such as esophagitis and d

162 ---------- Exam Section 4: Item 12 of 50 Natjonal, Board of Medical Examinersment ansive Rasic Salance * 12. In an experimental design evaluating assays for the bactericidal effect of inflammatory cells, which of the following compounds is considered the most efficient in the killing of bacteria by neutrophils? O A) Bactericidal permeability increasing protein B) Hypochlorite O C) Lactoferrin O D) Lysozyme E) Major basic protein

B. Hypochlorite is important in the efficient killing of bacteria by neutrophils as it is a component of the "respiratory burst," a process employed by neutrophils that results in rapid bactericidal activity. The respiratory burst describes a process by which neutrophils and phagocytes utilize oxygen to produce free radicals, hypochlorite, and peroxides, all of which are toxic to rapidly dividing bacterial cells. The process begins with oxygen free radical generation via the action of NADPH oxidase, with additional conversion to hydrogen peroxide by superoxide dismutase. The addition of a chloride ion to hydrogen peroxide by myeloperoxidase creates hypochlorite. The actions of superoxide anions, hydrogen peroxide, and hypochlorite result in rapid and efficient bactericidal activity. Incorrect Answers: A, C, D, and E Bactericidal permeability increasing proteins (Choice A) are released by neutrophils in response to infections with both Gram-positive and Gram-negative bacteria, but their role has been best described in Gram-negative infections. They bind to the bacterial lipopolysaccharide wall (LPS), where they increase bacterial membrane permeability and contribute to opsonization. Lactoferrin (Choice C) scavenges and sequesters iron, thereby preventing bacterial pathogens from using iron, an important component of many necessary oxidation-reduction reactions. It has bacteriostatic properties and does not directly act to kill bacteria. Lysozyme (Choice D) is a bactericidal enzyme that hydrolyzes the bonds between layers of the Gram-positive bacterial peptidoglycan wall. It reduces the integrity of this structure and indirectly contributes to bacterial death. Major basic protein (Choice E) is contained primarily within eosinophilic granules and is released in response to infection with helminths. While it also plays a r

142 ---------- Exam Section 3: Item 42 of 50 National Board of Medical Examiners andive Rasic Science mont 42. A 38-year-old man with a 3-year history of type 2 diabetes mellitus comes to the physician for a follow-up examination. In addition to taking an oral antihyperglycemic agent, he has tried controlling his condition with diet modifications and exercise. He is 188 cm (6 ft 2 in) tall and weighs 113 kg (250 Ib); BMI is 32 kg/m2. Physical examination shows no other abnormalities. His hemoglobin A1ç is 10%. The physician recommends initiation of insulin injections to obtain better control over the patient's blood glucose concentration. The patient responds, "I know that insulin would help control my blood sugar. But a lot of people in my family have diabetes, and insulin made them really sick at times." This patient is most likely at which of the following stages of change regarding insulin administration? A) Pre

B. In this scenario, the patient is aware of his poorly controlled blood glucose and insulin's potential benefits. He is considering initiating insulin, but he is indecisive about whether he is ready to take action due to concern about side effects, which is consistent with the contemplation stage. The stages of behavioral change are used to define a patient's readiness to change a health-related behavior such as treatment adherence, substance use, diet, or exercise habits. In sequential order, the stages of behavioral change are precontemplation, contemplation, preparation, action, maintenance, and termination. Physicians aim to move patients through these stages over time with an interview technique called motivational interviewing. Motivational interviewing involves using open-ended, non-judgmental questions to help the patient explore their reasons for wanting to change or maintain the habit. Incorrect Answers: A, C, D, and E. Patients in the precontemplation stage (Choice A) are not interested in changing their habit and may not see the benefit in making a change. This patient is aware of the potential benefit of insulin and is considering starting it, which means he has surpassed the precontemplation stage. Patients in the preparation stage (Choice C) have committed to making a change and are ready to discuss strategies and resources to help them make the change. For example, a patient may research information about a low-carbohydrate diet to improve his eating habits. This patient expresses knowledge of insulin's potential health benefits but is not yet committed to starting this medication. In the action stage (Choice D), patients start to take steps to change their behavior. For example, a patient may begin a low-carbohydrate diet to improve his unhealthy eating habits. After making the change, the patient mus

148 ---------- Exam Section 3: Item 48 of 50 National Board of Medical Examiners mant 48. A 56-year-old man with esophageal varices who is awaiting liver transplantation undergoes placement of a transjugular portosystemic shunt. During the procedure, a catheter with a stent is threaded through the right internal jugular vein and down through the vena cava to the liver. To relieve the portal hypertension via this route, the stent should be placed between a branch of the portal vein and which of the following additional veins? O A) Esophageal B) Hepatic C) Left gastric O D) Splenic O E) Superior mesenteric

B. Portal hypertension (PH) is often a consequence of liver cirrhosis but may also occur in patients with schistosomiasis or portal vein thrombosis. The portal venous system describes a series of interconnected veins that drain blood from the colon, small intestines, spleen, liver, and stomach. While this blood eventually makes its way to the systemic circulation via the inferior vena cava, it must first pass through the liver. In cirrhosis, obliteration of the hepatic sinusoids through progressive fibrosis increases the resistance to blood flow through the liver, which is transmitted to the portal venous system causing PH. Anatomically, the portal vein starts at the confluence of the left gastric, splenic, and superior mesenteric veins. Manifestations of PH include esophageal varices, gastric varices, caput medusae, hemorrhoids, splenomegaly, and ascites. The hepatic veins are not part of the portal venous system and act to drain blood from the liver into the inferior vena cava. In patients with cirrhosis who have ascites that is refractory to diuretic therapy or life-limiting complications of PH such as bleeding varices, the placement of a transjugular intrahepatic portosystemic shunt (TIPS) is an option for relieving portal hypertension. In this procedure, a stent is placed between the portal vein and the hepatic vein, effectively allowing blood to bypass the liver. One common side effect of TIPS is more frequent and severe episodes of hepatic encephalopathy, which is a drawback to this procedure. Incorrect Answers: A, C, D, and E. Esophageal veins (Choice A), if inferior, drain into the left gastric vein and then into the portal vein. They may become engorged in portal hypertension resulting in esophageal varices (EVs). These are usually treated endoscopically by band ligation. Patients with cirrhosis must be scree

141 ---------- Exam Section 3: Item 41 of 50 Natjonal, Board of Medical Examinersment ansive Basic Saance * 41. A 6-year-old girl is brought to the physician for a follow-up examination. She has been receiving treatment with potassium citrate monohydrate since the diagnosis of renal tubular acidosis was made at the age of 2 years. Physical examination shows no abnormalities. Serum studies show: Na+ K+ 142 mEq/L 3.5 mEq/L 115 mEq/L 18 mEq/L CI- НСО3- Urea 9 mg/dL nitrogen Creatinine 0.9 mg/dL A defect in renal ammoniagenesis is suspected. Which of the following substrates is the most likely source of ammonia production in this patient? O A) Creatinine B) Glutamine O C) Glycine O D) Leucine O E) Urea

B. Production of ammonia, or ammoniagenesis, takes place in the proximal convoluted tubule of the renal nephron. Ammonia, NH3, is able to accept a proton (H*) to become ammonium, NH4*, and thus aids in the renal excretion of acid into the urine. Unlike other compounds, the majority of the ammonia in the nephron lumen comes from renal tubular cell production rather than glomerular filtration. The amino acid glutamine is the primary substrate for the production of ammonia. Glutamine is converted into a-ketoglutarate and two molecules of ammonium through the sequential enzyme reactions of glutaminase and glutamate dehydrogenase. These ammonium molecules are then secreted into the lumen of the proximal convoluted tubule via a sodium-ammonium exchanger. There are three main types of renal tubular acidosis (RTA), all of which prevent the kidneys from either absorbing HCO3 or excreting titratable acid in the form of NH4*. In Type 1, or distal RTA, there is impaired secretion of protons by the alpha intercalated cells of the collecting tubule. Type 2, or proximal RTA, is caused by impaired HCO3 resorption in the proximal convoluted tubule. In Type 4 RTA, decreased levels of aldosterone result in decreased NH4T excretion. Types 1 and 2 are most common in the pediatric population and can be inherited or acquired. Any cause of decreased ammonia production will compromise the kidney's ability to excrete acid and lead to a metabolic acidosis. Incorrect Answers: A, C, D, and E. Creatinine (Choice A) is a byproduct of creatine, which is produced by muscle cells. Creatinine is excreted by the kidney and used clinically as an indicator of renal function, but it is not a substrate for ammoniagenesis. Glycine (Choice C) and leucine (Choice D) are both amino acids. Neither of their side chains have amine groups, and thus cannot be used as

194 ---------- Exam Section 4: Item 44 of 50 Natjonal, Board pf Medical Ęxaminersment. * 44. A healthy 21-year-old woman is concerned that her son will develop a familial neurologic disorder. Her older brother died at the age of 45 years after developing restlessness, forgetfulness, incoordination, altered handwriting and speech, and subsequently, chorea. Autopsy showed atrophy of the basal ganglia. Their mother died at the age of 55 years of a similar disorder. Which of the following best approximates the risk of this woman's son for developing the disorder? O AJEssentially zero O B) 25% O G+33% O Đ)50% E) 100%

B. The chance of this woman's son developing Huntington disease (HD) is 25%. HD demonstrates an autosomal dominant pattern of heritability. The woman's mother possessed HD symptoms and died early and therefore presumably possesses one copy of the mutated gene. This woman has a 50% chance of inheriting the gene from her mother and a 50% chance of passing on this mutated gene to her son (50% × 50% = 25%). HD is a heritable progressive neurodegenerative disorder typically arising in the third to sixth decades of life associated with atrophy of the caudate nuclei in the basal ganglia. The disease usually presents with an insidious onset of psychiatric (irritability, depression, psychosis) and cognitive symptoms. Motor symptoms (chorea, hypotonia, dystonia, loss of voluntary control of movements) typically appear later in the disease course. People with HD typically die 10 to 40 years after disease onset. Chorea, a key HD symptom, can be initially experienced as restlessness. Treatment is symptomatic and can include tetrabenazine or deutetrabenazine, which target chorea by depleting dopamine stores. HD also demonstrates anticipation, a genetic inheritance phenomenon in which future generations are affected at a younger age or with more severe symptoms. Incorrect Answers: A, C, D, and E. The son's risk would be essentially zero (Choice A) if the woman had demonstrated a negative result on genetic testing for HD. Alternatively, if the woman were older than 80 years old without any HD symptoms, the risk that she would develop HD may be low enough to assume she does not possess the mutated HD gene and that her son's risk is therefore negligible. Risks of 50% (Choice D) or 100% (Choice E) would signify that either the woman or her son demonstrated the mutated HD gene on genetic testing, respectively. The son's risk would be 50%

28 ---------- Exam Section 1: Item 29 of 50 National Board of Medical Examiners Comprehensive Basic Science Self-Assessment 29. A 40-year-old woman comes to the physician for a follow-up examination 1 week after having a second increased blood pressure measurement. Her pulse is 72/min, and blood pressure is 168/104 mm Hg in both arms while sitting. Physical examination shows no other abnormalities. Treatment with lisinopril and hydrochlorothiazide is initiated. One month later, her blood pressure is 120/80 mm Hg. Which of the following sets of hormonal changes is most likely in this patient? Angiotensin I Angiotensin II Bradykinin A) ↑ no change B) ↑ ↑ C) D) ↑ ↑ E)

B. Thiazide diuretics, such as hydrochlorothiazide (HCTZ), and ACE inhibitors, such as lisinopril, are first-line therapies for the treatment of hypertension. ACE inhibitors target the renin-angiotensin-aldosterone system which is one of the primary regulators of blood pressure. The system is activated by the release of renin from the renal juxtaglomerular apparatus in response to low-flow states in the afferent arterioles or through the distal convoluted tubule (decreased afferent arteriole pressure and decreased delivery of Na+ and Cl- ions to the macula densa both stimulate renin release). Renin acts on angiotensinogen via proteolytic cleavage to generate angiotensin I. Angiotensin I is then converted to angiotensin II by ACE, which is found in vascular endothelium (particularly in the lungs). Angiotensin II has several important effects, including potent vasoconstriction leading to increased systemic vascular resistance, increased Na+ reabsorption in the renal tubules, aldosterone release by the adrenal cortex leading to increased Na+ and fluid retention, vasopressin release from the posterior pituitary leading to increased fluid retention, enhanced adrenergic function through inhibition of norepinephrine reuptake in sympathetic nerve endings, and stimulation of hypothalamic thirst centers. All of these effects serve to increase blood pressure. ACE inhibitors decrease blood pressure by blocking the conversion of angiotensin I to angiotensin II. This results in increased angiotensin I, decreased angiotensin II, and increased bradykinin levels, as ACE is also responsible for the degradation of bradykinin (a potent vasodilator). Thiazide diuretics act on the distal convoluted tubule to decrease Na+ and Cl- reabsorption and do not directly affect the renin-angiotensin-aldosterone pathway. Incorrect Answers: A, C, D, an

45 ---------- Exam Section 1: Item 46 of 50 National Board of Medical Examiners Comprehensive Basic Science Self-Assessment 46. A 35-year-old man comes to the physician because of a 6-month history of burning abdominal pain that occurs 1 to 2 hours after he eats. He also has had black stools for 2 days. Use of over-the-counter antacids and histamine-2 (H2)-receptor blocking agents has not been effective in relieving his symptoms. He is sweating profusely and has light-headedness when he stands. His blood pressure is 105/70 mm Hg while sitting. Physical examination shows epigastric tenderness. A CT scan of the abdomen shows a 1-cm mass in the pancreas. Immunohistochemical labeling of neoplastic cells in a biopsy specimen is most likely to involve the use of antibodies directed to which of the following substances? A) Amylase B) Gastrin C) Glucagon D) Human pancreatic polypeptide E) Insulin F) Lipase G) Serotonin H) So

B. This patient's findings of postprandial abdominal pain, melena, diaphoresis, and orthostatic hypotension in the setting of a pancreatic lesion are suggestive of a peptic ulcer secondary to Zollinger-Ellison syndrome (ZES). ZES stems from a pancreatic or duodenal gastrin-secreting tumor. Ġastrin is typically produced by gastric G cells and stimulates parietal cells to produce hydrochloric acid. Excessive production of acid in the setting of ZES leads to recurrent, chronic duodenal or jejunal ulcers, which can present with abdominal pain, diarrhea secondary to malabsorption, and possible hematemesis, melena, or hematochezia. Immunohistochemical labeling of neoplastic cells using an anti-gastrin antibody reveals positive staining. Pancreatic gastrinoma and ZÉS may present as a component of multiple endocrine neoplasia type 1 (MEN1). Incorrect Answers: A, C, D, E, F, G, H, and I. Amylase (Choice A) assists in the digestion of carbohydrates and is secreted in saliva and from the exocrine pancreas. Increased levels of amylase may indicate sialadenitis or pancreatitis. Glucagon (Choice C) is produced by pancreatic alpha cells and primarily works to increase serum glucose. Positive glucagon staining on histology is characteristic of a glucagonoma, which presents with mild diabetes mellitus, necrolytic migratory erythema, and weight loss. Human pancreatic polypeptide (Choice D) regulates pancreatic endocrine and exocrine secretions. Its overexpression is nonspecific and may be observed in a variety of different hormonally active pancreatic tumors. Insulin (Choice E) is produced by pancreatic beta cells and primarily works to lower serum glucose. Positive staining for insulin on histology is characteristic of an insulinoma, a rare neoplasm that produces insulin. Patients present with lethargy, weakness, fatigue, diplopia, a

9 ---------- Exam Section 1: Item 10 of 50 National Board of Medical Examiners Comprehensive Basic Science Self-Assessment 10. A 37-year-old man comes to the physician because of a 1-week history of nausea, vomiting, and abdominal pain. He ate raw oysters while vacationing in Mexico 2 weeks ago. He has smoked 1 pack of cigarettes daily for the past 15 years but has now lost the desire to smoke. Physical examination shows scleral icterus and slight enlargement and tenderness of the liver. Which of the following pathogens is most likely responsible for the development of these symptoms in this patient? A) Epstein-Barr virus B) Hepatitis A virus C) Hepatitis B virus D) Hepatitis C virus E) Vibrio vulnificus

B. This patient's presenting findings of nausea, vomiting, abdominal pain, icterus, hepatomegaly and right upper quadrant tenderness two weeks after eating raw shellfish is consistent with an acute hepatitis A virus (HAV) infection. HAV is a non-enveloped, single- stranded RNA virus that is spread through fecal-oral transmission. It is commonly acquired through ingestion of poorly cooked, improperly handled, or raw foods, including shellfish. HAV has an incubation period of several weeks. Presenting signs and symptoms include fever, nausea, vomiting, poor appetite, abdominal pain, jaundice, and hepatomegaly, although many patients are asymptomatic. Laboratory findings include transaminitis, increased alkaline phosphatase, hyperbilirubinemia, and bilirubinuria. The diagnosis is confirmed by the serologic detection of anti-HAV IgM antibodies. HAV infection is typically a self-limited illness and does not progress to chronic infection. Incorrect Answers: A, C, D, and E. Epstein-Barr virus (Choice A) causes infectious mononucleosis, which presents with fever, hepatosplenomegaly, pharyngitis, and posterior cervical lymphadenopathy. It is transmitted primarily through saliva and respiratory secretions and is not transmitted through uncooked food. Hepatitis B (Choice C) and C virus (Choice D) are transmitted through bodily fluids. Hepatitis B is commonly transmitted through blood, sexual contact, or perinatally. Hepatitis C is commonly transmitted through blood and is especially common in the setting of intravenous drug use. The patient's history is suggestive of oral transmission through raw shellfish. Vibrio vulnificus (Choice E) is a Gram-negative, primarily water-borne bacterium that causes sepsis or bacteremia and is associated with the consumption of raw shellfish. V. vulnificus sepsis presents acutely after the ingesti

55 ---------- Exam Section 2: Item 6 of 50 National Board of Medical Examiners Comprehensive Basic Science Self-Assessment 6. An investigator obtains a serum sample from an individual previously immunized with a vaccine that elicits immunity to Clostridium tetani. Administration of this serum to a guinea pig prior to challenge with a potentially lethal dose of C. tetani is most likely to mediate protection to the challenge dose through binding of antibody to which of the following products of the bacterial cells? A) Capsular polysaccharide B) Cell wall polysaccharide C) Exotoxin D) Fimbria E) Flagellum F) Peptidoglycan

C. Clostridium tetani is the bacterial cause of tetanus, resulting in the development of spastic paralysis following bacterial inoculation via a penetrating traumatic wound. Physical examination findings include trismus, risus sardonicus, opisthotonos, and rigid abdominal muscles. In neonates, a foul-smelling and erythematous umbilical stump that was cut at birth using non-sterile technique is associated with C. tetani infection. The major virulence factor is tetanospasmin, which is a protease exotoxin that cleaves the soluble NSF- attachment protein receptor (SNARE) complex in Renshaw cells of the spinal cord, preventing the release of inhibitory GABA and glycine neurotransmitters by impairing the fusion of vesicles. It is carried from the site of inoculation to the spinal cord via retrograde axonal transport. Infection can be prevented by immunization with a tetanus toxoid vaccine. The tetanus toxoid is a modified exotoxin that is no longer virulent but still stimulates an antigenic response in the host. The antibodies produced after immunization target the exotoxin of C. tetani, disabling its function and preventing the onset of spastic paralysis. Incorrect Answers: A, B, D, E, and F. Capsular polysaccharide (Choice A) targets are used to make vaccines for pneumococcal disease, meningococcal disease, and Salmonella typhi. Clostridium tetani does not have a capsule. Cell wall polysaccharide (Choice B) is a bacterial virulence factor that forms a glycocalyx projecting from the bacterial cell wall. A glycocalyx protects bacteria from phagocytosis and aides in the adherence to other bacterial and inert materials. Fimbria (Choice D) is a virulence factor that aides in the adherence and colonization of bacteria to host cells. It is composed of glycoproteins and is a common virulence factor seen in Gram-negative bacteria,

176 ---------- Exam Section 4: Item 26 of 50 Natjonal, Board of Medical Examinersment 26. A 25-year-old primigravid woman at 12 weeks' gestation comes to the physician 1 hour after she passed a small amount of tissue vaginally at home. Gross examination of this tissue shows an obvious fetus. Microscopic examination of tissue obtained via dilatation and curettage shows chorionic villi with focal edema and trophoblastic proliferation. Which of the following is the most likely diagnosis? O A-Choriocarcinoma O B}Complete hydatidiform mele C) Partial hydatidiform mole O D) Placenta accreta O E} Placental-site trophoblastic tumor

C. Hydatidiform moles are either caused by the fertilization of an empty egg by a single sperm followed by duplication of the paternal DNA resulting in 46 chromosomes (complete) or the dual fertilization of a normal ovum resulting in 69 chromosomes (partial). Maternal symptoms include hyperemesis gravidarum or persistent nausea and vomiting related to abnormally increased human chorionic gonadotropin (hCG) levels. They can also cause first trimester vaginal bleeding, early preeclampsia, and uterine size greater than expected for dates. Complete moles contain no fetal tissue, while partial moles contain some fetal tissue, but neither are viable pregnancies and no fetal heartbeat will be found on Doppler evaluation. Pelvic examination may demonstrate cystic molar clusters in the vagina, and laboratory evaluation will reveal abnormally increased B-hCG levels, with greater increases associated with complete molar pregnancies. Ultrasound of the pelvis will typically show bilateral multilocular ovarian cysts (theca-lutein cysts) and echogenic structures in the uterus in the setting of a complete molar pregnancy but may only demonstrate fetal parts in a partial molar pregnancy. Molar pregnancies are treated with dilation and curettage, and histology will reveal hypertrophy of the chorionic villi with focal edema and trophoblastic hyperplasia, as seen in this patient. Incorrect Answers: A, B, D, and E. Choriocarcinoma (Choice A) can develop during or after a regular or molar pregnancy due to the malignant proliferation of trophoblastic tissue. It typically presents with marked increases in B-hCG and associated vaginal bleeding, as well as shortness of breath and hemoptysis due to a propensity for metastases to the lungs. Histologically, there will be proliferation of syncytiotrophoblast, cytotrophoblast, and intermediate troph

167 ---------- Exam Section 4: Item 17 of 50 National, Board pf Medical Examiners mont V 17. A 73-year-old man with polymicrobial sepsis begins treatment with a variety of antibiotics, including gentamicin (current dose indicated in table). Three days later, serum gentamicin concentrations are 8.8 ug/mL (peak) and 2.5 µg/mL (trough). Which of the following dose regimens is most appropriate for achieving a peak serum gentamicin concentration between 4 and 10 ug/mL and a trough concentration of less than 2 ug/mL? Dosage Intravenous Administration Time Between Each Dose (mg) (Current = 80 mg) (hours) (Current = 8 hours) O A) 60 B) 80 80 12 O D) 120 O E) 120 12

C. Increasing the time between doses is the most effective way to achieve a trough concentration of less than 2 µg/mL while maintaining a peak between 4 and 10 ug/mL. In this patient, gentamicin, an aminoglycoside antibiotic, is dosed intravenously for sepsis. Peak and trough concentrations are obtained 3 days into the course. The half-life of the medication can be estimated assuming steady state kinetics. At time = zero, the peak is 8.8 µg/mL; after one half-life, the concentration would be 4.4 ug/mL, and after two half-lives, the concentration would be 2.2 µg/mL. The trough is 2.5 ug/mL at 8 hours; therefore, the half-life of gentamicin in this patient is approximately 4 hours (half-life = hours/approximate number of half-lives passed). Steady state concentration generally requires four to five half-lives, which would have been reached on the first day of administration in this patient. The peak concentration of a drug in serum occurs immediately after bolus dosing, as the entire amount of administered drug is contained within serum before distribution to tissues. The trough concentration of a drug occurs immediately prior to the next dose and reflects the nadir of serum concentration, which is a function of redistribution into tissues plus drug metabolism. In this example, the peak concentration is 8.8 ug/mL, which is within the target range of 4 to 10 µg/mL. Increasing the dose would result in an increase in the peak concentration, while decreasing the dose would decrease the peak concentration. The dosed amount falls within the peak parameters and should not be changed. Trough concentration is a function of distribution plus metabolism. First-order metabolism results in a 50% reduction in drug concentration per half-life. From a peak of 8.8 ug/mL, one half-life (approximately 4 hours) would yield a serum conce

116 ---------- Exam Section 3: Item 16 of 50 Natjonal, Board of Medical Examinersment andive Rasic Science * 16. A 36-year-old man who is heterozygous for an LDL-receptor deficiency has a total serum cholesterol concentration of 330 mg/dL. After taking a drug that inhibits cholesterol synthesis, his total serum cholesterol concentration decreases to 200 mg/dL. Which of the following proteins would be upregulated as a result of drug therapy? O A) Cholesterol acyltransferase O B) Hepatic lipase O C) LDL receptor D) Lipoprotein lipase O E) Lysosomal cholesterol esterase

C. LDL receptors would be upregulated as a result of therapy with a statin medication. Statins inhibit the enzyme HMG-COA reductase, which catalyzes the rate limiting step in cholesterol biosynthesis, thereby resulting in reduced intrahepatic levels of cholesterol. Reduced intrahepatic cholesterol levels lead to increased expression of LDL receptors on the surface of hepatocytes with consequent reduction of circulating levels of LDL. As LDL is implicated in the pathogenesis of atherosclerosis, reducing circulating LDL levels has a positive impact on cardiovascular health. This patient with LDL-receptor deficiency due to a heterozygous mutation likely has familial hypercholesterolemia. The disorder is caused not only by mutations in the LDL receptor gene, but also by mutations of the PCSK9 or APOB genes. Defects in these genes lead to the decreased clearance of cholesterol by the liver and increased levels of cholesterol in the serum. Incorrect Answers: A, B, D, and E. Cholesterol acyltransferase (Choice A) is an enzyme that converts cholesterol to cholesteryl ester to be incorporated into HDL and LDL particles. Deficiency of this enzyme causes anemia, corneal opacities, and renal disease. Hepatic lipase (Choice B) is a hepatic enzyme that plays a role in the conversion of intermediate-density lipoprotein (IDL) to LDL. It does not affect LDL receptor expression directly. Lipoprotein lipase (LPL) (Choice D) exists primarily as an extracellular enzyme on the surface of vascular endothelial cells (not present in the liver) and acts to hydrolyze circulating triglycerides into their respective parts, monoacylglycerol and fatty acids, which are stored in adipose tissue. Its action is not affected by statin medications, but fibrate medications increase the activity of LPL and result in decreased serum triglyceride levels. Lyso

133 ---------- Exam Section 3: Item 33 of 50 National, Board pf Medical Examiners ensive Basi S ance ament 33. A 42-year-old man has an autoimmune disorder resulting in proximal muscle weakness of the lower extremities. Arrows in the photomicrograph shown indicate membranes that contain high concentrations of channels that are targeted by this condition. Which of the following is the most likely diagnosis? O A) Multiple sclerosis O B) Myasthenia gravis C) Myasthenic (Lambert-Eaton) syndrome D) Periodic paralysis O E) Tetany

C. Lambert-Eaton myasthenic syndrome is an autoimmune condition in which autoantibodies directed against voltage-gated calcium channels in the presynaptic neuron decrease the release of acetylcholine (ACh). Limited ACh release limits muscle cell depolarization and results in proximal muscle weakness. Additional symptoms include hyporeflexia, constipation, and autonomic dysregulation. With use, muscle strength improves due to the gradual accumulation of ACh in the synapse with repeated presynaptic depolarization. The diagnosis can be confirmed with testing for anti-voltage gated calcium channel antibodies. In many cases, Lambert-Eaton myasthenic syndrome is a paraneoplastic syndrome associated with small cell lung carcinoma. The electron micrograph demonstrates arrows pointing to the cell membrane of the presynaptic neuron terminal. The rounded structure enveloped by this membrane is known as the synaptic bouton. Within the bouton, preformed vesicles containing neurotransmitter are present. Directly distal to this, the undulating surface of the sarcolemma can be recognized. Incorrect Answers: A, B, D, and E. Multiple sclerosis (Choice A) is an autoimmune demyelinating disease that typically presents in Caucasian females in their fourth decade of life. It is characterized by central nervous system demyelination of white matter in both the brain and the spinal cord. This leads to weakness, hyperreflexia, and spasticity. Optic neuritis is a classic presenting diagnosis. Acute flares are treated with high-dose steroids. Myasthenia gravis (Choice B) is a disease similar to Lambert-Eaton myasthenic syndrome except autoantibodies are produced against ACh receptors of the neuromuscular junction. In myasthenia gravis, weakness becomes more profound with increased muscle use and bulbar weakness is common. Periodic paralysis (Choi

164 ---------- Exam Section 4: Item 14 of 50 National, Board pf Medical Examiners nent V 14. A 36-year-old man comes to the physician after he noticed a dark lesion on his back. Physical examination shows a dark, irregularly shaped, raised region on the back. He has a history of frequent sun exposure at the beach. A biopsy specimen of the region confirms the diagnosis of a melanoma. Which of the following embryonic tissues most likely formed this tumor? O A) Dermatome O B) Lateral plate mesoderm C) Neural crest O D) Nonneural ectoderm E) Prechordal plate mesoderm

C. Melanoma is a tumor of melanocytes, which are dendritic cells that reside in the basal layer of the epidermis and produce melanin that is distributed among the surrounding keratinocytes. Melanocytes are derived from the neural crest cells, which begin at the dorsal neural tube. During embryologic development they migrate from dorsal to ventral, and then to the epidermis. Melanocytes also migrate to the inner ear, nervous system, and eye. Failure of melanocytes to migrate leads to pigmentary abnormalities. For example, a white forelock, or a white patch of hair usually on the frontal hairline, occurs when the melanocytes do not fully migrate cephalad. Melanoma is a neoplastic proliferation of melanocytes and is likely to be present when a lesion demonstrates asymmetry, irregular appearing borders, variable coloration, a diameter greater than 6 mm, and rapid evolution in characteristics. Melanoma has the ability to rapidly invade and metastasize, which carries a poor prognosis when diagnosed late. Incorrect Answers: A, B, D, and E. A dermatome (Choice A) is one of the components of the embryonic somite, a paired segment of paraxial mesoderm. When fully developed, the dermatome is an area of skin whose afferent fibers travel within a single spinal nerve. Herpes zoster infection is often dermatomal because the virus replicates in the dorsal root ganglion of a single spinal nerve. Melanocytes do not follow dermatomes. The mesoderm is one of the three primary germ layers of the embryo. The mesoderm can be divided into three segments, moving medially to laterally: the paraxial mesoderm, the intermediate mesoderm, and the lateral plate mesoderm (Choice B). The prechordal plate mesoderm (Choice E) is located at the cephalic tip of the notochord. Langerhans cells, which are dendritic antigen-presenting cells also found in the

180 ---------- Exam Section 4: Item 30 of 50 Natjonal, Board of Medical Examinersment ansive Basic Saance S * 30. A 22-year-old man is brought to the emergency department 1 hour after being involved in a motor vehicle collision. Physical examination shows deep lacerations to the right lower extremity. Which of the following functional capabilities of the involved muscles will be decreased by fibrotic scar formation at the sites of injury? A) Formation of glycogen deposits B) Maintenance of prolonged tension O C) Passive stretching O D) Relaxation after contraction

C. Skeletal muscle cells (myocytes) convert energy stored as adenosine triphosphate into mechanical force via myosin motor fiber interaction with thin actin filaments. Myosin-actin interactions are regulated by troponins and calcium ions from the extracellular space and sarcoplasmic reticulum. Muscle injury and healing, particularly when muscle fibers are completely severed, is characterized by fibrosis and scarring within the muscle. Muscle fibers do not regenerate once destroyed. Areas of injured muscle are repaired by fibroblasts and myofibroblasts with collagen, forming a scar. Scarring of muscle with collagen will lead to decreased muscle movement as well as decreased range of motion of the nearby joints. In contrast to intact myofibrils with actin and myosin filaments that can slide past each other, collagen fibers are fixed in position and do not stretch or deform significantly. Muscle injuries, such as ischemia or crush, can lead to nearly complete replacement of muscle with scar tissue resulting in joint contractures, limited passive stretching, and limited mobility. Incorrect Answers: A, B, and D. Formation of glycogen deposits (Choice A) within a particular myocyte is not affected by the presence of scar tissue. Glycogen is a form of carbohydrate energy storage. Glycogen is synthesized during states of energy availability in both the liver and in skeletal myocytes. Glycogen stores are utilized during starvation states or high-energy exercise. Maintenance of prolonged tension (Choice B) is commonly needed in tissues that are comprised of smooth muscle that have basal tonic contraction. For example, the smooth muscle of the vasculature is required to have a basal force output. Myosin cross-bridges known as latch-bridges cycle at a slow rate which facilitates reduced energy requirements with sustained contracti

137 ---------- Exam Section 3: Item 37 of 50 National, Board pf Medical Examiners ancive Rasi Sciance nent * 37. An investigator is studying Helicobacter pylori strains isolated from the same patient over several years. The immune response to proteins produced by these strains is observed. It is found that the original H. pylori isolate from the patient expresses one protein recognized by the patient's antibodies, but subsequent isolates do not express this protein. Sequencing of the gene encoding the protein from the original and subsequent isolates is done. It is found that subsequent isolates have nine consecutive cytidine residues, whereas the original isolate has eight. The results are shown. Original isolate: AGC AСc ССС ССС АСТ САА ATT GAA CСТ ... ... Ser Pro Pro THr Gln lle Glu Pro ... Thr Subsequent isolates: Aсс ССС ССС САС ТСА AAТ TGA AСС ТAG Pro Pro His Ser Asn STOP Thr Wh

C. Slipped-strand mispairing is a type of mutation that may occur during DNA replication. In an area of repetitive nucleotides, such as the series of eight cytidine residues in this case, DNA polymerase slips and either inserts one or more additional nucleotides or removes one or more additional nucleotides by mistake. Here, the addition of one cytidine residue has led to a frameshift mutation in which the base pair reading frame has been shifted over by one, leading to a completely different amino acid sequence during translation and an early stop codon. In this case, the frameshift mutation caused by slipped-strand mispairing has led to a dysfunctional, truncated protein. If a slipped-strand mutation led to the insertion or deletion of nucleotides in groups of three, it would cause the addition or omission of amino acids to the original sequence as the reading frame would be preserved. Šince antibody recognition of proteins is highly specific, this truncated protein seen in this case does not bind to the same antibody that the wild-type protein previously did. Incorrect Answers: A, B, D, and E. Crossing over (Choice A) is part of the recombination process, which takes place in meiosis. It is the process by which two chromatids exchange a small portion of genetic material and separate, or unlink, two traits previously present on the same chromatid. The total genetic material on each chromosome remains the same, so it would not explain the addition of a nucleotide base pair. DNA excision repair (Choice B) is a proofreading process by which mutated nucleotides (nucleotide excision repair) or bases (base excision repair) are removed. This process leads to the removal of nucleotides rather than the insertion. Thymidine dimer formation (Choice D) is a mutation caused by ultraviolet radiation, which leads to covalent cross

191 ---------- Exam Section 4: Item 41 of 50 Natjonal, Board of Medical Examinersment ansive Basic Saance V 41. An 8-year-old girl is brought to the physician by her mother 1 hour after her cat scratched her on her right arm. The mother says that the patient developed bright red streaks along her arm within minutes of being scratched. The scratches were painful, and the patient was crying when she showed the injury to her mother. Examination of the right forearm shows the surface linear streaks with intact skin, but there is no blood. The appearance of the red streaks most likely indicated which of the following host responses to this injury? O A) Bacteria-generated capillary congestion O B) Complement-activated coagulation C) Histamine-stimulated vasodilation O D} IgE-mediated-eosinophil degranulation O E) Trauma-induced arteriolar hemorrhage

C. The body responds to harmful stimuli such as infectious agents, mechanical disruption, and chemical irritants with a localized inflammatory response mediated by the innate immune system. The cardinal signs of inflammation are rubor (redness), calor (heat), tumor (swelling), dolor (pain), and functio laesa (loss of fund vasodilation of the local arterioles. This leads to increased blood flow in the affected region, which generates the classic rubor of inflammation. Histamine also promotes increased vascular permeability, which allows for the leakage of plasma into the interstitial tissue resulting in local edema (tumor of inflammation) followed by recruitment and migration of immune cells. n). The first step in response to an injury of vascularized tissue is transient reflexive vasoconstriction followed by the release of histamine by mast cells and basophils, which leads to histamine-stimulated Incorrect Answers: A, B, D, and E. Bacteria-generated capillary congestion (Choice A) occurs in sepsis due to the vasodilatory and inflammatory effects of bacterial cell components such as lipopolysaccharide. This patient presents immediately following an injury with the classic signs of the initial inflammatory response, not sepsis. Complement-activated coagulation (Choice B) results from signaling between coagulation factors and the complement system to achieve hemostasis in areas of localized tissue damage. IgE-mediated eosinophil degranulation (Choice D) occurs in response to parasitic infections and IgE-mediated allergic reactions. It is not part of the initial inflammatory response to injury. Trauma-induced arteriolar hemorrhage (Choice E) would result in an ecchymosis in the area of injury due to the mechanical disruption of the arterioles and localized bleeding. Educational Objective: The innate immune system conducts

160 ---------- Exam Section 4: Item 10 of 50 Natjonal, Board of Medical Examinersment ancive Rasic Saance * 10. In a 3-week-old knockout mouse, a leg bone reaches only half the length of that of a similar-aged wild-type mouse. The epiphyseal plates in the bone of the transgenic animal are significantly narrower than in the bone of the wild-type animal. The knockout mouse most likely has a deficiency of which of the following? A) Calcitonin O B) Cortisol O C) Insulin-like growth factor I D) Platelet-derived growth factor O E) Thyroxine (T4)

C. The hypothalamus-pituitary-somatotropic axis is the endocrine regulatory feedback loop that is responsible for human growth regulation. In this axis, the pituitary gland secretes growth hormone (GH, somatotropin) which stimulates insulin-like growth factor-1 (IGF-1, somatomedin C) production in the liver and peripheral tissues. IGF-1 binds its receptor, a tyrosine kinase-based receptor, that stimulates cell growth, proliferation, and growth of the axial and appendicular skeleton. Deficiency of the GH receptor (Laron syndrome) leads to short stature from insensitivity to GH. Conversely, a GH-secreting pituitary adenoma can cause gigantism or acromegaly through excessive, dysregulated stimulation of GH receptors, leading to chronic elevations in IGF-1. Gigantism results from excess IGF-1 signaling before fusion of the growth plates, and results in enlargement of both long and flat bones, whereas acromegaly occurs after fusion, leading only to expansion of flat bones and tissues. Incorrect Answers: A, B, D, and E. Calcitonin (Choice A) is a peptide hormone produced by the parafollicular cells of the thyroid gland. Calcitonin is a regulator of calcium homeostasis and its primary function is to reduce blood levels of calcium. It inhibits osteoclast activity and also inhibits renal tubular resorption of calcium and phosphate. Cortisol (Choice B) is a steroid hormone which is produced in the zona fasciculata of the adrenal cortex. Cortisol production is cyclic with the human sleep-wake cycle and is a key regulator of glucose metabolism with higher levels of cortisol leading to gluconeogenesis. Ít also has important effects on the central nervous system relating to stress and mood regulation, and it is a powerful anti-inflammatory molecule as it inhibits production of tumor necrosis factor alpha. Platelet-derived growth fa

146 ---------- Exam Section 3: Item 46 of 50 National Board of Medical Examiners adive Rasic Saance ent 46. A 35-year-old woman comes to the emergency department 3 hours after the sudden onset of severe right-sided back pain. She has a history of recurrent urinary tract infections. Her most recent infection was 1 month ago; she was successfully treated for Klebsiella pneumoniae with antibiotics. The patient is not taking any medications. She says that she has been eating more fast food and drinking less water during the past month because of a busy schedule. She is sexually active and uses a spermicidal cream for contraception. Her temperature is 37°C (98.6°F), pulse is 100/min, respirations are 14/min, and blood pressure is 138/66 mm Hg. Physical examination shows marked tenderness to palpation over the right flank. Urinalysis shows: pH Protein 8 1+ 30- RBC 35/hpf 20- 25/hpf WBC An excess of which of the following

C. This patient's presenting findings of acute back pain, tachycardia, hematuria, pyuria, proteinuria, and mild alkalization of urine are suggestive of nephrolithiasis with a magnesium ammonium phosphate calculus. Magnesium ammonium phosphate, also known as struvite, is a less common cause of nephrolithiasis than calcium oxalate. Struvite calculi occur in the setting of infections caused by urease-positive microbial pathogens, such as Klebsiella or Proteus. Urease catalyzes the conversion of urinary urea to ammonia which results in urine alkalinization. The solubility of struvite is decreased under alkaline conditions and precipitates as a crystal. Struvite crystals have an orthorhombic configuration under light microscopy. Struvite calculi are radiopaque and can be potentially identified by a radiograph or CT scan. Struvite calculi may adopt a branching morphology that molds itself to the shape of the collecting system. Given their large size and often ramified structure, it is rare for struvite calculi to pass through the urinary tract and surgical removal is often necessary, along with treatment of the underlying infection. Incorrect Answers: A, B, D, and E. Calcium oxalate (Choice A) is the most common etiology of nephrolithiasis. Citric acid reduces the formation of calcium oxalate crystals by forming soluble complexes with calcium ions, and therefore, calcium oxalate calculi tend to form in the setting of hypocitraturia. Calcium oxalate crystals have an octahedral morphology under light microscopy and form radiopaque calculi. Patients can be treated with thiazide diuretics, a low-sodium diet, and supplementation with potassium citrate (Choice D), which increases the urinary concentration of citric acid. Cystine (Choice B) precipitates as a urinary calculus in the setting of increased urinary cystine concentration

143 ---------- Exam Section 3: Item 43 of 50 Natjonal, Board of Medical Examinersment 43. A 28-year-old woman comes to the physician because of a 4-day history of palpitations, severe neck pain, fatigue, and malaise. Her pulse is 120/min and regular. Physical examination shows a diffusely tender, mildly enlarged thyroid gland. There is no exophthalmos. Serum studies show a thyroid-stimulating hormone concentration of 0.01 pU/mL. Which of the following is the most likely diagnosis? O A) Factitious thyrotoxicosis O B}Graves disease C) Subacute granulomatous thyroiditis O Đ) Thyroid abscess O ETexie multinodular geiter

C. This patient's presenting findings of subacute fatigue, malaise, neck pain, palpitations, and an enlarged and tender thyroid are highly suggestive of subacute granulomatous thyroiditis, also known as subacute (de Quervain) thyroiditis. Subacute granulomatous thyroiditis is a self-limited inflammatory condition of the thyroid gland that often follows an acute viral illness. Presenting findings are typically suggestive of clinical hyperthyroidism and may include diaphoresis, palpitations, neck pain, dysphagia, fever, tachycardia, and a decreased serum concentration of thyroid-stimulating hormone (TSH). Patients may eventually become hypothyroid or euthyroid following the acute inflammatory, hyperthyroid phase. Subacute granulomatous thyroiditis is distinguished from other forms of hyperthyroidism by the presence of a painful, tender thyroid, which is highly suggestive of the diagnosis. It is also characterized by the absence of iodine uptake on a radionuclide scan, and by the presence of multinucleated giant cells on fine needle aspiration. Incorrect Answers: A, B, D, and E. Factitious thyrotoxicosis (Choice A) is due to the deliberate ingestion of excessive quantities of thyroid hormone, such as levothyroxine. Factitious thyrotoxicosis can be distinguished from subacute granulomatous thyroiditis and Graves disease by the absence of suggestive clinical findings, including goiter, thyroid swelling, pain or tenderness, orbital involvement, or pretibial myxedema. Graves Disease (Choice B) is the most common cause of hyperthyroidism and is caused by an autoantibody that activates TSH receptors on the thyroid. It presents with symptoms of hyperthyroidism, pretibial myxedema, and thyroid ophthalmopathy, which can cause diplopia, proptosis, and restrictive strabismus. The thyroid is not typically painful or tender. Thyroid a

178 ---------- nsive of Medical Examinersemant Exam Section 4: Item 28 of 50 Natjonal, Board V 28. A 52-year-old man comes to the physician because of a 5-day history of headaches and muscle weakness. His blood pressure is 175/95 mm Hg. Physical examination shows no other abnormalities. Laboratory studies show: Serum 142 mEq/L 2 mEq/L 33 mEq/L 1.4 mg/dL <2 ug/dL Na+ K+ HCO3- Creatinine Cortisol Arterial blood gas analysis on room air: pH Рсо2 7.45 48 mm Hg Plasma renin activity is undetectable. Which of the following is the most likely cause of this patient's hypertension? O AJAddisen disease O B} Chronic kidney disease O G}Gushing disease O D) Hyperaldosteronism O E) Pheochromocytoma

D. Aldosterone is produced by the zona glomerulosa of the adrenal cortex and plays an important role in the maintenance of blood volume and sodium homeostasis, primarily through the renin-angiotensin-aldosterone pathway. Renin is produced by the juxtaglomerular cells of the kidney as a response to decreased blood flow, decreased sodium delivery to the macula densa, or increased sympathetic tone. Renin converts angiotensinogen to angiotensin I, which is converted to angiotensin II by angiotensin-converting enzyme (ACE). Angiotensin Il subsequently signals the adrenal gland to secrete aldosterone. Aldosterone promotes the reabsorption of sodium ions in exchange for potassium and hydrogen ion secretion in the distal convoluted tubule and collecting ducts, causing the indirect reabsorption of water, which results in the development of hypertension. High levels of aldosterone provide negative feedback to renin, decreasing its production. Primary hyperaldosteronism presents with hypertension, metabolic alkalosis, hypokalemia, and an increased aldosterone-to-renin ratio due to the independent secretion of aldosterone, most commonly from an adrenal adenoma or bilateral adrenal hyperplasia. Incorrect Answers: A, B, C, and E. Addison disease (Choice A) or primary adrenal insufficiency is most commonly due to autoimmune destruction or tuberculosis. It presents with hypotension, hyperkalemia, metabolic acidosis, and skin hyperpigmentation secondary to a deficiency of aldosterone and cortisol. It does not describe this patient's clinical condition. Chronic kidney disease (Choice B) presents with elevated creatinine, as well as hyperkalemia due to the inability to secrete potassium, and metabolic acidosis due to the inability to reabsorb bicarbonate. It would not cause hypokalemia and metabolic alkalosis, as in this patient. Cushing

5 ---------- Exam Section 1: Item 6 of 50 National Board of Medical Examiners Comprehensive Basic Science Self-Assessment 6. A 62-year-old man is being evaluated for rectal bleeding. An x-ray of the gastrointestinal tract with contrast material is shown. Which of the following is the most likely explanation for the feathery appearance in the portion of the gastrointestinal tract indicated by X when compared with the portion indicated by Y? O A) Absence of circular muscle B) Fewer villi C) Greater bowel motility D) Greater mucosal surface area E) Less circular and longitudinal smooth muscle

D. The jejunum (X) can be distinguished from the ileum (Y) by several characteristic features. The inner mucosal folds, or plicae circulares, are more prominent, more numerous, and taller in the jejunum compared to the ileum. This leads to an increased mucosal surface area and an associated feathered appearance after the administration of oral contrast material. The jejunum also demonstrates a larger caliber, thicker muscular walls, longer vasa rectae, and fewer arcades. The ileum is notable for the presence of Peyer patches and mucosal lymphoid follicles, which can be identified histologically. Incorrect Answers: A, B, C, and E. Absence of circular muscle (Choice A) is not a feature of either the jejunum or the ileum. The muscularis externa, which is organized into circular and longitudinal layers, extends through all segments of the small intestine. Fewer villi (Choice B) is not a feature of the jejunum. Both the jejunum and the ileum contain villi. However, the plicae circulares are much more prominent and numerous in the jejunum, leading to an increased overall surface area and an associated increased number of mucosal villi compared to the ileum. Greater bowel motility (Choice C) is not a distinguishing characteristic of the jejunum as compared to the ileum. Bowel motility is controlled by peristaltic contractions and migrating motor complexes that are under the control of the enteric and central nervous systems. Less circular and longitudinal smooth muscle (Choice E) is a feature of the ileum (Y) rather than the jejunum (X). The jejunum has a thicker muscular wall in comparison to the ileum, but its feathered appearance with oral contrast material is primarily due to its prominent plicae circulares and increased mucosal surface area. Educational Objective: The jejunum possesses several characteristic features, in

98 ---------- Exam Section 2: Item 48 of 50 Natjonal, Board of Medical Examinersmen' 48. A 65-year-old man who is a veteran comes to the office because of a 3-month history of increasingly severe pain of his low back. He has no history of major medical illness. Physical examination shows point tenderness over the spine at the level of L5. Laboratory studies show: 9.8 g/dL Hemoglobin Hematocrit 28% Serum 11.2 mg/dL 2.2 mg/dL 8000 mg/dL (N=650- 1500) Ca2+ Creatinine IgG X-rays of the spine show a compression fracture at L5 and four lytic lesions in the iliac crest. Examination of a bone marrow biopsy specimen confirms the diagnosis and the presence of a chromosomal translocation. Treatment with a chemotherapeutic regimen, including a proteasome inhibitor, is started. Which of the following is most likely to occur in the affected cells as a result of this therapy? O A) Decreased calcium release O B) Decreased genomic in

E. This patient's presenting findings of bone pain, anemia, hypercalcemia, renal dysfunction, hypergammaglobulinemia, and lytic bone lesions are consistent with a diagnosis of multiple myeloma. Multiple myeloma is a malignancy caused by the neoplastic proliferation of plasma cells. Neoplastic plasma cells overproduce monoclonal immunoglobulin and light or heavy chains, which may result in acute renal failure. Patients also commonly present with constitutional symptoms of fatigue and weight loss, and hepatosplenomegaly, with symptoms of hypercalcemia (eg, abdominal cramping, kidney stones, psychiatric disturbance), symptoms of anemia (eg, pallor, light-headedness, dyspnea on exertion), or with opportunistic infections secondary to immune dysfunction. Further workup is recommended with complete blood count, complete metabolic panel, urinalysis, urine and serum protein electrophoresis, and imaging to determine the extent of malignancy. The diagnosis is confirmed with bone marrow biopsy. As with many hematologic and immunoproliferative malignancies, the initiation of chemotherapy results in the rapid induction of apoptosis in malignant cells. Proteasome inhibitors, such as bortezomib, result in impaired proteasomal activity, especially in myeloma cells. Due to the increased production of proteins by plasma cells, they depend on the ability to degrade unfolded or misfolded proteins to limit the stresses placed on the cell by such, which is the primary physiologic function of the proteasome. When the cells are unable to clear misfolded or abnormal proteins, the induction of apoptosis occurs due to the response to unfolded protein. Patients initiating chemotherapy should be monitored for signs of tumor lysis syndrome, including hyperkalemia, hyperphosphatemia, hypocalcemia, and hyperuricemia. Incorrect Answers: A, B, C, and D

30 ---------- Exam Section 1: Item 31 of 50 National Board of Medical Examiners Comprehensive Basic Science Self-Assessment 31. A 45-year-old man comes to the physician because of a 2-month history of a dull pain on the left side of his scrotum. Examination of the left side of the scrotum shows a bluish appearance of the skin and a palpable mass that feels like a bag of worms. Which of the following veins is most likely obstructed in this patient? A) Inferior mesenteric vein B) Inferior vena cava C) Inferior vesical vein D) Internal iliac vein E) Left renal vein

E. A varicocele refers to dilated veins of the pampiniform plexus due to elevated venous pressure. Varicoceles are commonly located on the left side due to increased resistance to venous flow as the left gonadal vein drains into the left renal vein. On the right side, the right gonadal vein drains directly into the inferior vena cava and is less likely to become obstructed. An obstruction of the left renal vein, such as may be caused by local invasion of a renal tumor, causes pressure to mount behind the obstruction which is transmitted through the left gonadal vein to the pampiniform plexus surrounding the testicle. The dilation of the pampiniform plexus causes a bluish appearance of the overlying skin. Palpation of the dilated plexus demonstrates varicosities, which are often described as having a vermiform (worm-like) feeling. Unlike a hydrocele, which is caused by failure of the processus vaginalis to close leading to accumulation of fluid around the testicle, a varicocele will not transilluminate with application of light to the scrotum. Infertility is a potential complication of varicoceles, as the dilated veins result in increased temperature of the testicle, which impairs spermatogenesis. Treatment for varicoceles may require surgical embolization or ligation to manage infertility. Incorrect Answers: A, B, C, and D. The inferior mesenteric vein (Choice A) drains the large intestine into the splenic vein, which in turn merges with the superior mesenteric vein to form the hepatic portal vein. Obstruction of the inferior mesenteric vein would cause venous congestion of the large intestine and superior rectum, not the pampiniform plexus in the scrotum. The inferior vena cava (Choice B) is the large vessel formed by the merging of the bilateral common iliac veins which carries blood from the abdomen, pelvis, and low

127 ---------- Exam Section 3: Item 27 of 50 Natjonal, Board pf Medical Examiners ansive Basic Saance ement V 27. An investigational drug (Drug X) under development for the treatment of hypertension is being tested in phase 1 clinical trials. A significant number of subjects develop dose-dependent increases in serum AST and ALT activities. The investigators hypothesize that this effect is due to oxidative stress. Assuming that biopsy samples of the liver can be obtained from the subjects, which of the following would be the best measure of hepatic oxidative stress caused by Drug X? O A) Concentration of mRNA for C-reactive protein O B) Detection of nitrated proteins in hepatic cell membranes O C) Microsomal cytochrome P450 concentration O D) Rate of state 3 mitochondrial respiration E) Ratio of GSH:GSSG

E. Glutathione is a primary hepatic antioxidant. Reactive oxygen species, such as superoxide and hydroxyl radicals, possess unpaired electrons (free radicals) that damage tissues through oxidation reactions. Glutathione contains a sulfur atom that is capable of reducing reactive oxygen species and free radicals, neutralizing these molecules before they can cause DNA strand breaks, lipid peroxidation, and protein damage. Glutathione exists primarily in a reduced form, with a hydrogenated sulfur group (GSH). Two molecules of GSH can be oxidized in order to form glutathione disulfide, GSSG. The ratio of GSH to GSSG therefore reflects the degree of oxidation of the liver's supply of available, reduced glutathione. Hepatic metabolism of drugs that produce oxidative stress through the production of free radicals will deplete hepatic stores of GSH, leading to a decreased ratio of GSH to GSSG. Incorrect Answers: A, B, C, and D. Concentration of MRNA for C-reactive protein (Choice A) is expected to be altered during inflammatory states. C-reactive protein is an acute phase reactant that is produced in states of inflammation and infection by the liver in response to interleukin-6. Detection of nitrated proteins in hepatic cell membranes (Choice B) is also a result of hepatic oxidative stress due to production of peroxynitrite. However, nitrated cell membrane proteins are only one type of oxidation product, whereas glutathione is involved ubiquitously in the neutralization of free radicals and reactive oxygen species, making it a more sensitive marker. Microsomal cytochrome P450 concentration (Choice C) will be altered by factors that induce P450, such as the use of alcohol, phenytoin, or rifampin. Microsomes are laboratory artifacts composed of fragments of endoplasmic reticulum and are not present in vivo. Rate of state 3 mitoc

195 ---------- Exam Section 4: Item 45 of 50 National Board of Medical Examiners mont V 45. A 24-year-old man who is serving overseas in the US Armed Forces is brought to a medical unit 30 minutes after sustaining multiple injuries caused by an explosion that forcibly ejected him from the vehicle he was driving. On arrival, he is conscious and hemodynamically stable. His respirations are 30/min. Physical examination shows visible shrapnel protruding from the lower sternum. Which of the following structures within the pericardial sac is most likely damaged in this patient? O A) Ascending aorta O B) Left atrium O C) Left ventricle O D) Right atrium E) Right ventricle

E. In the typical anatomical orientation, the right ventricle is the most anterior part of the heart. It also forms the inferior (or diaphragmatic) surface of the heart along with the left ventricle. It lies just posterior to the sternum and anterior mediastinum. A penetrating object entering the inferior sternum would affect the most superficial, anterior structure of the heart in this patient, which is the right ventricle. Incorrect Answers: A, B, C, and D. The ascending aorta (Choice A) originates at the left ventricular outflow tract and courses superiorly to the level of the angle of louis before turning posterior and lateral to form the aortic arch. It is located distant from the inferior sternum. The left atrium (Choice B) is the most posterior part of the heart and lies anterior to the esophagus in the mediastinum. It is less likely to be damaged by a penetrating object because it is deeper in the thoracic cavity, and a penetrating object would have to pass through multiple anterior structures to reach it. The left ventricle (Choice C) is located posterior to the right ventricle, and inferolateral to the left atrium. It abuts the left lung and makes up the majority of the left border of the heart. The right atrium (Choice D) abuts the right lung and makes up the right heart border. It is located posterior to and to the right of the sternum, whereas the right ventricle holds an anterior and inferior position, directly behind the sternum and is thus more susceptible to penetrating trauma. Educational Objective: The right ventricle is the most anterior and inferior part of the heart and lies just posterior to the sternum. A penetrating object through the inferior sternum would affect the most superficial, anterior structure of the heart, which is the right ventricle.

135 ---------- Exam Section 3: Item 35 of 50 Natjonal, Board of Medical Examinersment * 35. An 11-year-old boy receives succinylcholine and sevoflurane for anesthesia during surgical repair of a hernia. During the procedure, the patient develops muscle rigidity, tachycardia, hyperkalemia, hypocalcemia, and hyperthermia. This patient most likely carries a variant form of the gene coding for which of the following? A) N-Acetyltransferase B) Cytochrome P450 2D6 O C) Cytochrome P450 3A4 O Đ}Glutathione-synthetase E) Ryanodine receptor

E. Malignant hyperthermia (MH) is caused by an autosomal dominant genetic defect in the ryanodine receptor of muscle cells, leading to increased calcium release from the sarcoplasmic reticulum with exposure to triggering agents (eg, succinylcholine, volatile anesthetics). This leads to sustained muscle contraction with increased oxygen demand and carbon dioxide production. Physical examination and vital signs will demonstrate muscle contractures (eg, masseter muscle rigidity), fever, and tachycardia, while laboratory evaluation will show increases in CO2 and lactic acidosis. Risk factors include a family history of malignant hyperthermia, being born in the upper Midwest of the United States, and certain musculoskeletal disorders, such as multi-minicore myopathy. Treatment is with dantrolene, which acts as a ryanodine receptor antagonist, as well as external cooling and correction of electrolyte and acid-base abnormalities. Mortality is high, ranging from 5% to 30% with treatment. Incorrect Answers: A, B, C, and D. N-Acetyltransferase (NAT) (Choice A) transfers acetyl groups from acetyl-CoA to proteins, including serotonin, and is important in phase II hepatic metabolism of drugs. Polymorphisms in the genes that encode the NAT enzymes cause differences in speed of metabolism of medications between individuals but do not cause MH. Cytochrome P450 2D6 (Choice B) plays an important role in the metabolism and activation of many medications through hydroxylation, demethylation, and dealkylation. Variant genes lead to rapid elimination of medications or slower than normal elimination, but they do not cause MH. Cytochrome P450 3A4 (Choice C) plays an important role in the metabolism and activation of many medications through oxidation and hydroxylation. Most variant genes do not cause significant differences in elimination bet

99 ---------- Exam Section 2: Item 49 of 50 National, Board pf Medical Examiners nent * 49. A 58-year-old woman comes to the office for a follow-up examination 2 days after she noticed a lump in her left breast on self-examination. Menopause occurred 6 years ago. Physical examination shows a 1-cm, poorly circumscribed, nodular mass in the left periareolar tissue. Mammography shows an ill-defined density suspicious for malignancy. A needle biopsy specimen is nondiagnostic. Microscopic examination of the excised mass shows a large dilated space lined by glandular epithelium and surrounded by a dense lymphoplasmacytic inflammatory infiltrate; the associated lumen is filled with lipid-laden macrophages and amorphous debris. Which of the following is the most likely diagnosis? O A) Acute mastitis B) Fibroadenoma O C) Fibrocystic changes O D) Intraductal carcinoma O E) Mammary duct ectasia

E. Mammary duct ectasia is caused by widening of a lactiferous duct with concomitant fibrosis and inflammation, often near the nipple. It frequently occurs in perimenopausal and postmenopausal women. When ectatic ducts become blocked, they may become nodular and can be felt as a mass, as in this patic examination, mammary duct ectasia is marked by a dilated duct lined with glandular epithelium and surrounded by inflammatory cells. Lipid-laden macrophages result from lipid penetrating the duct wall and being engulfed by macrophages. Lipid is usually contained within the duct. Duct ectasia is not inherently malignant, nor does it increase a patient's risk for developing breast carcinoma. However, the presence of a malignancy may cause the original blockage to form and thus mammary duct ectasia could be an initial presentation of malignancy. When this is not the case, duct ectasia typically resolves with conservative management. Other symptoms include nipple dis arge, nipple retraction, and pain. On mammography, subareolar branching densities can be mistaken for spiculated neoplasms and calcifications may be present. On histopathologic Incorrect Answers: A, B, C, and D. Acute mastitis (Choice A) presents with erythema and induration of the breast in an area of tender, warm skin. It classically occurs in breastfeeding mothers as a consequence of microtrauma to the nipple leading to inoculation from cutaneous or oral flora. Staphylococcus aureus is a common pathogen. It is uncommon in postmenopausal women and does not classically present with a breast lump. Fibroadenoma (Choice B) presents as a unilateral, nontender, mobile, rubbery mass in the breast, often in a younger woman. Ultrasound or mammography demonstrates a well-circumscribed, round tumor and needle biopsy is often unnecessary. Fibrocystic changes of the breast (

171 ---------- Exam Section 4: Item 21 of 50 Natjonal, Board pf Medical Examinersment * 21. A 35-year-old woman with sickle cell trait is brought to the emergency department because of severe left flank pain for 1 hour. She has a history of frequent headaches. Her blood pressure is 150/90 mm Hg. Pressure on the costophrenic angle causes pain. Urinalysis shows a few RBCS and numerous neutrophils. A CT scan of the abdomen shows obstruction and dilation of the left ureter and blunting of several renal papillary pyramids. Which of the following is the most likely diagnosis? O AJAcute glomerulonephritis O B}Acute tubular necrosis O C) Diabetic nephropathy O D) Papillary transitional cell carcinoma E) Renal papillary necrosis

E. Renal papillary necrosis (RPN) occurs following ischemic, inflammatory, infectious, or toxin-mediated damage to the renal papilla and describes the sloughing and loss of the papillae including substructures such as the distal collecting tubule. RPN can be triggered by infections (eg, acute pyelonephritis), diabetes, sickle cell disease, or nonsteroidal anti-inflammatory drugs (NSAIDS). In sickle cell disease, renal papillary necrosis is common and occurs secondary to renal medullary ischemia and infarction. It typically presents with hematuria and acute flank pain. It can also be complicated by urinary tract obstruction and/or infection secondary to necrotic tissue sloughing into the renal collecting system and ureters. It characteristically presents with gross hematuria and proteinuria on urinalysis. Incorrect Answers: A, B, C, and D. Acute glomerulonephritis (Choice A) refers to a variety of glomerular diseases, including nephritic and nephrotic syndromes. Nephritic syndromes typically present with acute renal failure with associated hematuria, red blood cell urine casts, and hypertension. Nephrotic syndrome typicàlly presents with excessive proteinuria (greater than 3 g/day) hyperlipidemia, hypoalbuminemia, and edema. Ít would not typically cause ureteral obstruction. Acute tubular necrosis (Choice B) typically occurs following an ischemic or nephrotoxic insult to the kidneys, which results in necrosis of the tubular epithelium. Granular, muddy brown casts are typical on urinalysis. It would be less likely to cause ureteral obstruction. Diabetic nephropathy (Choice C) is a chronic process that occurs following nonenzymatic glycosylation of the glomerular basement membrane and efferent arterioles, characteristically presenting as Kimmelstiel-Wilson lesions on light microscopy. It progresses over time in patients

147 ---------- Exam Section 3: Item 47 of 50 National, Board pf Medical Examiners ment 47. A 68-year-old woman is brought to the emergency department 3 hours after falling onto her right wrist while working in her yard. Examination of the right wrist shows swelling and ecchymoses. Passive flexion of the right wrist exacerbates the pain. An x-ray of the right wrist is shown. Which of the following is the most likely cause of these findings? O A) Avascular necrosis of the scaphoid O B) Avulsion of the ulnar styloid process O C) Dislocation of the first proximal phalanx O D) Disruption of the radioulnar joint E) Fracture of the distal radius

E. The x-ray demonstrates a fracture of the distal radius. In this fracture, the carpal bones remain attached to the distal radius fragment by the radioscaphocapitate ligament and accessory carpal ligaments that maintain position of the carpal bones on the distal radius. Generally, there are two common fracture patterns involving the distal radius. A volar displacement of the distal fragment is termed a Smith fracture, which occurs when axially loading a flexed wrist, while a Colles fracture is more common and results from axially loading the wrist in extension. Colles fractures exhibit a dorsally displaced distal radius fragment. Both fracture patterns are common in older persons, especially those predisposed to osteoporosis (eg, women) as in this patient. Classic presentations include pain in the affected wrist, swelling, ecchymosis, and limited passive and active range of motion. These fractures can cause acute median nerve entrapment. In the long-term, abrasion, attenuation, and rupture of extensor tendons that glide over the fracture site may occur. Incorrect Answers: A, B, and D. Avascular necrosis of the scaphoid (Choice A) can occur in the setting of a displaced scaphoid fracture that fails to heal properly. The blood supply for the scaphoid is oriented in a distal to proximal direction. If the fracture extends through the scaphoid such that the proximal pole is separate from the distal pole, the proximal pole will not have sufficient blood supply and may subsequently necrose. Avulsion of the ulnar styloid process (Choice B) can sometimes occur along with distal radius fractures. This injury usually does not cause secondary complications; however, it can be associated with disruption of the ligaments that form the distal radioulnar joint. Dislocation of the first proximal phalanx (Choice C) is a common injury o

100 ---------- Exam Section 2: Item 50 of 50 Natjonal, Board pf Medical Examinersment * 50. An 1814-g (4-lb) female newborn is delivered in the hospital at 24 weeks' gestation to a 25-year-old primigravid woman via a spontaneous vaginal delivery. Examination of the newborn shows no peripheral cyanosis. There is a grade 3/6, continuous murmur over the left midclavicular, second intercostal space. At the age of 2 days, the newborn develops respiratory distress. Echocardiography shows a failure of a vascular structure to close. The embryologic origin of this anomaly is most likely which of the following aortic arches? O A) First O B) Second O C) Fourth D) Fifth O E) Sixth

E. This patient has a moderate to large patent ductus arteriosus (PDA) as evidenced by a continuous murmur and respiratory distress. In utero, oxygenated blood is obtained from the placenta, not from the lungs, and the ductus arteriosus (DA) allows the shunting of oxygenated blood from the pulmonary called a PDA, and if large enough, can lead to a profound left-to-right shunt, with consequent heart failure, respiratory distress, and pulmonary hypertension. PDAS that are small often go undetected until adulthood because the left-to-right shunt is not significant enough to cause symptoms. The DA arises from the sixth aortic arch, which also gives rise to the proximal right pulmonary artery. ectly to the aorta while bypassing the fetal lungs, which are filled with amniotic fluid. At birth, closure of the DA occurs by the release of prostaglandins and increased arterial oxygen tension. A DA that fails to close is Incorrect Answers: A, B, C, and D. First (Choice A) and second (Choice B) aortic arch structures include the maxillary artery, a part of the external carotid artery, the ascending pharyngeal artery, and the stapedial artery. Fourth (Choice C) aortic arch structures include the right subclavian artery and the aortic arch from the origin of the left carotid artery to the beginning of the DA. Fifth (Choice D) aortic arch structures regress and do not contribute to adult cardiovascular structures. Educational Objective: The sixth aortic arch gives rise to the ductus arteriosus, a structure that allows shunting of placental oxygenated blood from the pulmonary artery to the systemic circulation during development in utero but closes shortly after birth. Persistence of this structure is called a patent ductus arteriosus (PDA). Symptoms are dependent upon the size of the PDA and the degree of left-to-right shunting, with

179 ---------- Exam Section 4: Item 29 of 50 National Board of Medical Examiners ement 29. A 35-year-old homeless man is brought to the emergency department by police after they found him slumped on a bench. His pulse is 40/min, respirations are 10/min, and blood pressure is 100/60 mm Hg. He is stuporous but can be aroused when shaken mildly. Physical examination shows constricted pupils. This patient is most likely abusing which of the following substances? O A) Alcohol O B) Barbiturates O C) Benzodiazepines O D) Marijuana E) Opioids

E. This patient is likely intoxicated with opioids, which are central nervous system (CNS) depressants used as analgesics and recreational drugs. Opiates act throughout the CNS and peripheral nervous system and interact with several neurotransmitter systems, leading to diverse effects. Opiate intoxication causes euphoria (due to interaction with dopamine), altered mental status, sedation, bradycardia and hypotension, depressed respiratory drive, and constricted pupils. Miosis is a distinctive finding that is less common in intoxication with other CNS depressants and is caused by direct opiate receptor activity in brain areas responsible for pupillary control. Opiates also act on receptors within the enteric nervous system, reducing gut motility and causing constipation. Incorrect Answers: A, B, C, and D. Alcohol (Choice A) is a CNS depressant that can cause slurred speech, emotional lability, behavioral disinhibition, ataxia, and memory lapses (similar to benzodiazepine intoxication). Alcohol intoxication typically results in tachycardia, not bradycardia. In severe intoxication, stupor and respiratory depression can occur. Barbiturates (Choice B) are CNS depressants with similar toxidromes to alcohol and benzodiazepines. Bradycardia can occur, and the risk of dangerous respiratory depression is higher in barbiturates than benzodiazepines or alcohol. Pupils are typically of normal size or dilated in barbiturate intoxication, and physical examination may reveal nystagmus. Benzodiazepines (Choice C) are CNS depressants with a toxidrome of slurred speech, ataxia, and altered mental status. Co-ingestion with an opiate more commonly causes respiratory depression than benzodiazepine intoxication alone. Nystagmus may be seen, and pupillary constriction would be atypical. Marijuana (Choice D) is a hallucinogen that causes eupho

97 ---------- Exam Section 2: Item 47 of 50 Natjonal, Board pf Medical Examinersment * 47. A 66-year-old woman comes to the physician for a follow-up examination 3 months after she underwent a total colectomy for ulcerative colitis. She subsequently required an ileostomy. She feels generally well and has resumed a nearly normal diet. Her temperature is 37°C (98.6°F), pulse is 92/min, and blood pressure is 100/60 mm Hg. Physical examination shows mildly decreased skin turgor and a well-healed ileostomy site. Laboratory studies show: Serum 136 mEq/L 3 mEq/L 114 mEq/L 14 mEq/L 32 mg/dL 1 mg/dL Na+ K+ CI- HCO3- Urea nitrogen Creatinine Arterial blood gas analysis on room air: pH Рсо2 Po2 7.32 30 mm Hg 80 mm Hg Which of the following best describes this patient's acid-base disturbance? A) Anion gap metabolic acidosis, partially compensated O B) Anion gap metabolic acidosis, uncompensated O C) Chronic respiratory alka

E. This patient's laboratory and arterial blood gas evaluation demonstrates a non-anion gap metabolic acidosis with partial compensation. Non-anion gap metabolic acidosis is identified by a pH less than 7.35 (acidemia), with decreased PCO2 and HCO3. The respiratory system compensates for the acidosis by increasing minute ventilation, which eliminates carbon dioxide, an acid. This results in decreased PCO2 and increased pH back to approximately normal range. This patient's persistently decreased pH suggests that respiratory compensation is only partial. Metabolic acidosis describes a derangement involving excess acid and is subdivided into anion gap and non-anion gap categories. Anion gap ([Na*] - ([CI] + [HCO3]) calculates unmeasured anions in serum which are contributing to acidity, whereas non-anion gap acidosis generally results from excess supply of protons or chloride, retention of protons or chloride by the kidney, or failure to secrete bicarbonate buffers. Non-anion gap acidosis is often a hyperchloremic metabolic acidosis, as seen in this patient. Causes of non-anion gap acidosis include infusions of normal saline, Addison disease, renal tubular acidosis, gastrointestinal losses of bicarbonate (eg, diarrhea), spironolactone, and acetazolamide use. Chloride retention or oversupply occurs in nearly all of these etiologies. In the case of diarrhea or gastrointestinal disturbance, bicarbonate secreted in the small intestine is normally reclaimed in the large intestine in exchange for chloride ions. If the colon cannot reclaim bicarbonate, such as following colectomy or during diarrheal illness, the level of serum chloride will remain increased and serum bicarbonate decreased. Incorrect Answers: A, B, C, D, and F. Anion gap metabolic acidosis, partially compensated (Choice A) and anion gap metabolic acidosis, uncomp

157 ---------- Exam Section 4: Item 7 of 50 Natjonal, Board of Medical Examinersment V 7. An 80-year-old man comes to the physician because of bleeding gums and bruises under his skin. He lives by himself and follows a diet low in vegetables and fruits. His prothrombin time is normal. Which of the following enzymes is most likely compromised in this clinical scenario? O A) 2,3-Epoxide reductase O B) Glucose 6-phosphate dehydrogenase O C) Hexokinase O D) Methionine synthase E) Prolyl hydroxylase O F) Pyruvate carboxylase O G) Pyruvate dehydrogenase

E. Vitamin C (ascorbic acid) is an antioxidant, facilitator of iron absorption, and coenzyme in the synthesis of collagen via prolyl hydroxylase and neurotransmitters via dopamine hydroxylase. Deficiency in vitamin C leads to scurvy, which presents with signs and symptoms of impaired collagen synthesis including swollen, bleeding gums, easy bruising and bleeding (eg, hemarthrosis), petechiae, impaired wound healing, and short, fragile, curly hair. It should not be confused with hemophilia, as it does not result in a deficiency of factors. Instead, the collagen and connective tissue deficiency weakens blood vessel walls resulting in easy bruising and bleeding. Collagen is synthesized by fibroblasts and begins in the rough endoplasmic reticulum with translation of collagen chains, which are glycine and proline rich. Prolyl hydroxylase, in a reaction requiring vitamin C, hydroxylates proline and lysine residues. This step, along with glycosylation, forms alpha-chains through disulfide bridging plus hydrogen bonding. Procollagen is exocytosed, where it forms tropocollagen after removal of the terminal ends. It is cross-linked extracellularly in a reaction that requires copper. In this case, the patient's bleeding gums, bruising, and diet limited in vitamin C plus normal prothrombin time suggests a diagnosis of scurvy due to impaired collagen synthesis. Incorrect Answers: A, B, C, D, F, and G. 2,3-epoxide reductase (Choice A) is an enzyme involved in hepatic synthesis of clotting factors and involves vitamin K as a coenzyme. This patient's prothrombin time is normal, suggesting appropriate hepatic synthesis of coagulation factors and sufficient vitamin K. Glucose-6-phosphate dehydrogenase (Choice B) is an enzyme involved in the metabolism of nicotinamide adenine dinucleotide phosphate. In deficiency, it presents with hemoly

156 ---------- Exam Section 4: Item 6 of 50 Natjonal, Board of Medical Examinersment 6. One hundred patients discharged from the hospital with a diagnosis of proximal deep venous thrombosis were followed for the development of venous ulceration for up to 8 years (DVT group). They were compared to a group of 200 patients who were discharged from the hospital with a diagnosis of pneumonia (the control group). At the end of 5 years, 25 patients in the DVT group and 10 patients in the control group developed venous ulceration. Which of the following is the attributable (excess) risk per one hundred patients for development of venous ulceration following deep venous thrombosis over 5 years? O A) 1 O B) 2 O C) 2.4 O D) 2.5 O E) 10 F) 20 O G) 24 H) 25

F. Attributable risk (AR) describes the risk of developing disease that can be attributed to the exposure as compared to the risk that exists without exposure. AR is calculated as the difference between disease incidence among those with the exposure and those without the exposure. In biostatistics, the lowercase letters a, b, c, and d are frequently used to represent numbers of patients exposed to a risk, receiving an intervention, or having a disease. The number of persons having the exposure or receiving the intervention who have the disease or condition in question is represented by "a". The number of persons having the exposure or receiving the intervention who do not have the disease or condition in question is represented by "b". The number of persons who do not have the exposure or who do not receive the intervention in question and who do have the disease or condition is represented by ". The number of persons who do not have the exposure or who do not receive the intervention in question and who do not have the disease is represented by "d". By this convention, the incidence of those with the exposure is computed by determining the number of patients with a positive exposure and who developed disease (a) and dividing it by the total number of patients who were exposed (a + b). The incidence of those without the exposure who developed disease is computed by determining the number of patients without a positive exposure who developed disease (c) and dividing it by the total number of patients who were not exposed (c +d). AR is defined as AR = b)) - (c / (c + d)). In this scenario, AR = (25 / 100) - (10 / 200) = (25/ 100) - (5/ 100) = 20 per 100 patients. (a / (a + %3D Incorrect Answers: A, B, C, D, E, G, and H. 1 (Choice A) and 10 (Choice E), represent the incidence, in the study period, of ulcerations in the c

119 ---------- Exam Section 3: Item 19 of 50 Natjonal, Board of Medical Examinersment ancive Rasic Saance 19. A 50-year-old woman from Egypt comes to the physician because of painless blood in her urine for 2 weeks. Urinalysis is within the reference range. Urine cytologic findings show malignant squamous epithelial cells. Which of the following is the strongest predisposing risk factor for this type of tumor? O A) Chronic pyelonephritis O B) Cigarette smoking O C) Diabetic nephropathy O D) Excessive consumption of caffeine O E) Long-term use of phenacetin F) Schistosoma haematobium infection

F. Infection with Schistosoma haematobium, a trematode, is a risk factor for the development of squamous cell carcinoma of the bladder. In the United States, urothelial (transitional) cell carcinoma of the bladder is a more common variant, accounting for 90% of new bladder cancer diagnoses, but in Africa and the Middle East, where Schistosoma haematobium is endemic, squamous cell carcinoma predominates. The organism causes chronic inflammation in the bladder wall, which eventually induces atypia followed by neoplasia. Regardless of the subtype, bladder cancer classically presents with painless hematuria, which may be grossly visible or microscopic, and occasionally dysuria or urinary frequency. Among all patients with painless gross hematuria, the incidence of bladder cancer is 10% to 20%. If the disease becomes advanced or metastatic, systemic signs such as fatigue, weight loss, and anorexia may be present. Urine cytology demonstrating malignant cells is highly specific for bladder cancer, though this finding is not always present. Cystoscopy with biopsy is the gold standard for the diagnosis of bladder cancer. Incorrect Answers: A, B, C, D, and E. Chronic kidney disease has many etiologies but may be caused by chronic pyelonephritis (Choice A), diabetic nephropathy (Choice C), or analgesic nephropathy. Chronic kidney disease affects the function of the renal glomeruli and nephrons rather than the lower urinary tract. It is not a risk factor for squamous cell carcinoma of the bladder, though may increase the risk of renal cell carcinoma. Cigarette smoking (Choice B) is squamous cell carcinoma of the urinary tract, but in a patient from an area where Schistosoma haematobium is endemic, this is more likely to be the strongest predisposing factor. strong risk factor for transitional cell carcinoma of the urinary tract. C

108 ---------- Exam Section 3: Item 8 of 50 Natjonal, Board pf Medical Examiners S ent 8. A 36-year-old man comes to the physician's office without an appointment and demands to be seen immediately. He has had a sore throat for 24 hours and explains his problem is more important than the other patients' because he is a vocalist. He is extremely upset when the receptionist explains he will need to wait for 1 hour to see the physician. He talks with her constantly while he waits. He says, "My wife and I split like 3 months ago. We have a 6-month-old boy, but she was just so into her pregnancy, she had no time for me. And she doesn't really appreciate my music. I need full support from my lady. Especially when my band's just about to make it big." Which of the following is the most likely personality disorder? %3D O A) Antisocial O B) Borderline O C) Compulsive O D) Dependent O E) Histrionic F) Narcissistic O G) Paranoi

F. Narcissistic personality disorder, a cluster B disorder (the dramatic or emotional cluster), is characterized by fragile self-esteem and compensatory arrogant, self-aggrandizing behavior to gain admiration, sometimes at others' expense. For example, a patient with narcissistic personality disorder may berate their physician for making a small mistake, which unconsciously inflates the patient's ego. This patient also demonstrates this entitled attitude, expecting special treatment from both his wife and doctor's office. He expects the receptionist to be interested in learning about his life, and he likely hopes she will verbalize approval of his grandiose perspective. First-line treatment for narcissistic personality disorder is psychotherapy. Incorrect Answers: A, B, C, D, E, G, and H. Antisocial personality disorder (Choice A) is a cluster B personality disorder that arises from a deficit in empathy, resulting in pervasive violations of others' rights, aggression, and a hostile and manipulative attitude toward others. These patients may also be chronically bored and hence sensation-seeking, leading to a high rate of substance abuse and gambling. Though this patient self-promotes at the expense of others, he does not demonstrate the aggression or violations of basic human rights that would be expected in antisocial personality disorder. Borderline personality disorder (Choice B) is a cluster B personality disorder that features an unstable sense of self and tumultuous relationships. Likely due to a combination of genetics and emotional invalidation during childhood and maturation, patients with borderline personality disorder unconsciously learn to make impulsive and dramatic gestures, including self-harm, to obtain emotional fulfilment from others. This patient fulfils his emotional needs through gaining others' ad

159 ---------- Exam Section 4: Item 9 of 50 Natjonal, Board of Medical Examinersmen' ansive Basic Scionce 9. A 40-year-old woman has had hypercalcemia for 1 year and recently passed a renal calculus. Serum parathyroid hormone and calcium concentrations are increased, and serum phosphate concentration is decreased. Parathyroid hormone most likely causes an increase in the serum calcium concentration by which of the following mechanisms? O A) Decreased degradation of 25-hydroxycholecalciferol O B) Direct action on intestine to increase calcium absorption O C) Direct action on intestine to increase magnesium absorption O D) Increased synthesis of 25-hydroxycholecalciferol O E) Inhibition of calcitonin production F) Stimulation of 1,25-dihydroxycholecalciferol production

F. Parathyroid hormone (PTH) stimulates 1a-hydroxylase in the kidney, which leads to the conversion of stored 25-hydroxycholecalciferol to active 1,25-dihydroxycholecalciferol. Once produced, 1,25-dihydroxycholecalciferol stimulates the increased absorption of calcium and phosphate from the intestine. PTH also has a role in the regulation of calcium and phosphate by stimulating osteoclastic bone reabsorption and distal convoluted tubular calcium reabsorption and phosphate excretion in the nephron. Primary hyperparathyroidism results in hypercalcemia and hypophosphatemia secondary to the aberrant increased production of PTH. The most common cause of primary hyperparathyroidism is a parathyroid adenoma. Common symptoms of primary hyperparathyroidism are secondary to the patient's resultant hypercalcemia and include recurrent nephrolithiasis, bone pain from osseous resorption, polyuria, dehydration, constipation, and psychiatric disturbances. Incorrect Answers. A, B, C, D, and E. PTH has no direct effect upon either the degradation (Choice A) or the synthesis (Choice D) of 25-hydroxycholecalciferol. 25-hydroxycholecalciferol is produced and stored in the liver through the hydroxylation of cholecalciferol by the enzyme cholecalciferol 25- hydroxylase. It is an inactive form of vitamin D and does not play an active role in the absorption of calcium and phosphate from the intestines. Direct action on intestine to increase calcium absorption (Choice B) is a characteristic of 1,25-dihydroxycholecalciferol, the active form of vitamin D. While PTH does increase intestinal absorption of calcium, this is primarily an indirect effect that is mediated by vitamin D. Direct action on intestine to increase magnesium absorption (Choice C) is not a characteristic of PTH. Magnesium is absorbed in the small intestine. Its absorption is inf

151 ---------- Exam Section 4: Item 1 of 50 Natjonal, Board pf Medical Examinersment 1. A 40-year-old woman has easy bruising and menorrhagia. Platelet count is 100,000/mm3, and bleeding time is prolonged. Factor VIII (antihemophilic factor) concentration is decreased, and platelet adhesion to collagen is abnormal. Which of the following is the most likely diagnosis? O A) Hemophilia A O B) Hemophilia B O C) Immune thrombocytopenic purpura O D) Ingestion of aspirin O E) Systemic lupus erythematosus O F) Thrombotic thrombocytopenic purpura O G) von Willebrand disease

G. von Willebrand disease is one of the most common hereditary bleeding disorders and is due to quantitative or qualitative abnormality of von Willebrand factor (vWF), which binds platelets and subendothelial collagen in primary hemostasis. Impaired platelet adhesion to the subendothelial lining, mediated by vWF, leads to a prolonged bleeding time. von Willebrand factor also transports factor VIII in plasma, which degrades rapidly when unbound. Factor VIII is a critical component of the intrinsic coagulation pathway and decreased levels lead to a prolonged partial thromboplastin time (PTT). von Willebrand disease is inherited in an autosomal dominant pattern. It can present with epistaxis, gingival bleeding, petechiae, easy bruising, and menorrhagia. Additional diagnostic testing can be performed with the ristocetin cofactor assay, which requires functional vWF for platelet aggregation to occur. Desmopressin can be used for treatment by promoting release of additional vWF stored in endothelial cells. Incorrect Answers: A, B, C, D, E, and F. Hemophilia A (Choice A) and Hemophilia B (Choice B), X-linked coagulopathies, arise due to a genetic deficiency in factor VIII or IX synthesis or activity, respectively. Patients often present with easy bruising and hemorrhage involving muscles and joints. Hemophilia does not cause platelet dysfunction and would not cause an increased bleeding time. Immune thrombocytopenic purpura (Choice C) is an acquired disorder of immune-mediated platelet destruction. It classically presents with petechiae and purpura, and prolonged bleeding time on laboratory analysis. Platelet levels are typically less than 100,000/mm3. It does not affect factor VIII. Ingestion of aspirin (Choice D) prolongs bleeding time by irreversibly inhibiting platelet activation through covalent binding to the cyclooxyge

173 ---------- Exam Section 4: Item 23 of 50 National, Board pf Medical Examiners ement 23. The graph shows the relationship between serum leptin concentration and BMI, expressed as (weight in kg) / (height in meters)2 for eight patients labeled A through H. The dashed lines indicate the upper and lower limits of normal. Which of the following patients most likely has primary leptin deficiency? A C O A) Patient A Normal E O B) Patient B F G H. O C) Patient C O D) Patient D BMI (kg/m) O E) Patient E O F) Patient F O G) Patient G H) Patient H

H. Leptin, a peptide hormone, is produced by adipocytes and enterocytes. In the fed state or settings of energy excess (eg, obesity), leptin production is upregulated and acts on hypothalamic nuclei to mediate feeding behavior by reducing the sensation of hunger and increasing satiety. Counterregulatory to leptin is ghrelin, which is produced in states of low energy balance and stimulates hunger and anabolism. In disease states, mutations of leptin resulting in inactivation or decreased production, or decreased receptor response to binding by leptin (analogous to type 2 diabetes mellitus) have been associated with obesity. Such derangement results in obesity marked by an increased BMI, reduced desire to exercise, and increased appetite in spite of sufficient caloric intake. In the graph above, patient H demonstrates an increased BMI and a low serum level of leptin, suggesting a primary leptin deficiency. Primary deficiencies describe an absent or sub-functional protein, which prevents or limits the downstream effect of that protein. Incorrect Answers: A, B, C, D, E, F, and G. Choices A, B, and C reflect patients with increased serum leptin, which would not be seen in the case of a primary leptin deficiency. In a primary deficiency, leptin protein would be absent, mutated, or degraded early due to misfolding. Choice C in particular may reflect a suboptimal response by the hypothalamus to leptin as it is marked by both obesity and increased leptin level. Choice D and Choice E reflect patients with a normal leptin level with a low BMI or a normal leptin level with a high BMI, respectively. These cases may reflect polymorphisms in leptin generation or response but would not reflect a primary deficiency. Choices F and G may reflect primary leptin deficiency as the leptin level is low, however a secondary deficit is theoreti

181 ---------- Exam Section 4: Item 31 of 50 Natjonal, Board of Medical Examinersment V 31. A study is conducted to assess the prevalence and incidence of obesity in a group of middle-aged men. A total of 900 men between the ages of 40 and 50 years are chosen to participate. The table shows the number of participants at baseline and at the follow-up study 5 years later for each BMI range. Number of Participants Baseline 300 BMI (kg/m2) Normal (< 25) Overweight (between 25 and 30) Obese (2 30) Years Later 225 500 525 100 150 The prevalence of obesity in this study population after 5 years is which of the following? O A) 50/100 B) 50/500 O C) 50/800 O D) 50/900 O E) 150/375 O F) 150/525 O G) 150/750 H) 150/900

H. Prevalence is defined as the number of cases of a disease divided by the total number of persons in the at-risk population. It describes the percentage or fraction of those who are affected within those who might be affected. Point prevalence is the prevalence of a disease at a specific point in time. This distinction can be useful as the prevalence of a disease may change over time. In this study, the prevalence of obesity at the start of the study (100 obese persons/900 middle-aged men at risk) is less than the prevalence of obesity after five years (150 obese persons/900 middle-aged men at risk). Incorrect Answers: A, B, C, D, E, F, and G. 50/100 (Choice A), calculates the percentage increase in prevalence over time. At year 5, 150 persons were identified as obese. 50/100 represents a 50% increase in obesity, calculated as 150 obese persons - 100 obese persons / 100 obese persons. 50/500 (Choice B), calculates the ratio of the number of people newly diagnosed with obesity at time = 5 years to the number of persons who were overweight at the start of the study. 50/800 (Choice C), represents another important epidemiologic measure that can be contrasted with prevalence, incidence. Incidence is the number of new cases over the total population at risk over a specified period of time. The cumulative incidence of new diagnoses of obesity in this population is 50 (the number of new cases occurring during the study). Dividing this by 800 (the total number of persons at risk at the beginning of the study period) reflects the incidence. 50/900 (Choice D), calculates the ratio of the number of people newly diagnosed with obesity at time = 5 years to the total number of participants in the study. 150/375 (Choice E), calculates the percentage of obese persons at time = 5 years within the group of persons having a normal or o

163 ---------- Exam Section 4: Item 13 of 50 Natjonal, Board pf Medical Examinersment ansive Rasi aance 13. A 35-year-old woman is brought to the emergency department comatose after she sustained multiple injuries in a motor vehicle collision. During the next few days, she develops sepsis and respiratory insufficiency, and then she dies. A photomicrograph of her lungs obtained at autopsy is shown. If this patient had recovered, regeneration of the alveolar epithelium would have been accomplished via hyperplasia of which of the following cell types? O A) Alveolar capillary endothelial cell O B) Alveolar macrophage O C) Chondrocyte O D) Ciliated columnar epithelial cell O E) Club cell O F) Goblet cell O G) Kulchitsky cell O H) Squamous epithelial cell O 1) Type I pneumocyte J) Type Il pneumocyte

J. The patient likely suffered from acute respiratory distress syndrome (ARDS) secondary to sepsis. ARDS is characterized by acute onset, diffuse, inflammatory lung injury and bilateral pulmonary infiltrates on chest imaging leading to respiratory failure. There are numerous etiologies, including sepsis (especially in the setting of Gram-negative bacteria due to macrophage overactivation by lipopolysaccharide), pneumonia, chemical pneumonitis (eg, aspiration), pulmonary contusion, acute pancreatitis, trauma, transfusions, medications, and amniotic fluid embolism. Alveolar damage and inflammation lead to increased permeability of the alveolar-capillary interface. The initial (acute or exudative) phase is characterized by diffuse alveolar damage, a leukocytic infiltrate, and a protein-rich exudate which coats the alveoli in a thick hyaline membrane, as seen in the photomicrograph. The second (proliferative or organizing) phase begins around one week after the initial injury, with repair of the damaged interface and proliferation of fibroblasts. Repair is accomplished via hyperplasia of type II pneumocytes and alveolar septal fibrosis, which can be seen on histology. Type Il pneumocytes are cuboidal cells which are the main producers of pulmonary surfactant. They also differentiate into type I pneumocytes to replace damaged cells, as type I pneumocytes are unable to self-replicate. Incorrect Answers: A, B, C, D, E, F, G, H, and I. Alveolar capillary endothelial cells (Choice A) form a thin, single-cell layer between the vascular lumen and the basement membrane of the alveolar-capillary interface. They are damaged in ARDS. Endothelial cell hyperplasia is needed to restore the endothelial layer but would not regenerate the epithelial layer of the interface. Alveolar macrophages (Choice B) are responsible for phagocytosis of

40 ---------- Exam Section 1: Item 41 of 50 National, Board of Medical Examiners Comprehensive Basic Science Self-Assessment 41. An investigator is studying patients with West Nile virus infection. During a 5-year period, data are collected on 25 patients diagnosed with this condition as confirmed by testing at the Centers for Disease Control and Prevention. Demographic information on the patients is reported (age, gender, and ethnicity), as well as information about likely sources of infection. Which of the following best describes this study design? A) Case series B) Case-control study C) Cohort study D) Correlational study E) Cross-sectional study

A. A case series is a descriptive study design in which a number of consecutive or nonconsecutive cases of a disease and/or treatment are described in detail, with information about exposure, demographics, and comorbidities. Case series do not imply a cause-and- effect relationship. They do not test a hypothesis nor are they randomized. They are useful in characterizing the natural history of a disease or response to treatment. They are also useful in describing rare diseases, as small amounts of patients may not permit conduction of larger case-control, cohort, or randomized trials with sufficient power. Incorrect Answers: B, C, D, and E. A case-control study (Choice B) investigates an association between exposure and an outcome. In this study design, a group of patients with the disease (cases) are identified. A group of patients without the disease (controls) are matched on baseline characteristics to the cases. Exposure data for the two groups is collected, and these data are compared to determine association with the outcome (disease) in question. An odds ratio may be calculated to compare exposures between groups. A cohort study (Choice C) identifies a group of patients and follows them over time to identify whether an exposure is associated with an outcome of interest. Cohort studies may be retrospective or prospective in design. In a prospective design, the hypothesis and analysis protocols are established prior to the start of the study period. In a retrospective design the hypothesis or question is designed after the study time period has passed. Ăn example of a prospective cohort study would be following a group of 1,000 smokers for a time period of 10 years and identifying the proportion of these patients who develop pancreatic cancer to identify the relative risk of pancreatic cancer in smokers. Correlati

14 ---------- Exam Section 1: Item 15 of 50 National Board of Medical Examiners Comprehensive Basic Science Self-Assessment 15. A previously healthy 19-year-old woman comes to student health services because of a 3-day history of vulvar itching and vaginal discharge. She has a new sexual partner and uses no contraception. Speculum examination shows a profuse yellow-gray discharge in the vagina. The pH of the discharge is 6. Microscopic examination of the discharge shows clue cells. Addition of KOH to the discharge produces a strong amine odor. Which of the following is the most likely cause? A) Bacterial vaginosis B) Bacteroides fragilis C) Candida albicans D) Haemophilus ducreyi E) Herpes simplex virus F) HIV G) Human papillomavirus H) Pneumocystis jirovecii (formerly P. carinii) I) Treponema pallidum J) Trichomonas vaginalis

A. Bacterial vaginosis is a common gynecologic condition characterized by a shift of vaginal flora and overgrowth of particular bacterial species, most commonly Gardnerella vaginalis. G. vaginalis is a Gram-variable, facultative anaerobe. Bacterial vaginosis presents with gray, thỉn, malodorous vaginal discharge and may cause vulvovaginal pruritus. Vaginal pH is typically above 4.5, and a fishy odor is detected upon KOH testing. The identification of clue cells on wet mount preparation is diagnostic. Metronidazole is the first-line therapy. Bacterial vaginosis is not considered a sexually transmitted infection and treatment of asymptomatic sexual partners is not recommended. Incorrect Answers: B, C, D, E, F, G, H, I, and J. Bacteroides fragilis (Choice B) is an anaerobic, Gram-negative bacillus and a normal component of the gastrointestinal microbiota. It only results in infections when displaced from the colon such as following surgery, rupture, or trauma, where it has the potential to cause bacteremia, intra-abdominal infections, peritonitis, and subcutaneous abscesses. Candida albicans (Choice C) is a fungus that also forms part of the normal vaginal flora. Overgrowth of Candida species may produce vulvovaginitis. Candidal vulvovaginitis, in contrast to bacterial vaginosis, produces pruritus and a white discharge. Vaginal pH is normal, and hyphae may be visualized on wet mount. Haemophilus ducreyi (Choice D) is a Gram-negative organism that causes the sexually transmitted infection chancroid. Chancroid presents with painful genital ulceration and inguinal lymphadenopathy. Herpes simplex virus (Choice E) is an enveloped, double-stranded DNA virus that causes sexually transmitted infections among other syndromes (eg, encephalitis, herpes labialis, herpetic whitlow). Symptoms of genital herpes include pain, burning,

42 ---------- Exam Section 1: Item 43 of 50 National, Board of Medical Examiners Comprehensive Basic Science Self-Assessment 43. A female newborn delivered at term develops jaundice, hepatomegaly, and rash shortly after delivery. Pregnancy was complicated by a 2-week febrile illness during the first trimester. The newborn is at the 25th percentile for length, 30th percentile for weight, and 20th percentile for head circumference. One week later, a CT scan of the head shows intracranial calcifications. Two days later, she becomes increasingly lethargic and develops respiratory distress and seizures. She has a 2-year-old sister who attends day care. Which of the following is the most likely causal virus? A) Cytomegalovirus B) Herpes simplex virus C) Measles virus D) Mumps virus E) Varicella-zoster virus

A. Congenital infection with cytomegalovirus (CMV), also known as human herpesvirus 5 (HHV-5), causes hearing loss, seizures, a petechial rash, and intracranial calcifications in the neonate when acquired in utero. Hepatomegaly, splenomegaly, and lymphadenopathy are also commonly seen. The virus is transmitted via the placenta from the mother, who often contracts the virus after interacting closely with young children. Symptoms of maternal primary infection typically include fever, myalgias, and laboratory evidence of atypical lymphocytosis, often with abnormal liver enzymes. In an immunocompromised host, CMV can also present with a variety of atypical syndromes including colitis, encephalitis, pneumonia, esophagitis, or retinitis. Histopathologic examination of infected cells demonstrates prominent inclusion bodies. Unlike other viruses in the HHV family, CMV is not susceptible to guanosine analogs like acyclovir and valacyclovir as the virus does not possess the thymidine kinase necessary to activate this class of medications. Incorrect Answers: B, C, D, and E. Herpes simplex virus (Choice B) infection can occur via the placenta or through direct inoculation as the neonate passes through the birth canal in a mother with active lesions. Infection with herpes simplex virus can cause both cutaneous herpetic lesions and systemic manifestations like meningoencephalitis. It is not a cause of intracranial calcifications. Measles virus (Choice C) may be acquired via placental transmission or at the time of delivery. Clinical manifestations include a prodrome stage of fever, malaise, and anorexia. This is followed by the classic triad of cough, coryza, and conjunctivitis. An erythematous rash beginning on the face and spreading cephalocaudally is also characteristic. Neonates with measles infections may present with a spectru

7 ---------- Exam Section 1: Item 9 of 50 National, Board of Medical Examiners Comprehensive Basic Science Self-Assessment 9. After being given an infusion of mannitol (400 mM), a healthy person is most likely to have which of the following changes in plasma osmolality and plasma ADH (vasopressin) concentration? 10 →B 0- D 280 290 300 310 Plasma osmolality (mOsmol/L) O Control before infusion A) B) C) D) E)

A. Mannitol, an osmotically active carbohydrate monomer, raises serum osmolality when given as an infusion. Mannitol, when infused, is a hyperosmolar solution. Following infusion, the net serum osmolality will be increased as mannitol molecules remain within serum. In turn, the increased osmolality of serum will draw fluid into the intravascular space from the interstitium. This principle is used therapeutically in the treatment of acute increases in intracranial and intraocular pressure, for example in cases of acute cerebral edema, impending brain herniation, or glaucoma. Osmoreceptors detect the change in osmolality, and trigger production of antidiuretic hormone (ADH) by the hypothalamus with release by the posterior pituitary gland. ADH, in turn, increases insertion of aquaporin channels in the collecting duct of the nephron. Free water is reclaimed by the aquaporins, which dilutes the serum and in turn normalizes osmolality. In this graph, person A has serum studies that reflect increased osmolality and increased production of ADH, which reflects serum changes following the administration of mannitol. Incorrect Answers: B, C, D, and E. Choice B reflects increased osmolality without associated increased serum ADH. This set of values may be obtained in the immediate moments following a rapid bolus of mannitol, but ADH levels would quickly increase to compensate for increased serum osmolality. Choice C reflects increased osmolality and decreased ADH, which is not consistent with an appropriate physiologic response to increased osmolality. Choice D reflects decreased osmolality, not increased as would occur with the administration of mannitol. In a case of physiologically decreased serum osmolality, as in psychogenic polydipsia, low serum ADH would be expected as a normal physiologic response. Choice E reflects decre

15 ---------- Exam Section 1: Item 16 of 50 National, Board of Medical Examiners Comprehensive Basic Science Self-Assessment 16. A 9-year-old boy is brought to the physician because he told his teacher that his fingers felt "funny" after he swung on the monkey bars 2 hours ago. Neurologic examination shows numbness and tingling in the ring and small fingers of the left hand. The physician concludes that the ulnar nerve may have been stretched while he was hanging from the bars. Which of the following other functions should be checked to assess the integrity of the ulnar nerve in the left limb? A) Abduction of the index, middle, ring, and small fingers B) Abduction of the thumb C) Extension of the index, middle, ring, and small fingers D) Extension of the thumb E) Opposition of the thumb

A. Peripheral nerve injuries of the upper extremity are commonly caused by compression, direct trauma, or nerve traction. In this patient, hanging from the monkey bars placed tension on the nerves that originate from the lower brachial plexus. Roots C8 and T1 contribute to the ulnar nerve. Injury to this nerve is causing the patient's distal motor and sensory symptoms. The ulnar nerve innervates the intrinsic muscles of the hand (eg, lumbricals, interossei, hypothenar, and palmaris brevis), and muscles of the forearm (eg, flexor carpi ulnaris and the ulnar portion of flexor digitorum profundus). It also provides sensation to the volar aspects of the fourth and fifth digit. Injury to this nerve will cause weakness in abduction and adduction of the index, middle, ring, and small fingers, as the dorsal interossei and the palmar interossei control these movements, respectively. To isolate ulnar nerve motor function, the patient's abduction and adduction of the digits can be evaluated with resistive strength testing. Sensation can be tested along the volar aspect of the fourth and fifth fingers. Incorrect Answers: B, C, D, and E. Abduction of the thumb (Choice B) is primarily controlled by abductor pollicis brevis and abductor pollicis longus. Abductor pollicis brevis is innervated by the recurrent branch of the median nerve which can be injured during carpal tunnel surgery. The abductor pollicis longus is innervated by the posterior interosseous nerve, a branch of the radial nerve. This nerve can be injured in a humerus fracture. Extension of the index, middle, ring, and small fingers (Choice C) is provided by the extensor digitorum communis muscle, which is innervated by the radial nerve. Extension of the thumb (Choice D) is controlled primarily by extensor pollicis brevis and extensor pollicis longus, both of which are i

4 ---------- Exam Section 1: Item 4 of 50 National Board of Medical Examiners Comprehensive Basic Science Self-Assessment 4. A male newborn is found to have a defect in anchoring fibrils. Which of the following skin findings is most likely in this patient? A) Blisters B) Easy bruising O C) Eczematous rash O D) Inability to sweat O E) Thickened skin O F) Ulcers

A. The hemidesmosome is an intricate complex of proteins whose ultimate function is to anchor the basal keratinocytes of the epidermis to the dermis at the dermal-epidermal junction. Any impairment of the hemidesmosome will cause the basal keratinocytes to separate from the dermis, causing a blister to form. Because the hemidesmosomes of neighboring skin are still intact, these will be tense blisters. Anchoring fibrils are made of type VII collagen and are a component of the hemidesmosome. A mutation or antibody to collagen type VII, as seen in epidermolysis bullosa, will cause blistering to occur. Bullous pemphigoid is another disease which affects the hemidesmosome. In contrast, pemphigus vulgaris is caused by antibodies to desmosomes, the protein complex which maintains cell to cell adhesion in the epidermis. Because the target of pemphigus vulgaris is more superficial, in the epidermis, those blisters will be fragile and flaccid. Incorrect Answers: B, C, D, E, and F. Easy bruising (Choice B) and petechiae may be seen in Vitamin C deficiency, or scurvy. Vitamin C is necessary to produce collagen, as it is a cofactor in the hydroxylation of proline and lysine residues, which is a key step in the conversion of preprocollagen to procollagen. Eczematous rashes (Choice C) may be seen in many genetic disorders including atopic dermatitis, autosomal dominant hyper-IgE syndrome (Job syndrome), and Wiskott-Aldrich syndrome. While acute eczematous reactions can have small vesicles due to edema within the epidermis, blisters are not seen. Itchy, erythematous patches are a classic finding. Inability to sweat (Choice D) is seen in a group of inherited disorders called ectodermal dysplasias. In these disorders, ectoderm-derived structures including the sweat glands, hair, teeth, and nails are abnormal. The anchoring fibrils are i

33 ---------- Exam Section 1: Item 34 of 50 National, Board of Medical Examiners Comprehensive Basic Science Self-Assessment 34. A 54-year-old woman with a 25-year history of rheumatoid arthritis that is refractory to therapy is admitted to the hospital for a joint replacement operation. Preoperative laboratory studies show: Hemoglobin Mean corpuscular volume Leukocyte count Platelet count 10.5 g/dL 79 μm3 8900/mm3 230,000/mm 3 Serum Iron Transferrin 40 μg/dL 220 mg/dL (N=200-400) A photomicrograph of a Prussian blue iron-stained smear of bone marrow aspirate is shown. Which of the following is the most likely cause of the anemia? A) Autoimmune hemolysis B) Chronic disease C) Iron deficiency O D) Marrow replacement by neoplastic tissue O E) Myelodysplasia O F) B-Thalassemia minor

B. Chronic disease (anemia of chronic disease [ACD]) accounts for this patient's anemia. ACD is found in many chronic inflammatory, infectious, and malignant disease states including rheumatoid arthritis, lymphoma, and tuberculosis. This patient's rheumatoid arthritis with microcytic anemia, normal iron levels, and bone marrow smear with Prussian blue-iron staining revealing increased iron storage are consistent with this diagnosis. Anemia is often categorized into macrocytic, normocytic, and microcytic etiologies. Other causes of microcytic anemia include thalassemia, iron deficiency anemia, and sideroblastic anemia. To determine the specific cause of microcytic anemia (which guides therapy), diagnostic evaluation is necessary and includes red blood cell indices (eg, red cell distribution width, mean cell volume, mean corpuscular hemoglobin concentration) and iron studies (eg, transferrin, serum iron level, total iron binding capacity, ferritin). In ACD, iron studies will show normal or low serum iron level, and normal or increased ferritin level. While ACD usually causes microcytosis, it can present with normal mean corpuscular volume when early. While not routinely performed, a bone marrow aspirate with Prussian blue staining for iron will reveal increased deposits of iron in the bone marrow, as shown in the photomicrograph. The pathophysiology may relate to upregulated hepcidin, resulting in reduced intestinal absorption of iron and reduced release of iron from macrophages. While patients may not be iron deficient, their iron stores are unavailable for hematopoiesis, leading to anemia. Incorrect Answers: A, C, D, E, and F. Autoimmune hemolysis (Choice A) occurs as a result of antibody production against circulating red blood cell surface antigens, leading to phagocytosis or activation of complement, both of which l

82 ---------- Exam Section 2: Item 32 of 50 Natjonal, Board of Medical Examinersment asive Basic Science 32. A 52-year-old man with chronic renal failure receives a cadaveric kidney transplant. Postoperatively, he is given cyclosporine for immunosuppressive therapy. Six weeks after the operation, he develops hypoxemia with an arterial Po2 of 40 mm Hg. A photomicrograph of a transbronchial biopsy specimen is shown. Which of the following pathophysiologic mechanisms best explains this patient's hypoxemia? A) Alveolar exudation due to Cryptococcus neoformans infection O B) Cytomegalovirus pneumonitis with diffuse alveolar damage O C) Neutrophilic alveolar consolidation due to pneumococcal pneumonia O D) Pneumocystis jirovecii (formerly P. carinii) infection with alveolar exudation E) Vascular invasion by Aspergillus with pulmonary infarction

B. Cytomegalovirus (CMV), also known as human herpesvirus-5 (HHV-5), is an opportunistic infection that commonly occurs in immunocompromised patients in the setting of solid-organ or allogeneic stem cell transplantation, severe ulcerative colitis, or HIV/AIDS. CMV pneumonitis with diffuse alveolar damage is one of the many presentations that can occur in patients taking immunosuppressants to prevent organ rejection following solid organ transplant. It typically occurs in the first 3 to 6 months following transplantation. Radiologic imaging features are nonspecific but may demonstrate an interstitial pneumonitis pattern with the potential for infiltrative consolidations and ground-glass opacities to develop. The classic histologic findings of CMV pneumonitis are infected cells with prominent basophilic nuclear inclusions, as seen in this patient's lung biopsy. Other manifestations of CMV infection associated with immunosuppression include systemic infectious mononucleosis, retinitis, esophagitis, colitis, and encephalitis. Incorrect Answers: A, C, D, and E. Alveolar exudation due to Cryptococcus neoformans infection (Choice A) may be seen in pulmonary cryptococcosis. Cryptococcus neoformans is an encapsulated yeast and opportunistic pathogen in immunocompromised patients, especially patients with HIV/AIDS with low CD4 counts. Disseminated disease can also cause cryptococcal meningitis and encephalitis. Histology reveals fungal cells with narrow-based budding and a bright red inner capsule on staining with mucicarmine. Neutrophilic alveolar consolidation due to pneumococcal pneumonia (Choice C) is seen in lung infections due to Streptococcus pneumoniae. Pneumonia classically presents with fever, dyspnea, productive cough, and pleuritic chest pain. Gram-positive diplococci would be seen on sputum cultures or bronchoalveol

51 ---------- Exam Section 2: Item 2 of 50 National Board of Medical Examiners Comprehensive Basic Science Self-Assessment 2. A40-year-old man has moderate edema. A 24-hour urine collection contains 15 g of protein (NS150 mg/24 h). Which of the following is the most likely mechanism of the edema? A) Decreased capillary hydrostatic pressure O B) Decreased plasma oncotic pressure C) Increased plasma oncotic pressure D) Decreased permeability of postcapillary venules E) Increased permeability of postcapillary venules

B. Flow into the interstitial space is dependent on interactions between capillary hydrostatic pressure, interstitial fluid hydrostatic pressure, plasma colloid oncotic pressure, and interstitial fluid oncotic pressure. If capillary hydrostatic pressure is high relative to interstitial fluid hydrostatic pressure, fluid will flow down a pressure gradient into the interstitial space. If the opposite is true (interstitial fluid pressure is higher than capillary pressure), fluid will flow into the capillaries. If plasma fluid oncotic pressure is high relative to interstitial fluid oncotic pressure, fluid will be drawn back into the capillaries. Conversely, if interstitial fluid oncotic pressure exceeds plasma fluid oncotic pressure, fluid will move into the interstitial space. Typically, excess fluid in the interstitial space is removed by the lymphatic system. However, when this system is overwhelmed, edema develops. Therefore, this patient has developed peripheral edema due to decreased plasma oncotic pressure secondary to the loss of protein through proteinuria. Incorrect Answers: A, C, D, and E. Decreased capillary hydrostatic pressure (Choice A) and increased plasma oncotic pressure (Choice C) would draw fluid into the capillaries from the interstitial space, decreasing edema. Decreased or increased permeability of postcapillary venules (Choices D and E) would affect edema, as fluid would be prevented from moving into or out of the interstitial space despite the influence of plasma or interstitial fluid hydrostatic and oncotic pressures. Any influence of venule permeability is a less likely underlying mechanism given this patient's profound proteinuria and subsequent decreased plasma oncotic pressure. Educational Objective: Movement of fluid between capillaries and the interstitial space is dependent on the hydrostati

49 ---------- Exam Section 1: Item 50 of 50 National Board of Medical Examiners Comprehensive Basic Science Self-Assessment 50. An investigator conducts a phase 1 clinical trial to test the efficacy of a systemic antagonist to lymphocyte function-associated antigen 1 (LFA-1). The drug is approved for topical administration to treat keratoconjunctivitis sicca in patients with multiple sclerosis. During the trial, it is found that these patients are at an increased risk for bacterial infection as a result of this drug. The most likely explanation for this risk is drug-induced blockade of which of the following segmented neutrophil activities? A) Activation of phagocytosis B) Adhesion to the endothelium C) Diapedesis through the capillary wall D) Migration to the site of injury E) Rolling at the vessel periphery

B. Neutrophil recruitment to sites of infection is an essential component of the innate immune response. Neutrophils directly eliminate pathogens by generation of reactive oxygen species and phagocytosis, recruitment of macrophages, and creation of a pro- inflammatory environment. The stages of neutrophil recruitment are margination, rolling, adhesion, extravasation, and chemotaxis/migration. Lymphocyte function-associated antigen 1 (LFA-1) is an integrin protein involved in neutrophil adhesion to the endothelium. LFA-1 antagonism leads to decreased neutrophil adhesion and an impaired innate immune response. The genetic disorder leukocyte adhesion deficiency type 1 is caused by a defect in LFA-1. Incorrect Answers: A, C, D, and E. Activation of phagocytosis (Choice A) is achieved by numerous inflammatory mediators, including bacterial proteins such as lipopolysaccharides, peptidoglycan, and teichoic acids, as well as complement proteins, inflammatory cytokines, and prostaglandins. Integrins are not involved in this process. Diapedesis through the capillary wall (Choice C), also called extravasation, is the process by which neutrophils move from the vascular compartment into the extracellular space. It is mediated by the integrin platelet/endothelial cell adhesion molecule 1 (PECAM-1), not LFA-1. Migration to the site of injury (Choice D), also called chemotaxis, is the movement of inflammatory cells through the extracellular space to the site of injury. It is mediated by numerous chemotactic agents, including IL-8, C5a, leukotriene B4, 5-HETE, and formyl- methionyl peptides. Rolling at the vessel periphery (Choice E) occurs following margination of the neutrophils to the periphery of the blood vessel and prior to neutrophil adhesion to the endothelial wall. It is mediated by selectins and glycoproteins (eg, Sialyl-Lewi

12 ---------- Exam Section 1: Item 13 of 50 National Board of Medical Examiners Comprehensive Basic Science Self-Assessment 13. A 64-year-old woman develops pain and stiffness of the proximal interphalangeal joints and the right knee. The pain is made worse by activity and is relieved by rest. X-rays of the knee show a narrowed joint space with radiodense subchondral bone and cyst formation. Knee fluid aspirate shows: Appearance Leukocyte count Neutrophils Glucose clear, yellowish 250/mm3 (N<200) 5% (N<25%) 101 mg/dL (N=80-100) Which of the following is the most likely cause of her condition? A) Acute gouty arthritis B) Osteoarthritis C) Rheumatoid arthritis D) Septic arthritis E) Normal age-related changes in joints

B. Osteoarthritis is the most common disease of joints in humans. It affects 60% to 80% of adults over the age of 65. It is characterized by global degeneration of a joint with fraying, fibrillation, and degradation of cartilage, thickening of the subchondral plate with sclerotic bone, and subchondral cysts due to infiltration of synovial fluid into the bone. On radiographs, the disease is demonstrated by joint space narrowing, articular sclerosis, radiolucent subchondral cysts, and sharpened corners of the joints with new bony protrusions (osteophytosis). If occurring before age 65 years, patients may have a history of prior traumatic injury to a joint that incited an inflammatory cascade and accelerated the onset of osteoarthritis. This presentation is known as posttraumatic arthritis. Joint aspiration is characterized by normal appearing aspirate on Gram stain and culture. However, this synovial fluid typically contains cartilage and bone degradation particles as well as inflammatory cytokines and metalloproteases that are not detectable on traditional aspiration. Osteoarthritis may occur concomitantly with inflammatory or septic arthritis. The diagnosis of osteoarthritis does not preclude the existence or occurrence of more aggressive arthropathies. Incorrect Answers: A, C, D, and E. Acute gouty arthritis (Choice A) is an inflammatory arthropathy incited by hyperuricemia, with deposition of uric acid crystals inside the joint and surrounding tissues. This type of arthritis presents following consumption of a purine-rich meal or alcohol (which competes with renal excretion of uric acid). A knee aspirate would show negatively birefringent needle-shaped crystals and a leukocyte count of typically less than 50,000 cells/mm°. Rheumatoid arthritis (Choice C) is a polyarticular inflammatory arthritis that is characterize

13 ---------- Exam Section 1: Item 14 of 50 National Board of Medical Examiners Comprehensive Basic Science Self-Assessment 14. Which of the following labeled depolarizations is associated with the smallest stroke volume? A B C D E A) В) C) D) E)

B. The electrical complex labeled B in the rhythm strip is characteristic of a premature ventricular contraction (PVC) and is associated with a lower stroke volume when compared to a normal sinus beat. Normal cardiac conduction starts with an electrical impulse in the sinoatrial (SA) node located in the right atrium, which is depicted on the rhythm strip as the "P wave." The electrical impulse is conducted throughout the atria, causing coordinated contraction of the right and left atria, with consequent active filling of the right and left ventricles. The electrical impulse also reaches the atrioventricular (AV) node located in the interatrial septum. Conduction is temporarily halted in the AV node to allow for complete active and passive filling of the ventricles, followed by conduction of the electrical impulse down the His-Purkinje system in the interventricular septum, then to the right and left bundle, and finally to the myocardial cells of the ventricles. This results in contraction of the right and left ventricles (QRS complex) with expulsion of blood into the pulmonary and systemic circulations, respectively. The ventricles subsequently repolarize (T wave) and relax to allow filling for the next cardiac cycle. Stroke volume is defined as the difference between the end-diastolic-volume and the end-systolic-volume. To achieve an ideal stroke volume, diastolic filling time must be sufficiently long to allow the ventricles to fill completely before contraction occurs. This is achieved by delaying action potential conduction in the AV node. A premature ventricular contraction, which is shown by letter "B," indicates premature ventricular systole. When this occurs, diastolic filling time is reduced, so the amount of blood in the ventricles prior to ventricular systole is less than would exist had filling time been su

63 ---------- Exam Section 2: Item 13 of 50 National Board of Medical Examiners aiveB A lance S ment * 13. A 32-year-old woman comes to the physician because she is concerned about fertility. Menses occur at regular 28-day intervals. Physical examination shows no abnormalities. An endometrial biopsy specimen is obtained from the uterine wall. Histologic sections show a well-developed stratum basale, intact spiral arteries, abundant endometrial stroma, and long, straight uterine glands with no secretions in their lumens. Based on this description, which of the following labeled options best corresponds to the point in the patient's menstrual cycle at which the biopsy was done? A B Menses Ovulation O A) B) C) O D)

B. The first half of the menstrual cycle, the follicular phase, begins with menses. After menses, follicle-stimulating hormone and luteinizing hormone (FSH and LH, respectively) levels begin to increase, which stimulates the development of a follicle. The follicle produces estrogen, which leads to the proliferation of the endometrium. This proliferative endometrium consists of highly active endometrial epithelial cells, stromal cells, straight tubular endometrial glands, and extension of the uterine spiral arteries, as seen in this patient. As estrogen rises, a surge occurs, which in turn stimulates a surge in LH that causes ovulation 14 days prior to menses. Immediately following ovulation, the luteal (secretory) phase begins as the corpus luteum forms. The corpus luteum secretes progesterone to produce and maintain a secretory endometrial lining, which consists of tortuous glands full of secretions, full length spiral arteries, and edematous stromal cells. If no implantation occurs, the corpus luteum degrades to the corpus albicans, causing estrogen and progesterone levels to decrease, resulting in menstruation. The loss of progesterone causes the endometrial lining to slough off due to constriction of the spiral arteries, and the follicular phase begins again. Incorrect Answers: A, C, and D. Immediately after menses (Choice A), the endometrium would be much less developed, as the entirety of the stratum functionalis (eg, the superficial layer) would have been sloughed off during menses. During menses, the spiral arteries retract into the stratum basalis and the glands are partially lost. Well-developed spiral arteries and uterine glands, as well as abundant stroma, would not be expected immediately after menses. The secretory phase (Choices C and D) consists of the time from ovulation to menstruation, which is typic

22 ---------- Exam Section 1: Item 23 of 50 National Board of Medical Examiners Comprehensive Basic Science Self-Assessment Normal AB Type 2 Diabetes Mellitus Fasting Serum Glucose 23. During an investigational study, fasting serum glucose concentrations are collected from a population of healthy individuals and a population of patients with type 2 diabetes mellitus. The results are shown in the graph. Switching from cut point A to cut point B would most likely result in which of the following changes in test characteristics regarding identification of patients with type 2 diabetes mellitus? A) Decreased specificity B) Improved sensitivity C) Increase in false-negatives D) Increase in false-positives E) No change

C. Any population of patients will include those who have the disease and those who do not, and overlap may exist between these groups when considering testing for nonbinary diagnoses. For example, type 2 diabetes mellitus is understood to be an insensitivity of peripheral tissues to insulin, marked by diminished insulin uptake and increased blood glucose. It is impossible to specifically measure the degree to which cellular receptors are resistant to insulin, therefore, alternative diagnostic criteria are used reflecting increased blood glucose and its effects. As of 2020, the American Diabetes Association criteria include two fasting serum blood glucose levels greater than or equal to 126 mg/dL, a random serum blood glucose level greater than or equal to 200 mg/dL with signs and symptoms of diabetes, a hemoglobin-A1c level of greater than or equal to 6.5%, or a 2-hour oral glucose tolerance test with increased blood sugar beyond 200 mg/dL. Not all patients with a fasting serum blood glucose greater than or equal to 126 mg/dL will have diabetes mellitus. For example, confounding factors such as chronic steroid use may result in increased blood glucose without impaired cellular uptake. These patients may receive positive tests, but do not have the underlying condition. Therefore, the set points for diagnostic tests will result in some true positives (patients who have the disease and who the tests identifies as having the disease), true negatives (patients who do not have the disease who the test identifies as not having the disease), false positives (patients who do not have the disease who the test identifies as having the disease), and false negatives (patients who have the disease who the test identifies as not having the disease). The set point will determine the fraction of patients who are true or false positive

35 ---------- Exam Section 1: Item 36 of 50 National, Board of Medical Examiners Comprehensive Basic Science Self-Assessment 36. A 40-year-old woman with chronic paraplegia caused by multiple sclerosis is brought to the physician because of severe, debilitating muscle cramps for the past 3 weeks. Treatment with baclofen resolves her muscle cramps. Which of the following receptors most likely decreased the muscle spasticity in this patient? A) a2Adrenoreceptor B) B2Adrenoreceptor C) y-Aminobutyric acid B receptor D) Calcium-sensing receptor E) Ryanodine receptor

C. Baclofen is a y-aminobutyric acid (GABA) analogue that acts as an agonist at GABA-B receptors. GABA-B receptors are metabotropic G-protein coupled receptors that increase efflux of potassium and thus hyperpolarize skeletal muscle cells and decrease action potential frequency. As such, baclofen is a first-line treatment for muscle spasticity. GABA-B receptors are also found in the central nervous system (CNS); therefore, baclofen can cause sedation, especially when used in conjunction with other CNS depressants. Incorrect Answers: A, B, D, and E. a2-Adrenoreceptors (Choice A) are G-protein coupled receptors localized to the CNS that decrease sympathetic outflow when activated. Examples of a2-adrenergic agonists include clonidine and guanfacine, which treat attention-deficit/hyperactivity disorder (ADHD) and hypertensive urgency. Tizanidine is an a2-adrenergic agonist that is used as a muscle relaxant. B2-Adrenoreceptors (Choice B) are G-protein coupled receptors involved in smooth muscle relaxation. Albuterol, a bronchodilator used in asthma, is one example of a B2-Adrenergic agonist. B2-Adrenergic agonists do not act on skeletal muscle, though tremors can be seen as a consequence of increased sympathetic tone. Calcium-sensing receptors (Choice D) are G-protein coupled receptors located in the renal tubule and parathyroid gland that sense the extracellular concentration of calcium ions. These receptors influence calcium homeostasis by regulating renal calcium reabsorption and parathyroid hormone release. Ryanodine receptors (Choice E) are calcium channels located on the sarcoplasmic reticulum membranes in skeletal muscle. These channels mediate the intracellular release of calcium during the excitation-contraction coupling essential to muscle contraction. Dantrolene antagonizes ryanodine receptors to treat muscle spa

90 ---------- Exam Section 2: Item 40 of 50 Natjonal, Board of Medical Examinersmen' * 40. A 28-year-old white woman comes to the physician for a follow-up examination. Two days ago, she was discharged from the hospital after being treated for a hypertensive crisis with seizure. She has no other history of major medical illness. Current medications include a B-adrenergic blocker, dihydropyridine calcium channel blocker, and a centrally acting agent stimulating presynaptic a2 receptors. Her temperature is 37°C (98.6°F), pulse is 54/min, and blood pressure is 182/100 mm Hg. Physical examination shows trace ankle edema. Cardiac examination shows an S4 gallop. A soft abdominal bruit is heard left of the umbilicus. Abdominal ultrasonography shows normal-sized kidneys and a normal liver, spleen, pancreas, and gallbladder; a small left ovarian cyst is noted. Laboratory studies show that complete blood count, serum electro

C. Fibromuscular dysplasia is a non-inflammatory and non-atherosclerotic angiopathy of medium-sized arteries (eg, renal, carotid), that results in multifocal fibrous and muscular thickening of the arterial wall. The resultant stenosis causes secondary hypertension due to abnormal stimulation of the juxtaglomerular apparatus from low afferent blood flow leading to excessive production of renin and angiotensin. It can lead to severe hypertension in an otherwise healthy, young patient with no medical comorbidities or laboratory abnormalities. Examination findings often include signs of left ventricular hypertrophy such as an S4 gallop and a renal artery bruit auscultated lateral to the umbilicus on the affected side. Angiography may reveal a bead-like appearance of the renal artery. Treatment involves angioplasty or stenting of the stenosed renal artery to improve flow. ACE inhibitors can be considered for unilateral stenosis but can lead to acute renal failure in the setting of significant bilateral renal artery stenosis. Incorrect Answers: A, B, D, and E. Chronic kidney disease (Choice A) can have variable etiology, such as with chronic prerenal disease in the setting of heart failure or cirrhosis with decreased renal perfusion, intrinsic renal disease such as nephrosclerosis or atherosclerotic renal artery disease, chronic nephritic or nephrotic glomerular disease, or chronic postrenal obstructive disease. Chronic kidney disease is more common in older patients after years of underlying kidney injury. Coarctation of the aorta (Choice B) often presents in childhood with hypertension in an upper extremity and a systolic murmur. It is associated with weak lower extremity pulses and differentially lower blood pressure in the lower extremities. Hyperthyroidism (Choice D) may cause hypertension, but would be associated with

48 ---------- Exam Section 1: Item 49 of 50 National, Board of Medical Examiners Comprehensive Basic Science Self-Assessment 49. An investigator conducts a clinical study of 300 patients with stages III and IV colon carcinoma. It is found that over time, metastases show an increasing potential for growth and resistance to chemotherapy. Which of the following mechanisms of the cell cycle best explains this observation? A) Decreased expression of cyclin B during G 2/M transition B) Decreased phosphorylation of RB1 protein during G1-S transition C) Genomic instability during S, G2 and M phases D) Overexpression of p15, p16, and p18 during G1 phase E) Overexpression of p21, p27, and p57 proteins during all phases

C. Genomic instability during S, G2, and M phases explains the increased potential for growth and resistance to chemotherapy agents used to treat stage III and IV colon carcinoma. The two primary forms of genomic instability that predispose to the formation of malignant cells are microsatellite and chromosome instability. In the former, the failure of mismatch repair pathways (MMR) from mutations in MLH1, MSH2, APC, and other genes, results in the propagation of aberrant DNA and accumulation of mutations that predispose to malignant transformation. This is the mechanism of oncogenesis found in familial cancer syndromes such as Lynch syndrome or familial adenomatous polyposis (FAP). In the latter instance of chromosome instability, specific activating mutations of oncogenes or loss of function mutations of tumor suppressor genes results in oncogenic potential. This pathway accounts for the majority of sporadic colon cancers. Activating mutations of oncogenes such as KRAS and inactivating mutations of tumor suppressor genes such as P53 predispose cells to abnormal growth through abnormal entry into the cell cycle, which results in cellular proliferation. Within the cell cycle, errors of replication further predispose to malignant growth and may account for resistance to chemotherapeutic agents. The normal cell cycle in somatic cells involves several stages: G1, Š, G2, and M phase. Go phase denotes cell cycle arrest. During G1 phase, the cellular contents, with the exception of the chromosomes, are replicated. In S phase, chromosomal duplication occurs. In G2 phase, the fidelity of replication is checked. M phase involves the attachment of chromosomes to spindles followed by their separation and division into two identical cells, with the stages of M phase denoted as prophase, metaphase, anaphase, and telophase. Derangem

96 ---------- Exam Section 2: Item 46 of 50 National Board of Medical Examiners ment 46. A 45-year-old man comes to the physician because of a 6-month history of progressive shortness of breath with exertion. He has a history of frequent nosebleeds since adolescence. He is 178 cm (5 ft 10 in) tall and weighs 79 kg (175 lb); BMI is 25 kg/m2. Physical examination shows the findings in the photographs. Inhaled albuterol does not improve his symptoms. Which of the following is the most likely cause of this patient's condition? O A) Atrial septal defect O B) Hypertension o C) Pulmonary arteriovenous shunting O D) Reactive airway disease O E) Thromboembolism

C. Hereditary hemorrhagic telangiectasia (HHT) is an autosomal dominant disorder of the vasculature that is characterized by arteriovenous malformations (AVMS) and telangiectasias, which are small vascular malformations which appear as bright red, blanching lesions of the skin and mucous membranes (as seen in the oral photograph). There are several gene mutations associated with the disorder that result in abnormal formation of capillaries and the manifestations of the disease. Patients classically present with recurrent epistaxis, hematuria, and gastrointestinal bleeding. Cerebral AVMS may cause hemorrhagic stroke. Pulmonary AVMS can lead to embolic stroke as venous blood (and a potential embolus) is able to bypass the pulmonary capillary bed and enter the systemic arterial circulation. Pulmonary AVMS also result in pulmonary arteriovenous shunting with subsequent chronic hypoxemia, which can lead to hypertrophic osteoarthropathy (digital clubbing) as seen in the photograph of the fingers. HHT is associated with pulmonary arterial hypertension and iron deficiency anemia secondary to chronic blood loss. Incorrect Answers: A, B, D, and E. Atrial septal defect (Choice A) is a common congenital malformation of the interatrial septum that results in a left-to-right shunt with abnormal flow of blood from the left atrium to the right atrium. If the atrial septal defect remains uncorrected, it can result in the development of Eisenmenger syndrome secondary to prolonged pulmonary vasculature remodeling resulting in pulmonary arterial hypertension and shunt reversal leading to cyanosis. It is not associated with recurrent bleeding or mucocutaneous lesions. Hypertension (Choice B) is a leading cause of cardiovascular mortality. Chronic hypertension can result in left ventricular hypertrophy and ultimately left-sided heart failur

93 ---------- Exam Section 2: Item 43 of 50 National, Board pf Medical Examiners ancive Basic Saance S ement 43. A 15-year-old girl is brought to the physician because of a 3-day history of fever, sore throat, and malaise. Her temperature is 39.2°C (102.6°F). Physical examination shows diffuse pharyngeal erythema, moderately enlarged tonsils, and tender anterior and posterior cervical lymphadenopathy. A complete blood count shows: 19,500/mm3 (N=3500- 10,500) Leukocyte count Segmented neutrophils 30% Bands 7% 2% 25% Eosinophils Lymphocytes Lymphocytes, atypical Monocytes 30% 6% Incubation of this patient's serum with sheep erythrocytes results in agglutination. The atypical lymphocytes in this patient are most likely which of the following cell types? O A) B lymphocytes O B) CD4+ T lymphocytes C) CD8+ T lymphocytes D) FOXP3-expressing regulatory T lymphocytes O E) Natural killer cells

C. Infectious mononucleosis is a viral illness caused often by Epstein-Barr virus (EBV). EBV is transmitted through respiratory secretions and saliva, causing the illness to be commonly acquired by teenagers and young adults. EBV infects B lymphocytes through CD21. While B lymphocytes are the cell type infected, atypical lymphocytes seen on peripheral blood smear are actually cytotoxic, CD8+ T lymphocytes, which are reacting to the viral infection. As a result, patients often have leukocytosis with a lymphocytic predominance. Cytomegalovirus (CMV) is another cause of infectious mononucleosis and an acute HIV infection may present with a similar syndrome. Clinically, the syndrome commonly presents with fever, lymphadenopathy (typically involving the posterior cervical lymph nodes), and hepatosplenomegaly along with pharyngitis. Patients with infectious mononucleosis will typically test positive on the Monospot test, which detects heterophile antibodies through the agglutination of sheep or horse erythrocytes. Treatment is supportive and patients should be counseled to avoid any contact sports until the splenomegaly has resolved due to the risk of splenic rupture. If amoxicillin is administered for treatment of a misdiagnosed pharyngitis thought to be from Streptococcus pyogenes, patients may develop a generalized maculopapular rash. Incorrect Answers: B, D, and E. While the EBV virus infects B lymphocytes through CD21 (Choice A), these are not the cell type that appear atypical on peripheral blood smear. T lymphocytes can be broken into three groups. CD8+ T lymphocytes are cytotoxic and recognize major histocompatibility complex (MHC)-I on the surface of antigen presenting cells. They are responsible for the direct killing of infected cells. CD4+ T lymphocytes (Choice B) recognize MHC-II on the surface of antigen presen

21 ---------- Exam Section 1: Item 22 of 50 National Board of Medical Examiners Comprehensive Basic Science Self-Assessment 22. An 18-month-old boy in West Africa with perinatally acquired HIV infection is exposed to a child in his village with a fever and an erythematous, maculopapular rash. Three weeks later, the 18-month-old boy develops giant cell pneumonia, but there is no evidence of a rash. Which of the following viruses is the most likely cause of the pneumonia? A) Coxsackievirus B) Influenza virus C) Measles virus D) Rubella virus E) West Nile virus

C. Measles (rubeola) virus is a highly contagious paramyxovirus that causes the disease measles, which is associated with acute febrile illness and potentially severe sequelae. It is more common in children than adults. It characteristically presents with prodromal fever, cough, coryza, conjunctivitis, and a confluent maculopapular rash that starts at the head/neck and spreads to the trunk, excluding the palms/soles. Physical examination may reveal bright red macules with a bluish-white center on the buccal mucosa and lymphadenopathy. The virus replicates in epithelial cells of the respiratory tract and lymph nodes causing lymphadenitis. Virulence factors include the F (fusion) protein which aids in viral fusion to host cells and can result in the formation of multinucleated giant cells. Hemagglutinin (HA) protein is involved in viral binding to host cells. Treatment is supportive. Vitamin A deficiency is associated with severe disease and complications, and supplementation should be provided, especially in children who are malnourished. Complications include subacute sclerosing panencephalitis (a neurodegenerative disease associated with intellectual regression, personality changes, motor deterioration, seizures, and premature death), encephalitis, and giant cell pneumonia. Giant cell pneumonia is rare, except in patients who are immunosuppressed. It is often fatal and characterized by the presence of inclusion bodies and multinucleated giant cells in the respiratory epithelium. Incorrect Answers: A, B, D, and E. Coxsackievirus (Choice A) is an RNA enterovirus which is transmitted via oral secretions or feces. Coxsackievirus type A causes hand, foot, and mouth disease, and herpangina. Coxsackievirus type B may cause myocarditis and pericarditis. Influenza virus (Choice B) is an orthomyxovirus which is transmitted via

57 ---------- Exam Section 2: Item 8 of 50 National, Board of Medical Examiners Comprehensive Basic Science Self-Assessment 8. A 56-year-old man dies 1 month after the onset of difficulty with short-term memory and anxiety. He underwent surgical excision of small cell carcinoma 1 year ago. Degenerative changes are most likely to be concentrated at which of the following labeled sites on the normal MRI of the head? A C A) B) D) E) F)

C. Memory and emotional regulation are mediated by the limbic system, which includes the amygdala, mammillary bodies, stria medullaris, hippocampus, cingulate gyrus, prefrontal cortex, parahippocampal gyrus, ventral tegmentum, and multiple deep nuclei. Label "C" in the axial T2-weighted MRI image shown identifies the mesial temporal lobe, where the hippocampus and parahippocampal gyrus are located. The hippocampus is particularly sensitive to radiation, and degenerative changes or damage of the hippocampus commonly result in short-term memory loss. Chemotherapy has also been shown to injure hippocampal synapses. This patient's symptoms of short-term memory loss, anxiety, and a history of malignancy suggest damage to the limbic system with likely involvement of the hippocampus and mesial temporal lobe. Mesial temporal lobe atrophy is also associated with Alzheimer disease and frontotemporal dementia. Incorrect Answers: A, B, D, E, and F. Choice A identifies the eye and superior orbit, in which rectus muscles and the globe are visible. These structures are involved in vision and do not play a role in memory or emotional regulation. Choice B identifies the region of the frontal pole, inferior frontal gyrus, straight gyrus, olfactory gyrus, and olfactory sulcus. Olfactory sensations are closely related to emotional processing and memory. However, this region is less likely than the mesial temporal lobe to result in short-term memory loss. Choice D identifies the right substantia nigra. The substantia nigra is a nucleus for dopaminergic cells and is critical for the modulation of movement in coordination with the basal ganglia. Pathology affecting this area is associated with Parkinson disease. Choice E identifies the superior cerebellar vermis. Chronic alcoholism is associated with atrophy of this region. Atrophy and lesio

53 ---------- Exam Section 2: Item 4 of 50 National Board of Medical Examiners Comprehensive Basic Science Self-Assessment 4. A 22-year-old woman who has B-thalassemia has a B-hemoglobin allele that carries a single base substitution (A → G) at the splice acceptor site of the second intron. Which of the following events in expression of the B-hemoglobin gene is most likely to be impaired by this mutation? A) Binding of nucleosomal histones to the transcript B) Cleavage of the transcript by RNA ligase C) Modification of the transcript by small nuclear ribonucleoproteins D) Recognition of the transcript by signal recognition particle E) Transport of the transcript out of the nucleus

C. Modification of the transcript by small nuclear ribonucleoproteins (snRNPs) is most likely to be affected by a single base substitution at the splice acceptor site of the second intron of the B-hemoglobin allele. Transcription is the process by which pre-messenger RNA (MRNA) is made from DNÅ, but pre-mRNA must undergo post-transcriptional modification to create mature mRÑA, including the removal of introns and connection of exons. Splice sites exist at the end of an intron and the beginning of an exon, and are recognized by snRŇPs, which are RNA-protein complexes that combine to form a spliceosome. These snRNPs remove the introns, and exons are subsequently spliced together. A mutation in the splice acceptor site can result in the inclusion of introns into the final mRÑA, or exclusion of exons, thereby rendering the MRNA and its translated protein non-functional. In the instance of B-thalassemia, a single abnormal B-hemoglobin gene would result in B-thalassemia minor. Incorrect Answers: A, B, D, and E. Binding of nucleosomal histones to the transcript (Choice A) would not be affected by a mutation of the splice site. Histones are proteins with positively charged amino acids that bind to negatively charged DNA. A complex of histones is called a nucleosome. Unwinding of the nucleosome is required for transcription and occurs via the action of histone acetylase. Cleavage of the transcript by RNA ligase (Choice B) is not affected by mutations in the splice acceptor site. RNA ligases are a class of enzymes that join RNA strands to one another. Recognition of the transcript by signal recognition particle (SRP) (Choice D) occurs after transcription has already occurred. The SRP binds new peptides as they are being synthesized by the ribosome and results in a transient reduction in the pace of translation to facilitate

69 ---------- Exam Section 2: Item 19 of 50 National, Board pf Medical Examiners mont 19. A newborn delivered at 28 weeks' gestation is in severe respiratory distress. The immature alveoli of this newborn's lungs have a diminished ability to serve as sites of effective gas exchange. An increase in which of the following parameters best explains this finding? O AJAlveolar radii O B} Lung compliance C) Lung elastic recoil O D) Pleural pressure O E}Surfactant secretion

C. Neonatal respiratory distress syndrome presents with dyspnea and tachypnea during the first few hours of life, most often occurring in premature neonates. Physical examination reveals evidence of increased respiratory effort with nasal flaring, expiratory grunting, and intercostal retractions. This typically occurs secondary to an insufficient quantity of pulmonary surfactant production related to immature type Il pneumocytes, which occurs in the setting of premature delivery or the decreased stimulation of surfactant production from mature pneumocytes (eg, cesarean delivery with reduced fetal cortisol production in the absence of vaginal compressive stress). Pulmonary surfactant is a phospholipid mixture that lines the alveoli and reduces surface tension. Surface tension is directly proportional to the collapsing pressure experienced by the alveolus. Impaired surfactant production leads to alveolar collapse with two important consequences. The first consequence is a substantially reduced area for gas exchange resulting in hypoxemia, while the secondary consequence is decreased lung compliance, requiring greater force to expand the lungs. Incorrect Answers: A, B, D, and E. An increase in alveolar radii (Choice A) would result in increased lung compliance and decreased lung elastic recoil. This can be observed with the law of Laplace, which describes the pressure on a system (in this case, alveolar collapsing pressure) as a function of surface tension divided by radius. As the radius of the alveolus increases, the collapsing pressure decreases. Lung compliance (Choice B) is decreased in neonatal respiratory distress syndrome. Compliance is the change in volume of a system in response to a change in pressure. The increased lung elastic recoil from collapsed alveoli leads to a decreased lung compliance, as it takes a g

23 ---------- Exam Section 1: Item 24 of 50 National, Board of Medical Examiners Comprehensive Basic Science Self-Assessment 24. A 30-year-old man develops urinary incontinence 2 weeks after successful treatment of a fracture of the left pelvis that was sustained at work. He was pinned against a loading dock by a truck moving in reverse. Physical examination shows a distended bladder. Cystometrography shows absence of micturition reflexes. After the bladder fills to capacity, overflow of urine occurs through the urethra a few drops at a time. This patient most likely sustained additional injury to which of the following during his initial accident? O A) External urinary sphincter B) Hypogastric nerve C) Pelvic nerves D) Pudendal nerves E) Skeletal motor fibers

C. Overflow incontinence is characterized by chronic urinary retention and a chronically distended bladder. When intravesical pressure exceeds outlet resistance, incontinence results as urine flows. It may be caused by physical barriers such as tumors or urethral stricture, neurologic lesions, or as a side effect of some medications (eg, anticholinergics). After pelvic trauma, the pelvic nerve plexus can be damaged. Pelvic parasympathetic nerves in the pelvic plexus function to excite and contract the detrusor muscle of the bladder via muscarinic acetylcholine receptors while sympathetic nerves mediate relaxation of the internal urethral sphincter via a1-adrenergic receptors, leading to normal urination. Damage to the pelvic nerve plexus can affect both bladder contraction and urethral sphincter relaxation, leading to overflow incontinence. Overflow incontinence is treated with managing inciting conditions, timed voiding, or placement of a urethral catheter. Incorrect Answers: A, B, D, and E. The external urinary sphincter (Choice A) is innervated by the pudendal nerve (Choice D). The pudendal nerve supplies sensory neurons to the external genitalia along with somatic skeletal motor fibers (Choice E) to pelvic muscles, including the external urethral sphincter. The pudendal nerve does not carry any parasympathetic fibers. Motor fibers in the pudendal nerve act on the external urethral sphincter via acetylcholine release on nicotinic receptors, causing contraction and urinary retention. Damage to these nerves would result in loss of voluntary control over voiding. The hypogastric nerve (Choice B) contains sympathetic fibers that release norepinephrine and result in relaxation of the detrusor muscle and contraction of the urethral smooth muscle to prevent voiding. Damage to these fibers would lead to total incontinence.

54 ---------- Exam Section 2: Item 5 of 50 National, Board of Medical Examiners Comprehensive Basic Science Self-Assessment 5. A 22-year-old woman comes to the physician because of a 6-month history of frequent headaches and fatigue. She tells the physician that she often overeats when she is anxious and has been making herself vomit and using laxatives for the past 2 years. She is 168 cm (5 ft 6 in) tall and weighs 59 kg (130 lb); BMI is 21 kg/m2. Physical examination will most likely show which of the following conditions? A) Arthritis B) Decreased body hair C) Dental caries D) Galactorrhea E) Pituitary adenoma F) Tachycardia O G) Thyroid gland enlargement

C. Patients with eating disorders such as anorexia nervosa and bulimia nervosa who purge by vomiting may demonstrate dental caries on physical examination. This patient likely has bulimia nervosa (purging type). Bulimia nervosa (purging type) involves cycles of uncontrollable eating and compensatory behaviors such as vomiting, laxative, or diuretic overuse that occur at least once a week over 3 months or more. Unlike patients with anorexia nervosa, patients with bulimia nervosa typically have a normal BMI. Patients with the binging/purging type of anorexia or bulimia nervosa can demonstrate dental caries from gastric hydrochloric acid erosion of enamel, parotid gland swelling, and scars on the knuckles secondary to abrasions from the incisors when inducing vomiting. The loss of gastric hydrochloric acid leads to hypochloremia and metabolic alkalosis. In severe cases, signs of hypovolemia such as tachycardia and hypotension may be present. Treatment of bulimia nervosa is through a combined medical and psychiatric approach and involves correcting fluid and electrolyte derangements alongside behavioral and pharmacologic therapy. Incorrect Answers: A, B, D, E, F, and G. Arthritis (Choice A) is not a known eating disorder complication. Decreased bone mineral density and an increased risk for stress fractures are possible musculoskeletal sequelae of eating disorders, which are related to malnutrition and estrogen deficiency. Decreased body hair (Choice B) is a common physical examination finding in anorexia nervosa resulting from decreased protein stores. The severe malnutrition and associated examination findings in anorexia nervosa are less common in bulimia nervosa. Galactorrhea (Choice D) refers to abnormal, non-physiologic discharge of milk from the nipple. It typically results from hyperprolactinemia, which can occur d

78 ---------- Exam Section 2: Item 28 of 50 National, Board pf Medical Examiners aiveB A ance ement 28. A 78-year-old man comes to the physician with his daughter because of low back pain for 3 months. He underwent a bilateral orchiectomy 2 years ago for prostate cancer. He speaks French fluently, but he knows only a few English words. His daughter offers to serve as an interpreter. After thanking the daughter for her offer, it is most appropriate for the physician to state which of the following? O A) "Have you interpreted for your father before in a medical capacity?" O B) "I believe it would be awkward for a daughter to interpret for her father." C) "It is best that I use a professional French interpreter." O D) "Please ask your father if he's ready to begin the examination." E) "That would be fine. I'll just ask you to step out of the room for the physical examination."

C. Professional interpreters can provide high-quality translations that maintain patient safety. Credentialed interpreters are trained in medical knowledge and vocabulary, ethics, and cultural sensitivity. Relying on a family member for interpreting increases the risk of miscommunication about medical or culturally relevant details and can jeopardize informed consent for treatment and patient safety. Incorrect Answers: A, B, D, and E. Having past experience translating for a family member (Choice A) is suboptimal compared to the rigorous training professional interpreters receive. Irrespective of interpersonal awkwardness or the physician's personal beliefs (Choice B), family members should not act as interpreters for patients (Choices D and E). The nuances of a medical situation may be misunderstood by an untrained interpreter and therefore miscommunicated to the patient. The only situation in which interpretation by a family member is acceptable is an emergency when no other option exists. Educational Objective: Physicians should utilize professional interpreters instead of family members to translate for patients. Professional interpreters are trained to accurately communicate in medical settings and are therefore crucial for patient safety and informed consent to treatment.

61 ---------- Exam Section 2: Item 11 of 50 Natjonal, Board of Medical Examinersment 11. A bronchial smooth muscle preparation is maintained in a tissue bath. Bioactive substances can be applied to the smooth muscle, and contraction and relaxation of the smooth muscle can be measured. The results of administration of thromboxane A2 (TXA2) and prostaglandin E2 (PGE2) are shown. Which of the following is the most likely mechanism of the effect of application of the PGE2? PGE, O A) Activation of phospholipase A2 TXA, O B) Closing K+ channels C) Increase in CAMP O D) Increase in inositol 1,4,5-trisphosphate E) Inhibition of 5-lipoxygenase

C. Smooth muscle cell contraction occurs when myosin light chains are phosphorylated, leading to cross-bridge formations between myosin heads and actin filaments. This reaction is performed by myosin light chain kinase (MLCK). This process is opposed by myosin light chain phosphatase (MLCP), which dephosphorylates the myosin light chains resulting in decreased muscle tone. Prostaglandin E2 (PGE2) stimulates adenylyl cyclase in smooth muscle cells, which leads to an increase in CAMP. CAMP inhibits the activity of myosin light chain kinase, resulting in decreased phosphorylation of myosin light chains, which leads to decreased smooth muscle tension. Incorrect Answers: A, B, D, and E. Activation of phospholipase A2 (Choice A) occurs in proinflammatory states. Phospholipase A2 catalyzes the conversion of phospholipids into arachidonic acid, which is then modified into eicosanoids (such as thromboxane A2) to mediate inflammatory effects. Closing K* channels (Choice B) is not a direct action of PGE2. K* channels are ubiquitous on cell membranes and important in regulating action potentials and membrane depolarization/repolarization. Increase in inositol 1,4,5-triphosphate (Choice D) causes increased smooth muscle tension. Inositol 1,4,5-triphosphate (IP3) is a signaling molecule that increases intracellular calcium release from the sarcoplasmic reticulum of muscle cells. Calcium binds to calmodulin, and the calcium-calmodulin complex activates MLCK, leading to increased muscle contraction. Inhibition of 5-lipoxygenase (Choice E) decreases the production of leukotrienes from arachidonic acid. 5-lipooxygenase inhibitors are commonly used in the treatment of asthma. PGE2 does not affect the activity of 5-lipoxygenase. Educational Objective: Prostaglandin E2 is a potent bronchodilator that acts by stimulating adenylyl cyclase in

16 ---------- Exam Section 1: Item 17 of 50 National Board of Medical Examiners Comprehensive Basic Science Self-Assessment 17. A 22-year-old man is brought to the emergency department 20 minutes after sustaining a knife wound to the right side of his chest in a fight at a local bar. On arrival, he is short of breath. His pulse is 108/min, respirations are 30/min, and blood pressure is 100/60 mm Hg. Physical examination shows that the trachea is deviated to the left. Further physical examination of the traumatized side is most likely to show which of the following pulmonary findings? A) Crackles B) Egophony C) Hyperresonant percussion D) Increased fremitus E) Stridor F) Wheezing G) Whispered pectoriloquy

C. Tension pneumothorax can be a complication of a penetrating thoracic trauma and occurs when air is able to enter the pleural space but is prevented from exiting. Increasing amounts of trapped air in the pleural space compress the lung parenchyma, creating an inability for the lung to expand and causing it to collapse. As the pneumothorax progresses and continues to increase in size, it can result in tracheal and/or mediastinal deviation away from the affected hemithorax. It can also result in decreased venous return to the heart, which manifests with hypotension and obstructive shock. Ďullness in percussion occurs over solid structures such as bones or pleural effusions. Tension pneumothorax is characterized by absent breath sounds and hyperresonant percussion due to the increased air in the intrathoracic space displacing normal lung parenchyma. Incorrect Answers; A, B, D, E, F, and G. Lesions that cause increased consolidation of the lung parenchyma, such as pneumonia, result in rales or crackles (Choice A), egophony (Choice B), dullness to percussion, increased fremitus (Choice D), and whispered pectoriloquy (Choice G). Rales or crackles refers to an abnormal rattling sound with respiration, which may be seen with pneumonia, but is more common in the setting of pulmonary edema. Dullness to percussion as well as decreased fremitus occurs when there is atelectasis or a pleural effusion present. Egophony occurs when there is an increased resonance of voice sounds heard when auscultating the lungs with a stethoscope, which is due to a change in the frequency of sound passing through consolidated lung tissue. Fremitus refers to vibration and tremors that can be palpated on the chest during patient phonation, which increases in lung consolidation and decreases when there is a pleural effusion that is separating the lun

32 ---------- Exam Section 1: Item 33 of 50 National, Board of Medical Examiners Comprehensive Basic Science Self-Assessment 33. A 45-year-old homeless man is brought to the emergency department by police 30 minutes after he was found unconscious. His breath and clothes smell of alcohol. His temperature is 36.8°C (98.2°F), pulse is 68/min, respirations are 14/min, and blood pressure is 110/55 mm Hg. Physical examination shows bronzed skin and spider angiomata on the chest. Laboratory studies show: 10 g/dL 30% Hemoglobin Hematocrit Mean corpuscular volume Leukocyte count Segmented neutrophils Lymphocytes Monocytes Platelet count 110 um3 9000/mm3 70% 20% 10% 160,000/mm3 Serum 200 ng/mL 500 pg/mL (N=160-950) 20 ng/mL (N=166-640) Ferritin Vitamin B12 (cobalamin) RBC folate A peripheral blood smear shows occasional hypersegmented neutrophils and 3+ oval macrocytes. Serum studies are most likely to show which of the foll

D. Folic acid, or vitamin Bg, is converted in the body to tetrahydrofolic acid which functions as a coenzyme in the synthesis of nucleic acids. Folate is contained in leafy vegetables and primarily absorbed in the jejunum. Folate supplementation is recommended in pregnancy to decrease the risk of neural tube defects. It is also commonly warranted in patients with rapid cellular turnover, such as acute leukemia and sickle cell disease, to offset the consumption of folic acid in cell division. Folate deficiency is often seen in patients with malnutrition, alcoholism, and patients taking anti-folate medications (eg, phenytoin, methotrexate). Megaloblastic anemia occurs in the setting of impaired DNA synthesis, most commonly related to folate or vitamin B12 (cobalamin) deficiency and is characterized by erythrocyte macrocytosis and hypersegmented neutrophils. Laboratory evaluation reveals anemia, increased mean cell volume, and normal white cell and platelet indices. The presence of hypersegmented neutrophils on peripheral smear is a characteristic finding in megaloblastic anemia. Since folate and vitamin B12 (cobalamin) are needed for the conversion of homocysteine to methionine, deficiency is associated with elevated homocysteine levels. However, vitamin B12 (cobalamin) also acts as a cofactor for the conversion of methylmalonyl-CoA to succinyl-CoA, and its deficiency is associated with increased methylmalonic acid levels. Therefore, folate deficiency would result in increased homocysteine levels but normal methylmalonic acid levels. Incorrect Answers: A, B, C, E, F, G, H, and I. Increased methylmalonic acid levels (Choices A, B, C, and D) would result from vitamin B12 (cobalamin) deficiency, which would also cause increased homocysteine levels. Vitamin B12 (cobalamin) deficiency presents with macrocytic, megaloblastic a

87 ---------- Exam Section 2: Item 37 of 50 Natjonal, Board of Medical Examinersment andive Rasic Saance 37. A 68-year-old woman comes to the physician because of a 1-year history of severe abdominal pain after meals; she also has had an unintentional 9-kg (20-lb) weight loss during this period. The pain is relieved when she decreases the amount of food that she eats. She has a history of atherosclerosis and underwent triple coronary artery bypass grafting 2 years ago. Physical examination shows a soft, nontender abdomen and an abdominal bruit. Pedal pulses are diminished. The most likely cause of this patient's symptoms is stenosis of which of the following arteries? O A) Greater pancreatic о в) Нерatic O C) Right gastric D) Superior mesenteric O E) Supraduodenal

D. Mesenteric ischemia occurs when there is a reduction in blood flow to the small intestine. It is either acute or chronic, in which acute mesenteric ischemia occurs due to an embolic or thrombotic arterial obstruction, and chronic mesenteric ischemia is caused by chronic atherosclerotic stenosis of the celiac or mesenteric arteries, with the superior mesenteric artery the most commonly affected. Chronic mesenteric ischemia is characterized by recurring episodes of abdominal pain after eating, which is hypothe secondary to mismatch between intestinal blood flow and metabolic demands, or secondary to small intestinal hypoperfusion when blood is shunted to the stomach during meals. Symptoms can be progressive and lead to the avoidance of eating and weight loss. Clinicians should have a high index of suspicion for chronic mesenteric ischemia especially in patients who exhibit other sequelae of atherosclerotic disease, including peripheral arterial disease with diminished pedal pulses or coronary artery disease. ed to be Incorrect Answers: A, B, C, and E. The greater pancreatic artery (Choice A) is a branch of the splenic artery that supplies the pancreatic tail and body. The common hepatic artery (Choice B) supplies the liver, pylorus, duodenum, pancreas, and gallbladder. It branches into the proper hepatic artery, gastroduodenal artery, and the right gastric artery (Choice C), which supplies the lesser curvature of the stomach. The supraduodenal artery (Choice E) is a branch of the gastroduodenal artery that supplies the first and second parts of the duodenum. None of these arteries supply the small intestine and would be unlikely to contribute to the symptoms of chronic mesenteric ischemia. Educational Objective: Mesenteric ischemia occurs when there is a reduction in blood flow to the small intestine. Chronic mesenter

81 ---------- iv Medical Examinersment Exam Section 2: Item 31 of 50 National Board * 31. A strain of Escherichia coli produces a temperature-sensitive tryptophan synthase and therefore requires exogenously supplied tryptophan for growth at 42°C but not at 30°C. Which of the following mutations is most likely responsible for this phenotype? O A) Deletion O B) Frameshift C) Insertion O D) Missense O E) Nonsense

D. Missense mutations are a type of point mutation in which the change of a single base pair forms a codon that results in substitution of a single, different amino acid from that usually encoded. Substitution of a single amino acid may result in a wide range of changes in protein function depending on the position of the substitution. The change in amino acid may result in no or minimal change to protein function or may lead to non-functional or misfolded proteins. The described change in the mutant tryptophan synthase, in which the protein retains normal function except under particular temperature conditions, is a minor change in protein function that is most consistent with a missense mutation. Other types of mutation typically result in more severe changes in protein function. Incorrect Answers: A, B, C, and E. Deletion (Choice A) of a portion of the genetic sequence results in shifts of the reading frame if the deletion does not involve a number of base pairs that is divisible by three. Deletion mutations typically result in more severe alterations of protein function (generally loss of function) compared to missense mutations. Frameshift mutations (Choice B) result from changes to the reading frame after insertion or deletion of a number of base pairs that is not divisible by three. Frameshift mutations typically result in severe loss of function. Insertion mutations (Choice C) involve the addition of base pairs to the genetic sequence and may result in shift of the reading frame if the insertion does not involve a number of base pairs that is divisible by three. Insertion mutations typically result in more severe alterations of protein function compared to missense mutations. Nonsense mutations (Choice E) are point mutations that result in premature stop codons and protein truncation. Nonsense mutations typical

80 ---------- Exam Section 2: Item 30 of 50 National, Board pf Medical Examiners ent 30. A 14-year-old girl is brought to the physician by her mother because of episodes of increasingly severe facial blemishes during the past 6 months. She says that she does not eat high-fat foods or use makeup. Use of an astringent soap has not resolved her symptoms. Menarche occurred at the age of 12 years. A photograph of her face is shown. Which of the following best describes the pathologic mechanism of this patient's condition? O A) Acceleration of the proliferation of cells in the dermis by increased estrogen production O B) Colonization of apocrine sweat glands by fungi O C) Decreased blood flow to skin capillaries by overproduction of elastic fibers D) Follicular epidermal hyperproliferation with excess production of sebum E) Thickening of the basement membrane in response to increased pubertal serum estrogen concentrations

D. The initial lesion of acne vulgaris is a comedo, a hair follicle that has been blocked by keratin debris. The inciting event in comedo formation is hyperproliferation of the epidermis and abnormal keratinization. This is compounded by androgenic stimulation of sebaceous glands associated with the hair follicle, which are together called a pilosebaceous unit, leading to increased sebum production. This increased sebum provides a substrate for bacterial overgrowth of normal skin flora including Štaphylococcus epidermidis and Propionibacterium acnes. With the accumulation of keratin debris, increased sebum production, and bacterial overgrowth, the comedo becomes inflamed and ruptures, causing a papule or cyst to form. Clinically, areas with increased sebaceous activity including the face, upper back, and chest are prone to acne. It is a common disorder that affects the vast majority of adolescents. Treatments are aimed at reducing comedo formation (retinoids), decreasing sebum production (anti-androgens), and mitigating bacterial overgrowth (antibiotics). Incorrect Answers: A, B, C, and E. Androgens, not estrogen (Choice A), contribute to acne vulgaris. While the sebaceous glands reside in the dermis, it is not proliferation of the sebaceous cells but increased production of their secretions that contribute to acne. Bacterial colonization of the pilosebaceous unit, not colonization of the apocrine glands by fungi (Choice B) leads to acne vulgaris. Apocrine glands are located in the axillae, areolae, and anogenital regions. These areas are not typically affected by acne vulgaris.Dermal collagen fibers, rather than elastic fibers (Choice C), increase as a result of scarring. While the majority of collagen fibers in healthy skin are type I collagen, scar is initially created by type III collagen. The basement membrane (C

2 ---------- Exam Section 1: Item 3 of 50 National Board of Medical Examiners Comprehensive Basic Science Self-Assessment 3. A 42-year-old man is struck by a motor vehicle. His only injury is a closed fracture of the proximal tibia. Initial neurovascular examination shows no deficits. Twenty-four hours later, he has increased leg pain and paresthesias in the dorsal space between his first and second toes. The patient begins to pass dark red urine and becomes oliguric. Urinalysis is positive for blood but no erythrocytes are seen on microscopic examination. Which of the following acute disorders is the most likely cause of the renal failure? A) Glomerulonephritis B) Hemolytic-uremic syndrome C) Interstitial nephritis D) Nephrotic syndrome E) Tubular necrosis

E. Tibial fractures present a high risk for compartment syndrome. The fracture results in blood vessel injury and muscle injury, inflammation, and edema. Because the fascia containing the anterior compartment of the leg does not stretch, bleeding and swelling can cause increased pressure in the compartment. This increased pressure in turn inhibits venous drainage, further increasing pressure in the compartment. Eventually the nerve supply and associated arteries are compromised, leading to the classic signs and symptoms of compartment syndrome. Signs and symptoms of compartment syndrome include pain out of proportion to examination findings, pain with passive movement of the muscles, paresthesia, pallor, pulselessness, and paralysis. Compromised blood supply deprives muscle and tissue of oxygen and glucose, leading to tissue ischemia and necrosis. Muscle necrosis leads to rhabdomyolysis, myoglobinuria, and acute renal failure. Evaluation of rhabdomyolysis reveals red or brown urine and urinalysis is typically positive for blood due to the presence of myoglobinuria without microscopic evidence of red blood cells. A complication of rhabdomyolysis is acute kidney injury from acute tubular necrosis secondary to the release of nephrotoxic myoglobin and nonprotein heme pigments. Acute tubular necrosis typically occurs following an ischemic or nephrotoxic insult to the kidneys, which results in loss of the tubular epithelium. Granular, muddy brown casts are common on urinalysis. Compartment syndrome is treated by immediate fasciotomy to decrease compartment pressure and support tissue perfusion. Incorrect Answers: A, B, C, and D. Glomerulonephritis (Choice A) refers to a variety of glomerular diseases, including nephritic and nephrotic syndromes. Nephritic syndromes typically present with acute renal failure associated with h

56 ---------- Exam Section 2: Item 7 of 50 National Board of Medical Examiners Comprehensive Basic Science Self-Assessment 7. A 7-year-old boy is about to undergo an appendectomy. An intravenous catheter needs to be inserted, but the patient is extremely fearful of being stuck with a needle while awake. The most appropriate anesthesia administered by mask to anesthetize this patient quickly would have which of the following characteristics? A} High blood-selubility B} High-cerebrespinal-fluid-solubility C) High lipid solubility D) Low blood solubility E) Low lipid solubility

D. The rate of induction, or onset of anesthesia, with volatile anesthetics relies on blood solubility, cardiac output, and minute ventilation. Induction is primarily dependent on the partial pressure of inhaled volatile anesthetic present in the alveoli. Solubility in this case can be equated with the blood gas partition coefficient, which describes the relative division between two phases that is typically taken by an anesthetic. Low blood solubility increases the speed of induction, as it allows the alveolar concentration to quickly come into equilibrium with the concentration of anesthetic in the blood, preventing further anesthetic from being removed by the blood. Similarly, low cardiac output decreases the amount of blood delivered to the alveoli, and the anesthetic will more quickly equilibrate with the blood that is present. Increases in minute ventilation also speed induction time by delivering more anesthetic to the alveoli, leading to a more rapid increase in the gas concentration. The converse of these statements is true, and induction speed decreases with high blood solubility, high cardiac output, and low minute ventilation. Therefore, choosing a volatile anesthetic agent with low blood solubility will lead to rapid induction of anesthesia. Incorrect Answers: A, B, C, and E. High blood solubility (Choice A) will delay the speed of induction, as it increases the amount of time it takes for the concentration of gas in the alveoli to reach equilibrium with the concentration of anesthetic in the blood. High cerebrospinal fluid solubility (Choice B) would also increase the time required for induction, as anesthetic would be rapidly taken up into the cerebrospinal fluid from the bloodstream, increasing the time it takes for alveolar gas concentration to equilibrate with blood anesthetic concentration. Neither h

68 ---------- Exam Section 2: Item 18 of 50 Natjonal, Board of Medical Examinersment Cell Membrane Phospholipids 18. The illustration shows the arachidonic cascade that is activated when skin is exposed to poison ivy. Within this cascade, which of the following is the major mechanism of analogs of corticosteroids? Phospholipase A, Arachidonic Acid A) Acceleration of the metabolism of arachidonic acid Cyclooxygenase Lipoxygenase O B) Inhibition of receptors mediating selected responses to prostaglandins and leukotrienes Prostaglandins Leukotrienes O C) Nonselective inhibition of cyclooxygenase and lipoxygenase Tissue receptors Tissue receptors O D) Regulation of phospholipase A2 E) Selective inhibition of lipoxygenase

D. Topical corticosteroids, such as triamcinolone, betamethasone, and clobetasol, are the treatment of choice for allergic contact dermatitis. These medications are analogues of the corticosteroids produced by the zona fasciculata of the adrenal gland. While corticosteroids decrease levels of virtually all cytokines, in the arachidonic cascade they specifically inhibit phospholipase A2. They also inhibit NF-KB, a positive regulator of cyclooxygenase (COX) 2 and thus indirectly inhibit COX2. Allergic contact dermatitis is a type IV hypersensitivity reaction caused by exposure of the skin to either an irritant or allergen causing an eczematous rash to occur in the distribution of the exposure. The role of topical corticosteroids in decreasing cytokines is helpful because in this type of hypersensitivity, sensitized helper T cells rely on cytokines to cause inflammation and attract macrophages. By decreasing production of cytokines, the inflammatory response and therefore the degree of symptom severity is significantly reduced. Incorrect Answers: A, B, C, and E. Acceleration of the metabolism of arachidonic acid (Choice A) would increase the rate of production of prostaglandins and leukotrienes. Prostaglandins and leukotrienes increase neutrophil chemotaxis and decrease vascular tone, respectively, which both facilitate the inflammatory response to an insult. This would cause worsening of inflammation, not improvement. Inhibition of receptors mediating selected responses to prostaglandins and leukotrienes (Choice B) is a mechanism used by several medications, such as montelukast. Montelukast is an example of a leukotriene receptor antagonist that is selective for leukotrienes C4, D4, and E4. This medication is used to prevent smooth muscle contraction and airway edema in asthma. However, inhibition of these or prostagland

89 ---------- Exam Section 2: Item 39 of 50 Natjonal, Board of Medical Examinersent 39. A 76-year-old woman comes to the physician because of a 2-day history of abdominal pain, nausea, and vomiting. Her temperature is 38.9°C (102°F), pulse is 88/min, respirations are 26/min, and blood pressure is 117/79 mm Hg. Abdominal examination shows tenderness of the left lower quadrant. Laboratory studies show a leukocyte count of 22,500/mm3 with 8% bands. A CT scan of the abdomen shows an abscess in the left lower quadrant. The patient undergoes drainage of the abscess and a partial sigmoidectomy for a ruptured diverticulum. Neutrophil activation in this patient is directed, in part, by bacterial lipopolysaccharides. As a result, these lipopolysaccharides most likely bind to which of the following neutrophil receptor types? O A) Cytokine O B) G protein-linked O C) lon channel-linked D) Nuclear E) Toll-like

E. Acute diverticulitis presents due to infection or inflammation of existing colonic diverticula, which are small outpouchings of the bowel wall, most commonly located in the sigmoid colon. Diverticulitis presents with acute left lower quadrant abdominal pain, sometimes accompanied by lipopolysaccharide binding to toll-like receptors on the neutrophil surface. Toll-like receptors bind to pathogen-associated molecular patterns (PAMPS) such as bacterial lipopolysaccharides and activate the NF-KB pathway. NF-KB is a transcription factor that induces the expression of several proinflammatory genes, promoting the production of cytokines, chemokines, and other inflammatory mediators. diarrhea, fever, and tenderness to palpation in the left lower quadrant. Complications of acute diverticulitis include abscess or fistula formation, and perforation (rupture). The innate immune response to acute diverticulitis includes neutrophil activation Incorrect Answers: A, B, C, and D. Cytokine (Choice A) receptor binding modulates the response of neutrophils to infection. Proinflammatory cytokines include interleukin-1 (IL-1), 6, 8, 12 and 18, interferons, and tumor necrosis factor. Anti-inflammatory cytokines include IL-4, 10, 11, and 13. Lipopolysaccharides bind to toll-like receptors on neutrophils, not cytokine receptors. G protein-linked (Choice B) receptors are involved in neutrophil chemotaxis. Ligands include leukotriene B4, platelet-activating factor, and complement fragment C5a. lon channel-linked (Choice C) receptors are involved in modulating several stages of neutrophil recruitment. They do not bind with pathogen-associated molecular patterns. Nuclear (Choice D) receptors are ligand-related transcription factors that control gene expression in response to external signals. Lipopolysaccharides bind to extracellular toll-like

27 ---------- Exam Section 1: Item 28 of 50 National Board of Medical Examiners Comprehensive Basic Science Self-Assessment 28. A female newborn is delivered vaginally at 37 weeks' gestation after an uncomplicated pregnancy, labor, and delivery. Physical examination shows a narrow cranium that is elongated in the anteroposterior dimension. Premature closure of which of the following during embryogenesis is the most likely cause of this anomaly? A) Bregma B) Coronal suture C) Lambda D) Lambdoid suture E) Sagittal suture

E. Craniosynostosis refers to the premature ossification that occurs between the plates of the cranial bones leading to deformity of the skull. Most infants with this condition are asymptomatic. Genetic etiologies may involve abnormalities in expression of FGF and TGF. Normally, the skull is composed of five independent bones. These include the two frontal bones, two parietal bones, and the occipital bone. The fontanelles are cartilaginous gaps that exist between these bony plates that, along with the cartilaginous sutures, allow for movement, growth, and compression of the cranial bones particularly during childbirth when the skull passes through the pelvis and vaginal canal. Closure of one or more of these regions prematurely leads to abnormal growth and fusion of the skulI. The resulting deformity can be both cosmetic (abnormal head shape) and pathologic (limiting normal central nervous system growth). A number of patterns of deformity occur depending on which sutures prematurely fuse. In this patient, fusion of the sagittal suture arrests the medial-lateral growth of the skull. This leads to continued anterior-posterior growth in disproportion to medial-lateral growth. A narrow, elongated anteroposterior cranium results, known as scaphocephaly. Incorrect Answers: A, B, C, and D. The bregma (Choice A) is the point at which the sagittal suture and the coronal suture meet. It is the anterior counterpart to the lambda. Premature closure of the coronal suture (Choice B) leads to brachycephaly, which is a short, broad, flattened skull. This results in arrest of the anterior-posterior growth of the skull. Lambda (Choice C) is the landmark where the sagittal suture intersects the lambdoid sutures, that is, the point at which the two parietal bones meet the occipital bone. A unilateral lambdoid suture (Choice D) craniosynos

31 ---------- Exam Section 1: Item 32 of 50 National, Board of Medical Examiners Comprehensive Basic Science Self-Assessment 32. A 55-year-old man comes to the physician because of a 2-month history of decreased appetite; he also has had a 9-kg (20-lb) weight loss and an intermittent rash during this period. Physical examination shows necrolytic migratory erythema over the axillae and groin. Laboratory studies show a serum glucose concentration of 280 mg/dL and plasma glucagon concentration of 1500 pg/mL (N=20-100). A CT scan of the abdomen shows a mass at the head of the pancreas. Which of the following processes is most likely occurring in this patient's liver? A) Decreased activity of carnitine acetyltransferase O B}Glycogen synthesis C) Increased activity of acetyl-CoA carboxylase D) Inhibition of ketogenesis E) Partial oxidation of fatty acids

E. Glucagonoma is a rare functional malignancy that secretes glucagon, which raises blood glucose. Occasionally, glucagonoma can present as a component of multiple endocrine neoplasia. Symptoms are nonspecific and include mild diabetes mellitus, paraneoplastic rash (eg, necrolytic migratory erythema), and weight loss. Glucagon acts on hepatocytes via a CAMP pathway activating protein kinase A that ends in the activation of glycogen phosphorylase, with release of glucose monomers into serum. When a glucagonoma is present, glucagon is continuously secreted, unlike in the physiologic state. When the glycogen stores are exhausted, the body then relies on the breakdown of fat through beta oxidation of fatty acids to provide energy. Beta oxidation occurs in the mitochondria of the cell. In each cycle of beta oxidation, two carbon atoms are cleaved to form acetyl-ČOA, which leaves the mitochondria via the carnitine shuttle. In addition to a molecule of acetyl-CoA, which then enters the citric acid cycle, a molecule of NADH is garnered with each cycle. The ability to generate energy from the body's fat stores during a period of starvation is critical to maintaining the function of the vital organs, such as the brain. Incorrect Answers: A, B, C, and D. Carnitine acetyltransferase (Choice A) is a transmembrane protein found on the surface of mitochondria in liver, muscle, and brain, and catalyzes the bond between acyl-CoA and carnitine, which permits the long-chain fatty acid to translocate into the mitochondrial matrix where beta-oxidation (breakdown of the fatty acid) occurs. Its activity is increased in the presence of glucagon, not decreased. Glycogen synthesis (Choice B) occurs when there is a surplus of glucose in order to store energy for future use. In contrast, a glucagonoma secretes glucagon which signals a state of s

3 ---------- Exam Section 1: Item 5 of 50 National, Board of Medical Examiners Comprehensive Basic Science Self-Assessment 5. A 65-year-old woman has ascites. Which of the following additional findings indicates a diagnosis of constrictive pericarditis rather than cirrhosis? O A) Edema of the lower extremities B) Esophageal varices C) Hypoalbuminemia D) Hyponatremia E) Increased jugular venous pressure F) Splenomegaly

E. Increased jugular venous pressure (JVP) is an expected finding in constrictive pericarditis (CP) and would not be found in patients with volume overload secondary to cirrhosis. CP describes a pathologic state whereby the pericardium, which encases the entirety of the heart and the origins of the great vessels, loses its elasticity. This can occur in patients with viral infections, connective tissue disease, tuberculosis, or as a result of cardiac surgery or radiation. During the normal cardiac cycle, increased venous return to the right atrium (RA) and right ventricle (RV) during inspiration leads to transient expansion of the RV with slight bowing of the interventricular septum into the left ventricle (LV). This increased RV preload does not impair LV filling as the pericardial sac expands to accommodate the increased RV volume. In constrictive pericarditis, the pericardial sac loses its elasticity. When venous return to the right heart increases, the pericardial sac is unable to expand, which exacerbates movement of the interventricular septum into the LV. This impairs diastolic filling of the LV and reduces cardiac output. LV diastolic filling is further reduced by a reduction in preload from the pulmonary veins. The constricted pericardium does not respond to normal changes in intrathoracic pressure during inspiration, but the pulmonary venous system, which lies outside of the pericardium, experiences a normal drop in pressure during inspiration. This difference creates an abnormal pressure gradient that reduces LV preload and leads to reduced cardiac output. The RV and LV develop interventricular dependence whereby increased pressure in each ventricle begins to affect the other ventricle. CP ultimately results in equalization of pressures in all four chambers, with clinical evidence of right heart failure exhib

43 ---------- Exam Section 1: Item 44 of 50 National, Board of Medical Examiners Comprehensive Basic Science Self-Assessment 44. A 62-year-old man undergoes a physical examination on his arrival at the penitentiary to serve his prison sentence. A routine PPD skin test shows a 14-mm area of induration at 48 hours. A chest x-ray is shown. Reciprocal activation of the two immune cell types involved in the pathogenesis of this patient's pulmonary lesions is mediated by interferon-gamma and which of the following cytokines? O A) Interleukin-4 (IL-4) B) IL-5 C) IL-7 D) IL-10 E) IL-12

E. Interleukin-12 (IL-12) and interferon-gamma activate the two primary cells responsible for granuloma formation in this patient with Mycobacterium tuberculosis (MTB) infection. IL-12 is an inflammatory cytokine secreted by macrophages that are infected with MTB, an intracellular pathogen. IL-12 acts on both natural killer cells (NK cells) and cytotoxic T lymphocytes, causing them to release interferon-gamma, which has several roles including the promotion of Th1 T-cell differentiation, the augmentation of antigen presentation by macrophages, and the differentiation of macrophages into multinucleated giant cells. Collectively, this activating pathway of IL-12 and interferon-gamma allow for granuloma formation, a histologic hallmark of infection with MTB, isolating the invasive pathogen. Contrary to other diseases that result in granuloma formation, MTB infection often results in caseating (necrotic) granulomas. Incorrect Answers: A, B, C, and D. Interleukin-4 (Choice A) induces differentiation of helper T cells to Th2 cells, not Th1 cells. It is important in the process of activating macrophages to assist with wound repair and fibrosis, not granuloma formation. IL-5 (Choice B) is secreted by Th2 helper T cells and mast cells. It activates B lymphocytes to secrete IgA. IL-7 (Choice C) is secreted by numerous types of cells and is important in the maturation of both B and T lymphocytes but does not play a role in granuloma formation. IL-10 (Choice D) is secreted by Th2 and regulatory T cells. It has myriad effects but overall decreases the inflammatory response. It downregulates the expression of MHC class II antigens and Th1 cytokines. Additionally, it inhibits the activity of activated macrophages, which would decrease granuloma formation. cational Objective: MTB infects macrophages and results in granuloma formation

64 ---------- Exam Section 2: Item 14 of 50 Natjonal, Board of Medical Examinersment encive Rasic S ance V 14. An 18-month-old boy is brought to the emergency department because of lethargy for 3 hours. He has not eaten well for the past 24 hours. He had cardiorespiratory arrest associated with hypoglycemia after an episode of diarrhea 10 months ago. On arrival, he is unresponsive. His temperature is 37°C (98.6°F), pulse is 140/min, respirations are 25/min, and blood pressure is 100/60 mm Hg. Physical examination shows hepatomegaly. Laboratory studies show: Serum Glucose Ketones Carnitine Urine Ketones decreased decreased decreased decreased Dicarboxylic acids present Following intravenous administration of glucose, he becomes responsive. Medium-chain triglycerides are administered daily for 6 months. At a follow-up examination, his serum glucose concentration is within the reference range. A deficiency of which of

E. Long-chain acyl-CoA dehydrogenase is an enzyme involved in the beta-oxidation of long-chain fatty acids. Deficiency of this enzyme is an autosomal recessive disorder and results in the accumulation of long-chain fatty acids in the mitochondrial matrix. It is characterized by two forms: an early-onset form that results in severe, life-threatening cardiomyopathy, and a late-onset form that is characterized by episodes of symptomatic hypoglycemia. Signs and symptoms may include irritability, lethargy, hepatomegaly (due to excess fat accumulation), and hypoglycemia. Symptomatic episodes can be triggered by illness, fasting, stress, or exercise. Hypoglycemia occurs without the production of ketones due to impaired fatty acid metabolism and ketone body production since there is deficient production of acetyl-COA from long-chain fatty acids. Immediate treatment involves correction of the patient's hypoglycemia and long-term management involves preventing acute episodes of hypoglycemia and dietary supplementation with medium-chain fatty acids, which will utilize medium-chain acyl-CoA dehydrogenase for beta-oxidation, circumventing the patient's enzymatic deficiency for long-chain fatty acid beta-oxidation. Incorrect Answers: A, B, C, D, and F. Glucose 6-phosphatase (Choice A) is an enzyme located in the liver that functions in gluconeogenesis and converts glucose-6-phosphate to glucose for its release into the blood. Impaired gluconeogenesis can lead to severe hypoglycemia during periods of fasting. Glycogen storage disease type I (von Gierke disease) is secondary to glucose 6-phosphatase deficiency and is characterized by excess glycogen in the liver, fasting hypoglycemia, increased levels of triglycerides, uric acid, and hepatomegaly. HMG-COA lyase (Choice B) is an enzyme that converts HMG-CoA into acetoacetate in the mit

20 ---------- Exam Section 1: Item 21 of 50 National Board of Medical Examiners Comprehensive Basic Science Self-Assessment 21. A 35-year-old woman, gravida 3, para 2, develops a massive hemorrhage after the vaginal delivery of a healthy female newborn at 38 weeks' gestation. She underwent a cesarean delivery 2 years ago because of cephalopelvic disproportion. A hysterectomy is required at this time to control the bleeding. The gross and microscopic appearances of the uterus are shown. Which of the following is the most likely cause of the postpartum hemorrhage in this patient? A) Abruptio placentae B) Ectopic pregnancy O C) Endometriosis D) Leiomyomata uteri E) Placenta accreta F) Placental site trophoblastic tumor

E. Placenta accreta refers to the abnormal attachment of the placenta to the myometrium, rather than just the endometrium. This can be seen in the gross specimen and histology presented in this case. It most commonly presents with postpartum hemorrhage due to retained products of conception but is also commonly recognized on routine prenatal ultrasound examination. Risk factors include prior cesarean delivery or uterine surgery, increased maternal age, placenta previa, and multiparity. Management of known placenta accreta consists of elective cesarean delivery as early as 34 weeks, potentially followed by a hysterectomy depending on the extent of the accreta and retained products of conception. Incorrect answers: A, B, C, D, and F. Abruptio placentae (Choice A) presents with vaginal bleeding, severe uterine pain, and tetanic contractions, typically in the third trimester prior to delivery. It commonly results from lower abdominal trauma, smoking, or cocaine use. It is a less likely cause of postpartum hemorrhage in this patient with prior uterine instrumentation. Ectopic pregnancy (Choice B) occurs when the fertilized embryo implants in a location outside of the uterine cavity. Rupture of an ectopic pregnancy can cause vaginal bleeding and abdominal pain. An ectopic pregnancy is an unlikely cause of postpartum hemorrhage in this patient, as she delivered a healthy term baby. Endometriosis (Choice C) is the development of endometrial tissue outside of the uterine cavity. It can cause abnormal vaginal bleeding, pelvic pain, dyspareunia, and pain with defecation (dyschezia), but does not cause postpartum hemorrhage. Leiomyomata uteri (fibroids) (Choice D) are benign tumors of the uterus, which can present with abnormal uterine bleeding and pelvic pain. However, they are more common in older women and are associated with a

47 ---------- Exam Section 1: Item 48 of 50 National, Board of Medical Examiners Comprehensive Basic Science Self-Assessment 48. A 23-year-old woman is brought to the emergency department because of shortness of breath for 2 weeks. Her respirations are 28/min. Physical examination shows no other abnormalities. Laboratory studies show: Serum Na+ K+ 135 mEq/L 4.0 mEq/L 110 mEq/L 15 mEq/L CI- HCO3- Arterial pH on room air 7.25 Urine Sodium Chlorine 20 mEq/L 30 mEq/L 15 mEq/L Potassium Which of the following is the most likely diagnosis? O A) Alcoholic ketoacidosis B) Crohn disease C) Diabetic ketoacidosis D) Lactic acidosis E) Renal tubular acidosis F) Salicylate poisoning

E. Renal tubular acidosis (RTA) results in a hyperchloremic, non-anion gap metabolic acidosis from the dysregulation of the renal tubules causing the loss of bicarbonate or inability to excrete hydrogen ions. Metabolic acidosis can present with a compensatory respiratory alkalosis, manifesting as tachypnea and shortness of breath. Types of RTA vary in etiology and pathophysiology, but all result in a non-anion gap metabolic acidosis. RTAS are not due to the accumulation of an unmeasured anion and do not present with an increased anion gap. This patient demonstrates a low bicarbonate level, with a normal anion gap of 12, consistent with a non-anion gap acidosis. Distal RTA (type 1) results from a defect in the renal secretion of H*, which results in the lack of HCO3 generation, ultimately leading to a metabolic acidosis. It is characterized by a urine pH greater than 5.5. Some common causes of type 1 RTA include analgesic nephropathy, congenital obstructive urinary tract anomalies, medications, and autoimmune disease. Incorrect Answers: A, B, C, D, and F. Alcoholic ketoacidosis (Choice A) refers to a starvation state induced by excess alcohol intake that results in increased ketone production and an anion gap metabolic acidosis due to alterations in the ratio of NAD+ to NADH. Diabetic ketoacidosis (Choice C) typically occurs in patients with type 1 diabetes mellitus as a result of increased insulin requirements during times of physiologic stress (eg, infection) or insulin nonadherence. Diabetic ketoacidosis results in a metabolic acidosis with an increased anion gap. Lactic acidosis (Choice D) can result from any hypoperfusion state, hypoxia, deficiency in the citric acid cycle, impaired oxidative phosphorylation, or from vitamin deficiencies (eg, thiamine), and is associated with an increased anion gap metabolic acidos

8 ---------- Exam Section 1: Item 8 of 50 National, Board of Medical Examiners Comprehensive Basic Science Self-Assessment 8. A 47-year-old man with a history of rheumatic valvular disease comes to the physician because of chest pain and difficulty breathing for the past 2 days. An ECG shows an acute myocardial infarction. Examination shows a systolic murmur. Pressure tracings from the aorta, left ventricle (LV), and left atrium (LA) are shown. Which of the following is the most likely diagnosis? O A) Aortic obstruction Aortic B) Aortic regurgitation C) Left ventricular aneurysm D) Mitral obstruction LV E) Rupture of the chordae tendineae LA Time

E. Rupture of the chordae tendineae with development of mitral regurgitation accounts for the findings depicted in the graph. The diagram depicts changes in pressure within the aorta, LV, and LA over a single cardiac cycle. Normally the LV will undergo isovolumetric contraction with expulsion of blood through the aortic valve once the pressure inside the LV exceeds the aortic pressure. When pressure in the aorta (diastolic pressure) exceeds or equals the LV pressure, the aortic valve closes and the LV is able to relax, corresponding to a rapid decrease in pressure. This will allow for the ventricle to fill during diastole. During ventricular systole, the LA should demonstrate only a minimal increase in pressure because a competent mitral valve prevents regurgitation of blood back into the LA. However, if the mitral valve is incompetent (mitral regurgitation), blood from the LV will enter the LA through the mitral valve. This is depicted in the diagram as an increase in pressure in the LA during LV systole, which is abnormal. Following a myocardial infarction, a common cause of mitral valve regurgitation is rupture of a papillary muscle and associated chordae tendineae, which typically anchor the leaflets of the mitral valve to the ventricles and prevent regurgitation during ventricular systole. When ruptured, there is no support structure to keep the mitral valve leaflets closed, so the increased LV pressure forces them back into the LA with consequent development of mitral regurgitation. Depending upon the severity, this can result in acutely decompensated heart failure, and surgical repair may be required. Incorrect Answers: A, B, C, and D. Aortic obstruction (Choice A) from aortic stenosis means that the LV must generate a much higher pressure to overcome the stenotic valve and effectively eject blood into the aorta

72 ---------- Exam Section 2: Item 22 of 50 Natjonal, Board of Medical Examinersment. 22. Receptors for all of the members of the steroid hormone family share which of the following features? O A) Calcium-mediated intracellular effects O B) GTP-binding proteins O C) Mitochondrial membrane association O D) Plasma membrane association E) Zinc-containing DNA-binding domains

E. Steroid hormones act on intracellular receptors and have the ability to traverse the cell membrane due to their lipophilic nature. Some of these hormones include cortisol, estrogen and progesterone, testosterone, and aldosterone. All of these molecules are derived from cholesterol as a common precursor. Steroid hormone receptors are intracellular and reside either in the cytoplasm or the nucleus and regulate gene expression. Because of the effects mediated through gene regulation, the onset of action of steroid hormones occurs over the course of hours to days. Intracellular steroid receptors have a number of different protein domains. The DNA-binding domain classically has a zinc finger motif as a tertiary structure. This domain has affinity with the regulatory (eg, promoter, silencer) regions of DNA. Incorrect Answers: A, B, C, and D. Activation of Go-protein coupled receptors leads to an increase in calcium-mediated intracellular effects (Choice A). The signaling cascade of Gg includes activation of phospholipase C, which cleaves membrane bound phospholipids, leading to the formation of inositol triphosphate (IP3) and diacylglycerol (DAG). This results in the subsequent increase in intracellular calcium concentration, which then activates protein kinase C and/or smooth muscle contraction. GTP-binding proteins (Choice B) are also known as G proteins which are components of many transmembrane receptors. G proteins enter an activated state when they are bound to GTP molecules. Once the GTP molecule is cleaved to GDP, the G protein becomes inactive. G proteins are commonly involved in signaling pathways of peptide hormones. Mitochondrial membrane association (Choice C) is a description of pathways of cellular apoptosis. During activation, caspases will cleave proteins causing permeability of the cell membrane. In addi

46 ---------- Exam Section 1: Item 47 of 50 National Board of Medical Examiners Comprehensive Basic Science Self-Assessment 47. A 56-year-old man is brought to the emergency department 1 hour after having a generalized tonic-clonic seizure during his morning run. Vital signs are within normal limits. MRIS of the brain show a mass in the lateral aspect of the precentral gyrus of the frontal lobe. Based on the MRI findings, which of the following additional deficits is most likely in this patient? A) Atrophy of the left upper extremity B) Decreased proprioception in the right upper extremity C) Decreased sensation in the left lower extremity D) Deviation of the tongue to the right E) Weakness of the left lower area of the face F) Weakness of the right lower extremity

E. The precentral gyrus of the frontal lobe contains the primary motor cortex that controls voluntary movements of the contralateral body. Medially, the precentral gyrus contains neurons that control the lower extremity. Laterally, the precentral gyrus controls the contralateral face. The contralateral arm is controlled by neurons between the two areas. This distribution of control is known as somatotopy and permits localization of physical examination findings to a particular brain region. In this case, a lesion in the lateral aspect of the right precentral gyrus would disrupt motor neurons controlling the left face, resulting in weakness, facial droop, and an upper motor neuron pattern of dysfunction (eg, hyperreflexia). Importantly, the forehead would be spared, as the facial nerve (cranial nerve VII) nucleus that controls forehead musculature is dually innervated by upper motor neurons from bilateral precentral gyri. Because of this, the patient would be expected to symmetrically raise his eyebrows. Alternatively, lesions involving the lower motor neurons of facial expression (eg, facial nerve inflammation in Bell palsy) affect the forehead and lower face together. Incorrect Answers: A, B, C, D, and F. Atrophy of the left upper extremity (Choice A) may be associated with an upper motor neuron lesion located in the right precentral gyrus that would be located medially relative to the location of this patient's mass. The contralateral arm is innervated by neurons located in the parasagittal region of the precentral gyrus. As well, atrophy of the left upper extremity may be associated with lower motor neuron pathology in the ipsilateral spinal cord, spinal nerves, or peripheral nerves. Decreased proprioception in the right upper extremity (Choice B) or decreased sensation in the left lower extremity (Choice C) could b

88 ---------- Exam Section 2: Item 38 of 50 Natjonal, Board of Medical Examinersment. 38. A 52-year-old woman with type 2 diabetes mellitus and hypercholesterolemia comes to the physician for a follow-up examination. Since her diagnosis 14 months ago, she has been exercising regularly, eating healthier foods, and taking omega-3 fatty acid supplements. She has had a 9-kg (20-lb) weight loss during this period. She is 168 cm (5 ft 6 in) tall and now weighs 62 kg (137 Ib); BMI is 22 kg/m2. Physical examination shows no abnormalities. Serum studies show decreased glucose and cholesterol concentrations since her last visit 2 months ago. She says, "I'm glad everything is fine, but I am worried that I won't be able to stay on this regimen and that my health will get worse." Which of the following best describes this patient's stage of change? O A) Precontemplation O B) Contemplation O C) Preparation D) Action E) Maintenance

E. This patient has already made significant behavioral changes, resulting in improved health parameters, but is concerned she may revert to her previous unhealthy diet and exercise habits. This scenario is consistent with the maintenance stage of behavioral change. The physician and patient may collabora a health-related behavior such as treatment adherence, substance use, diet, or exercise habits. In sequential order, the stages of behavioral change are precontemplation, contemplation, preparation, action, maintenance, and termination. Physicians aim to move patients through these stages over time with an interview technique called motivational interviewing. Motivational interviewing involves using open-ended, non-judgmental questions to help the patient explore their reasons for wanting to change or maintain the habit. vely strategize about how to avoid relapse, which may involve bolstering skills for coping with temptation or joining a diabetes support group. The stages of behavioral change are used to define a tient's readiness to change Incorrect Answers: A, B, C, and D. Patients in the precontemplation stage (Choice A) are not interested in changing their habit and may not see the benefit in making a change. This patient is aware of the benefits of exercise and a healthy diet and has already changed her habits, which signifies she has surpassed the precontemplation stage. Patients in the contemplation stage (Choice B) are typically aware of the benefits of the behavior change but may be ambivalent or indecisive about change. This patient has already improved her diet and exercise habits, which signifies that she is beyond the contemplation stage. Patients in the preparation stage (Choice C) have committed to making a change and are ready to discuss strategies and resources to help them make the change. For examp

62 ---------- Exam Section 2: Item 12 of 50 Natjonal, Board of Medical Examinersment 12. A 25-year-old man comes to the physician because of sores on his penis for 2 days. Physical examination shows vesicular and ulcerative lesions on the penis. Drug therapy is initiated. One week later, his symptoms resolve. Which of the following enzymes is required to activate the most commonly prescribed drug? O A) ATPase O B) Cytochrome P450 O C) Protease O D) Reverse transcriptase E) Thymidine kinase

E. This patient's vesicular and ulcerative lesions on the penis are likely caused by herpes simplex virus in the form of genital herpes. Treatment for herpetic infections involves drugs that inhibit viral DNA polymerase, classically by guanosine analogs such as acyclovir, valacyclovir, and famciclovir. Prior to exerting their antiviral effects, guanosine analogs must be phosphorylated by the viral enzyme thymidine kinase. Phosphorylation is the process by which a phosphate group is added to the molecule. These molecules are then able to inhibit the viral DNA polymerase by terminating the nascent DNA chain during replication. These drugs are effective against herpes simplex virus and varicella zoster virus, weakly effective against Epstein-Barr virus, and not effective against cytomegalovirus. Development of a mutation in the viral thymidine kinase enzyme would prevent drug phosphorylation and confer resistance to guanosine analog medications. Incorrect Answers: A, B, C, and D. ATPases (Choice A) are a heterogenous group of enzymes that hydrolyze ATP into ADP and a molecule of inorganic phosphate. Guanosine analogs require phosphorylation, not dephosphorylation, in order to be active. Cytochrome P450 (Choice B) enzymes are found in the liver and play a key role in the metabolism of many drugs. In most cases, this process alters a drug from its active form to an inactive metabolite. However, some drugs are converted from their prodrug form into an active metabolite by the P450 enzymes, such as clopidogrel, but guanosine analogs are not among them. Proteases (Choice C) cleave the initial polypeptides produced by the translation of viral RNA into smaller, functional parts. Protease inhibitors are used in the treatment of HIV and include darunavir, indinavir, ritonavir, and saquinavir. Reverse transcriptase (Choice D) trans

36 ---------- Exam Section 1: Item 37 of 50 National Board of Medical Examiners Comprehensive Basic Science Self-Assessment 37. A 20-month-old girl is brought to the physician by her mother because the mother is concerned about her daughter's development and behavior. The girl is able to walk and run and has a vocabulary of at least 100 words. However, when she attends playdates with other children, she prefers to play by herself and will strike another child on the head if that child attempts to join her in playing with a particular toy. Which of the following best explains this patient's actions during play? A) Attention-deficit/hyperactivity disorder B) Conduct disorder C) Early sign of autistic disorder D) Oppositional defiant disorder E) Normal behavior for age

E. This toddler demonstrates normal behavior for age with age-appropriate gross motor, language, and social functioning. Children are expected to be able to walk up stairs by age 18 months and run by age 2 years, and the typically developing 18-month-old has a vocabulary of approximately 100 words. Social skills may include pointing to show others something interesting, playing pretend, and showing affection to familiar others. Cooperative play is not expected to develop until age 4 years, so this child's inclination not to share her toy is typical. Hitting and aggression are relatively common in toddlers. Typical fine motor skills at this age include stacking several blocks. Incorrect Answers: A, B, C, and D. Attention-deficit/hyperactivity disorder (Choice A) is a childhood disorder that presents with hyperactivity, impulsivity, and/or inattention. Though this child impulsively hits others, this behavior is common for her age. This child does not demonstrate hyperactivity or inattention. Conduct disorder (Choice B) refers to persistent emotional and behavioral problems such as purposeful aggression toward people and animals in children and adolescents. The behavioral problems can also include lying and serious rule violations. This child's aggression is borne out of age-appropriate impulsivity rather than intentional bullying. Early signs of autistic disorder (Choice C) may include deficits in social and language skills, sensory overstimulation, and frequent tantrums. This patient has age-appropriate social and language skills. Oppositional defiant disorder (Choice D) manifests as a persistent pattern of angry mood, defiant behavior, and vindictiveness that causes distress and impaired functioning in the child or their caretakers. This child's aggression is borne out of age-appropriate impulsivity rather than defianc

t disease) O D) Sarcoidosis E) Tuberculosis

E. Tuberculous disease of the spine, also known as Pott disease, is a spinal osteomyelitis or osteodiscitis caused by Mycobacterium tuberculosis. M. tuberculosis is a slow-growing organism transmitted via respiratory secretions. The slow growth and evasion of immune defenses result in the disease presenting with primary, latent, and reactivation patterns. In the primary infection, patients present with subacute fevers, weight loss, night sweats, cough, and malaise. Latent tuberculosis can occur as the organism proliferates within macrophages and is maintained within caseating granulomas. Although tuberculosis is primarily a disease of the respiratory tract, it can have a number of extrapulmonary manifestations. These include meningitis, dermatologic tuberculosis, and vertebral discitis or osteomyelitis. In this case, the vertebral body and intervertebral disc are involved. Radiographs or CT/MRI imaging will demonstrate localized destruction of bony anatomy, potentially with compression of the spinal nerve roots or spinal cord. Because of its airborne transmission pattern, patients with tuberculosis must be isolated to prevent transmission to other persons. Tuberculosis remains prevalent in homeless, incarcerated, and immunocompromised patients. Active disease is treated primarily with a combination of rifampin, isoniazid, pyrazinamide, and ethambutol. Incorrect Answers: A, B, C, and D. Hyperparathyroidism (Choice A) leads to bony resorption and hypercalcemia through increased production of parathyroid hormone. Primary hyperparathyroidism results from functionally active parathyroid adenoma, hyperplasia, or carcinoma. Secondary hyperparathyroidism is commonly due to chronic hyperphosphatemia and hypocalcemia in the setting of end-stage renal disease. As parathyroid hormone activates osteoclasts to maintain calcium level

71 ---------- Exam Section 2: Item 21 of 50 National Board of Medical Examiners mont 21. A 3-year-old girl is brought to the physician because of a 2-week history of easy bruisability. She has had frequent, large, foul-smelling stools for 3 months. She was found to have cystic fibrosis at the age of 18 months. She is below the 3rd percentile for height and weight. Physical examination shows several ecchymotic areas over the extremities. This patient most likely has a deficiency of which of the following? O A) Antithrombin III O B) Platelets O C) Protein C O D) Vitamin B12 (cobalamin) O E) Vitamin C F) Vitamin K O G) von Willebrand factor

F. Cystic fibrosis is an autosomal recessive disorder due to a defect in the CFTR gene, leading to a deficiency in a chloride channel that secretes chloride in the lungs and gastrointestinal tract and reabsorbs chloride in sweat glands. This abnormal chloride transport causes decreased chloride and water secretion and increased water reabsorption, leading to abnormally viscous mucous in the lungs and gastrointestinal tract. The thick mucus in the gastrointestinal tract contributes to pancreatic insufficiency, steatorrhea, and malabsorption, leading to a deficiency of fat-soluble vitamins A, D, E, and K. Vitamin K plays a critical role in the synthesis of hepatic coagulation proteins; it is oxidized in the liver during carboxylation of glutamic acid residues on coagulation factors II, VII, IX, X, and proteins Č and S. A deficiency in vitamin K leads to coagulopathy with elevated prothrombin time (PT) and activated partial thromboplastin time (APTT) and is characterized by easy bruising. Incorrect Answers: A, B, C, D, E, and G. Antithrombin Il (Choice A) deficiency can be inherited or acquired secondary to nephrotic syndrome. Antithrombin III functions to inhibit coagulation factors II, VII, IX, X, XI, and XII. Deficiency leads to hypercoagulability. Platelets (Choice B) function in primary hemostasis in which they adhere to areas of endothelial damage and form a temporary platelet plug to stop bleeding prior to the activation of the coagulation cascade. Qualitative or quantitative platelet defects are all characterized by increased bleeding time, and generally present with prolonged mucous membrane bleeding (eg, epistaxis, menorrhagia), petechiae, or purpura. Protein C (Choice C) inactivates factors Va and VIlla in the coagulation cascade, and deficiency leads to hypercoagulability, not easy bruising. Vitamin B12 (coba


Conjuntos de estudio relacionados

California Real Estate Principles Chapter 11: Real Estate Taxation

View Set

Solving Linear Equations: Variables on Both Sides

View Set

Life policy riders, provisions, options and exclusions

View Set

Meiosis and Male Reproductive System

View Set